Re: [obm-l] Matriz de Vandermonde

2002-06-26 Por tôpico Johann Dirichlet

Duda,eu me lembro de que uma matriz e nao inversivel
se e so se for singular,ou seja, seu determinante for
0.Entao o que voce quer provar e que se os t's forem
diferentes o determinante nao e zero.Se eu nao me
engano ha uma formula para a matriz de Vandermonde que
so usa as diferenças entre os t's.Se voce conseguir
acha-la(deve ter em qualquer livro sobre
isso),COMEMORE
Peterdirichlet


 --- Eduardo Casagrande Stabel [EMAIL PROTECTED]
escreveu:  Ola pessoal da lista!
 
 Uma matriz de Vandermonde é uma matriz P da forma
 P_(i,j) = [t_(i-1)]^j onde i e j estão entre 0 e n
 um jeito mais explicito é o seguinte
 P =
 [ 1  t_0  (t_0)^2  (t_0)^3 ...  (t_0)^n  ]
 [ 1  t_1  (t_1)^2  (t_1)^3 ...  (t_1)^n  ]
 [ ...]
 [ 1  t_n  (t_n)^2  (t_n)^3  ...  (t_n)^n ]
 
 Eu não estou conseguindo demonstrar que se os t_i's
 são todos distintos
 então a matriz P é inversível.
 
 Alguém demonstra?
 
 Obrigado pela futura ajuda
 
 Eduardo Casagrande Stabel. Porto Alegre, RS.
 

=
 Instruções para entrar na lista, sair da lista e
 usar a lista em
 http://www.mat.puc-rio.br/~nicolau/olimp/obm-l.html
 O administrador desta lista é
 [EMAIL PROTECTED]

= 

___
Copa 2002
Yahoo! - Patrocinador oficial da Copa do Mundo da FIFA 2002
http://br.sports.yahoo.com/fifaworldcup/
=
Instruções para entrar na lista, sair da lista e usar a lista em
http://www.mat.puc-rio.br/~nicolau/olimp/obm-l.html
O administrador desta lista é [EMAIL PROTECTED]
=



Re: [obm-l] dois problemas

2002-07-03 Por tôpico Johann Dirichlet

Este problema 1 ja e famoso.Eu resolvo com
trigonometria.Seja x=anguloCQT.SLS no QCT,   
2*sen 60=TQ*sen .No PAT,PT=2/cos x.Pela
equilateralidade,tg x=sen 60.E como
x=anguloPTA(prove!),PT e facil de ser calculado e
vale 7^1/2.Com isso voce finaliza a questao.
Te mais




 --- Luis Lopes [EMAIL PROTECTED] escreveu:
 Sauda,c~oes,
 
 Acabo de receber estes dois problemas
 por fax. Alguém saberia resolvê-los?
 
 1) No triângulo ABC desenhado abaixo,
 A=90, C=60,AC=4.
 
B
 
 
 
   PQ
 
 
 
   ATC
 
 T é ponto médio de AC
 
 O triângulo PQT é equilátero. Calcule a área
 do círculo circunscrito ao triângulo PQT.
 


=
TRANSIRE SVVM PECTVS MVNDOQVE POTIRE
CONGREGATI EX TOTO ORBE MATHEMATICI OB SCRIPTA INSIGNIA TRIBUERE
Fields Medal(john Charles Fields)

___
Yahoo! Encontros
O lugar certo para encontrar a sua alma gêmea.
http://br.encontros.yahoo.com/
=
Instruções para entrar na lista, sair da lista e usar a lista em
http://www.mat.puc-rio.br/~nicolau/olimp/obm-l.html
O administrador desta lista é [EMAIL PROTECTED]
=



Re: [obm-l] Racionalização

2002-07-03 Por tôpico Johann Dirichlet


 --- [EMAIL PROTECTED] escreveu: 
Estava resolvendo algumas questões do
 selecionados, e me deparei com algumas
 dúvidas de teoria.
 *Como faço para racionalizar denominadores com
 mais de 3 raízes ?
 Exemplo simples :
1/[sqrt(2) + sqrt(3) + sqrt(5)]
 
 *Como faço para racionalizar denominadores com
 mais de uma raiz , do tipo
 :
 1/[raiz4(2) + 1 ]
 Será que a relação 
 1/[raiz n (a^p)] = raiz n (a^p - 1)/raiz n (a^p
 - 1) é válida ?
 
 *A relação do radical duplo , serve para raízes
 que não sejam quadradas
 ?
 Ex:
 raiz 5 [2 + raiz 3(3)]
 
 Obrigado.
 
Pelo que eu saiba,a maioria das questoes de
racionalizaçao se relaciona com o fato de que os
denominadores sao algebricos(RAIZES DE EQUACOES
POLINOMIAIS DE COEFICIENTES INTEIROS).
Vou exemplificar:2^1/2+5^1/2=x.Veja que 
x^2=7+2*(10)^1/2,e (x^2-7)^2=40,   
x^4-14*x^2+9=0.Assim sendo,x(x^3-14*x)=-9
Logo x^3-14*x e racionalizante de x.
Te mais!Peter Gustav 

=
TRANSIRE SVVM PECTVS MVNDOQVE POTIRE
CONGREGATI EX TOTO ORBE MATHEMATICI OB SCRIPTA INSIGNIA TRIBUERE
Fields Medal(john Charles Fields)

___
Yahoo! Encontros
O lugar certo para encontrar a sua alma gêmea.
http://br.encontros.yahoo.com/
=
Instruções para entrar na lista, sair da lista e usar a lista em
http://www.mat.puc-rio.br/~nicolau/olimp/obm-l.html
O administrador desta lista é [EMAIL PROTECTED]
=



Re: [obm-l] Infinitos

2002-07-03 Por tôpico Johann Dirichlet

 --- Gabriel_Pérgola [EMAIL PROTECTED]
escreveu:  Pergunta: Quantas numeros tem de 0 a
1 ?
 Resposta: Infinitos numeros...
 
 Pergunta: Quantos numeros tem de 0 a 10 ?
 Resposta: Infinitos numeros...
 
 Aonde tem mais numeros, de 0 a 1 ou de 0 a 10?
 
 Gabriel
 
Para começar,podemos dizer que encontraremos
tantos numeros em um conjunto quanto em outro.
Usando os cardinais de Cantor(dois conjuntos tem
o mesmo numero de elementos se existir uma
bijeçao entre eles),estamos diante dessa
definiçao.
Em bons livros de Conjuntos vc tem + info sobre isso.

___
Yahoo! Encontros
O lugar certo para encontrar a sua alma gêmea.
http://br.encontros.yahoo.com/
=
Instruções para entrar na lista, sair da lista e usar a lista em
http://www.mat.puc-rio.br/~nicolau/olimp/obm-l.html
O administrador desta lista é [EMAIL PROTECTED]
=



Re: [obm-l] Re: [obm-l] Racionalização

2002-07-05 Por tôpico Johann Dirichlet

Ah,essa historia de poder usar o fato
(a+b)(a-b)=(a^2-b^2) vale sim.A ideia e
racionalizar um por vez.
E essa de radical duplo,voce precisa de novas
regras para fazer isso.
Fui claro?
[],Peterdirichlet
--- [EMAIL PROTECTED] escreveu:  
 
 -- Mensagem original --
 
 
  --- [EMAIL PROTECTED] escreveu:
 
 Estava resolvendo algumas questões do
  selecionados, e me deparei com algumas
  dúvidas de teoria.
  *Como faço para racionalizar denominadores
 com
  mais de 3 raízes ?
  Exemplo simples :
 1/[sqrt(2) + sqrt(3) + sqrt(5)]
  
  *Como faço para racionalizar denominadores
 com
  mais de uma raiz , do tipo
  :
  1/[raiz4(2) + 1 ]
  Será que a relação 
  1/[raiz n (a^p)] = raiz n (a^p - 1)/raiz n
 (a^p
  - 1) é válida ?
  
  *A relação do radical duplo , serve para
 raízes
  que não sejam quadradas
  ?
  Ex:
  raiz 5 [2 + raiz 3(3)]
  
  Obrigado.
  
 Pelo que eu saiba,a maioria das questoes de
 racionalizaçao se relaciona com o fato de que
 os
 denominadores sao algebricos(RAIZES DE
 EQUACOES
 POLINOMIAIS DE COEFICIENTES INTEIROS).
 Vou exemplificar:2^1/2+5^1/2=x.Veja que 
 x^2=7+2*(10)^1/2,e (x^2-7)^2=40,  
 
 x^4-14*x^2+9=0.Assim sendo,x(x^3-14*x)=-9
 Logo x^3-14*x e racionalizante de x.
 Te mais!Peter Gustav 
 
 =
 Obrigado pela correção , mais eu estou com
 dúvidas com racionalização de
 denominadores ,foi o que eu exemplifiquei logo
 na primeira dúvida que eu
 tive.
 será que você poderia me ajudar , ou alguém da
 lista , eu queria saber ,
 se eu posso fazer uma coisa tipo isso .
 
 1/[sqrt(2) + sqrt(3) + sqrt(5)]= 
 
 1/[sqrt(2) + sqrt(3) + sqrt(5)]x [sqrt(2) -
 sqrt(3) - sqrt(5)]/[sqrt(2)
 - sqrt(3) - sqrt(5)]
 
 Será que me inteudeu ?
 
 
 
 E queria saber também , se a relação do radical
 duplo , vale para raízes
 que não sejam quadradas??
 
 Agradeço qualquer ajuda .
 Abraço.
 Rick.
 
 
 
   
  |-=Rick-C.R.B.=- |
  |ICQ 124805654   |
  |e-mail [EMAIL PROTECTED]  |
   
 
 
 --
 Use o melhor sistema de busca da Internet
 Radar UOL - http://www.radaruol.com.br
 
 
 

=
 Instruções para entrar na lista, sair da lista
 e usar a lista em

http://www.mat.puc-rio.br/~nicolau/olimp/obm-l.html
 O administrador desta lista é
 [EMAIL PROTECTED]

= 

___
Yahoo! Encontros
O lugar certo para encontrar a sua alma gêmea.
http://br.encontros.yahoo.com/
=
Instruções para entrar na lista, sair da lista e usar a lista em
http://www.mat.puc-rio.br/~nicolau/olimp/obm-l.html
O administrador desta lista é [EMAIL PROTECTED]
=



Re: [obm-l] Re: [obm-l] Re: Racionalização

2002-07-10 Por tôpico Johann Dirichlet

Eu ja tinha dito isso antes,nao?
Peterdirichlet


 --- Carlos Yuzo Shine [EMAIL PROTECTED]
escreveu:  Na verdade, pode-se racionalizar o
denominador
 de
 qualquer fração com denominador algébrico. Um
 número é
 algébrico se e somente se é raiz de um
 polinômio de
 coeficientes inteiros.
 
 A idéia é a seguinte: digamos que queremos
 racionalizar 1/a, onde a é algébrico.
 Encontramos um
 polinômio de coeficientes inteiros p(x) que
 admite a
 como raiz e fazemos:
 
 p(a) = 0 = p(a) - p(0) = p(0)
  = [p(a) - p(0)]/[ap(0)] = 1/a
 
 Como p(0) é o coeficiente independente de p(x),
 
 p(x)-p(0) é divisível por x, e obtemos um
 polinômio de
 coeficientes inteiros.
 
 Exemplificando: se a = sqrt(2) + sqrt(3) +
 sqrt(5),
 temos que a^2 = 9 + 2(sqrt(6) + sqrt(10) +
 sqrt(15)).
 Elevando mais uma vez ao quadrado (tenha fé!),
 temos
 
 (a^2-9)^2 = 4(31 +
 2(sqrt(60)+sqrt(90)+sqrt(150)))
   = 124 +
 2sqrt(30)(sqrt(2)+sqrt(3)+sqrt(5))
   = 124 + 2sqrt(30)*a
 
 Logo a^4 - 18a^2 + 81 = 124 + 2sqrt(30)a =
  a^4 - 18a^2 - 43 = 2sqrt(30)a
 
 A gente poderia elevar ao quadrado mais uma
 vez, mas
 não vai ser necessário. Veja:
 
 a^4 - 18a^2 - 43 = 2sqrt(30)a
 = a^3 - 18a - 2sqrt(30) = 43/a
 = [a^3 - 18a - 2sqrt(30)]/43 = 1/a.
 
 Pronto, está racionalizado (vc pode substituir
 a no
 numerador, mas estou contente assim). OK, deu
 mais
 trabalho que a outra solução, mas agora vc pode
 racionalizar frações mais complicadas, como
 (18^(1/3)
 + 12^(1/3) - 1)^(-1), por exemplo (ou até mesmo
 coisas
 mais estranhas como 1/cos(pi/9)!!). Tente!
 
 []'s
 Shine
 
 --- [EMAIL PROTECTED] wrote:
  
  Obrigado , amigo Davidson .
  Abraço.
  Rick
  -- Mensagem original --
  
 
  Parece que houve problemas, com o
 arquivo em
  anexo que enviei.
  
  Mas a idéia é a seguinte: multiplica-se
 o
  numerador e o denominador
  por:
   3*(2)^(1/2) + 2*(3)^(1/2) - (30)^(1/2). Que
  resultarar em: (3*(2)^(1/2)
  + 2*(3)^(1/2) - (30)^(1/2))/12.
  
 Felicidades.
  
 Davidson Estanislau
  
  
  -Mensagem Original- 
  De: Davidson Estanislau 
  Para: obm 
  Enviada em: Sexta-feira, 5 de Julho de 2002
 16:34
  Assunto: [obm-l] Re: Racionalização
  
  
  
 Olá luiz! Espero que esteja tudo bem com
 você.
  Veja como fiz:
  
  
  
  
 Felicidades!
  
 Davidson Estanislau
  
  
  -Mensagem Original- 
  De: [EMAIL PROTECTED]
  Para: [EMAIL PROTECTED]
  Enviada em: Terça-feira, 2 de Julho de 2002
 23:29
  Assunto: [obm-l] Racionalização
  
  
  Estava resolvendo algumas questões do
 selecionados,
  e me deparei com algumas
  dúvidas de teoria.
  *Como faço para racionalizar denominadores
 com mais
  de 3 raízes ?
  Exemplo simples :
 1/[sqrt(2) + sqrt(3) + sqrt(5)]
  
  *Como faço para racionalizar denominadores
 com mais
  de uma raiz , do tipo
  :
  1/[raiz4(2) + 1 ]
  Será que a relação 
  1/[raiz n (a^p)] = raiz n (a^p - 1)/raiz n
 (a^p -
  1) é válida ?
  
  *A relação do radical duplo , serve para
 raízes que
  não sejam quadradas
  ?
  Ex:
  raiz 5 [2 + raiz 3(3)]
  
  Obrigado.
  
  

   |-=Rick-C.R.B.=- |
   |ICQ 124805654   |
   |e-mail [EMAIL PROTECTED]  |
 
  
  

   |-=Rick-C.R.B.=- |
   |ICQ 124805654   |
   |e-mail [EMAIL PROTECTED]  |

  
  
  --
  Use o melhor sistema de busca da Internet
  Radar UOL - http://www.radaruol.com.br
  
  
  
 

=
  Instruções para entrar na lista, sair da
 lista e
  usar a lista em
 

http://www.mat.puc-rio.br/~nicolau/olimp/obm-l.html
  O administrador desta lista é
  [EMAIL PROTECTED]
 

=
 
 

__
 Do You Yahoo!?
 Sign up for SBC Yahoo! Dial - First Month Free
 http://sbc.yahoo.com

=
 Instruções para entrar na lista, sair da lista
 e usar a lista em

http://www.mat.puc-rio.br/~nicolau/olimp/obm-l.html
 O administrador desta lista é
 [EMAIL PROTECTED]

= 

___
Yahoo! Encontros
O lugar certo para encontrar a sua alma gêmea.
http://br.encontros.yahoo.com/
=
Instruções para entrar na lista, sair da lista e usar a lista em
http://www.mat.puc-rio.br/~nicolau/olimp/obm-l.html
O administrador desta lista é [EMAIL PROTECTED]
=



Re: [obm-l] Re:_[obm-l]_teoria_dos_números

2002-07-26 Por tôpico Johann Dirichlet

Esse teorema e bem fraco perto do Postulado de
Bertrand(ha  um primo entre n e 2n,n natural
positivo),cuja demonstraçao e longa e pode ser
achada na pagina da casa da OBM,no artigo de
Bruno Leite,prata da casa na OBM.


 --- Eduardo Casagrande Stabel
[EMAIL PROTECTED] escreveu:  Oi Korshinoi,
 
 um jeito é o seguinte.
 Sejam p_1 p_2 ... p_k todos os primos
 menores ou iguais a n, o número
 (p_1p_2p_3...p_k)+1 não é divisível por nenhum
 dos p_i's e é maior que n,
 caso contrário ele seria um primo menor que n e
 haveria um número a mais na
 nossa lista (absurdo!), portanto:
 n  (p_1p_2p_3...p_k) + 1 = (2.3k) + 1 =
 (n-1)! + 1  n!, para n=3
 portanto ou (p_1p_2p_3...p_k) + 1 é primo ou
 ele é produto de primos maiores
 que n. Ou seja, existe pelo menos um primo
 entre n e n!.
 
 Existem estimativas bem melhores que essa. Por
 exemplo, existe sempre primo
 entre n e 2n, isso é um teorema. Existe primo
 entre n^2 e (n+1)^2, essa é
 conjectura, pelo que disse o Nicolau uma vez.
 
 Era essa que você tinha em mente?
 
 Eduardo.
 Poa, RS.
 
 
 
 From: [EMAIL PROTECTED]
  Fiz uma demonstração baseada em certas
 argumentaçõesgostaria de saber
 se alguem tem uma demonstração formal do que
 segue abaixo. Agradeço
 antecipadamente quem puder demonstrar.
  Prove que entre n e n! existe um primo p(
 n=2)
Korshinoi
 

=
  Instruções para entrar na lista, sair da
 lista e usar a lista em
 

http://www.mat.puc-rio.br/~nicolau/olimp/obm-l.html
  O administrador desta lista é
 [EMAIL PROTECTED]
 

=
 
 
 

=
 Instruções para entrar na lista, sair da lista
 e usar a lista em

http://www.mat.puc-rio.br/~nicolau/olimp/obm-l.html
 O administrador desta lista é
 [EMAIL PROTECTED]

= 

___
Yahoo! PageBuilder
O super editor para criação de sites: é grátis, fácil e rápido.
http://br.geocities.yahoo.com/v/pb.html
=
Instruções para entrar na lista, sair da lista e usar a lista em
http://www.mat.puc-rio.br/~nicolau/olimp/obm-l.html
O administrador desta lista é [EMAIL PROTECTED]
=



Re: [obm-l] Re: [obm-l] Primeira questão obm ano passado

2002-07-29 Por tôpico Johann Dirichlet

A soluçao por medias eu conheço;e quase trivial.


 --- Marcelo Souza [EMAIL PROTECTED]
escreveu:  
 A ideia do Lucas não me parece bem simples
 assim. O que ele fez foi Usar o 
 semi-perímetro (no caso S) e a área (P) de um
 triângulo de lados 
 (a+b),(b+c),(a+c)...a solução é bem bonita,
 fica imediata até se vc desenhar 
 o triângulo, não é trivial esta idéia, mas eh
 uma boa tecnica para 
 desigualdades supor que sao lados de um
 triangulo.
 []'s
 Marcelo
 
 From: Fernanda Medeiros
 [EMAIL PROTECTED]
 Reply-To: [EMAIL PROTECTED]
 To: [EMAIL PROTECTED]
 Subject: [obm-l] Re: [obm-l] Primeira questão
 obm ano passado
 Date: Sat, 27 Jul 2002 21:35:43 +
 
   Oi Duda!
 Bem, eu fiz assim:
 desenvolvendo fica a(a+b+c) +bc
 =2sqrt[abc(a+b+c)] pela desigualdade das 
 medias! :)
   té
 []´s
 Fê!
 
 Lembram daquela desigualdade, sendo a,b,c0
 prove
 (a + b)(a + c) = 2raiz(abc(a+b+c)).
 
 Olhem essa solução que o Lucas Mocelim me
 apresentou.
 Chame S=a+b+c e P=abc
 (a + b)(a + c) =
 (S - c)(S - b) =
 S^2 - (b + c)S + bc =
 S^2 - (S - a)S + P/a =
 Sa + P/a = 2raiz(SaP/a) = 2raiz(SP)
 Só isso, não é muito mais fácil que a solução
 da Eureka!?
 Pena que na hora ele não percebeu...
 
 Um abraço!
 Duda.
 
 PS David Turchick, valeu pela correção da
 questão da imo, agora eu já
 compreendi.
 

=
 Instruções para entrar na lista, sair da
 lista e usar a lista em

http://www.mat.puc-rio.br/~nicolau/olimp/obm-l.html
 O administrador desta lista é
 [EMAIL PROTECTED]

=
 
 
 
 

_
 MSN Photos é a maneira mais fácil e prática de
 editar e compartilhar sua 
 fotos: http://photos.msn.com.br
 

=
 Instruções para entrar na lista, sair da lista
 e usar a lista em

http://www.mat.puc-rio.br/~nicolau/olimp/obm-l.html
 O administrador desta lista é
 [EMAIL PROTECTED]

=
 
 

_
 Chat with friends online, try MSN Messenger:
 http://messenger.msn.com
 

=
 Instruções para entrar na lista, sair da lista
 e usar a lista em

http://www.mat.puc-rio.br/~nicolau/olimp/obm-l.html
 O administrador desta lista é
 [EMAIL PROTECTED]

= 

___
Yahoo! PageBuilder
O super editor para criação de sites: é grátis, fácil e rápido.
http://br.geocities.yahoo.com/v/pb.html
=
Instruções para entrar na lista, sair da lista e usar a lista em
http://www.mat.puc-rio.br/~nicolau/olimp/obm-l.html
O administrador desta lista é [EMAIL PROTECTED]
=



RE: [obm-l] IMO dia 1, Q1 (solucao?!)

2002-07-29 Por tôpico Johann Dirichlet

Peguei as provas em PS e PDF da IMO.Se alguem
puder me dizercomo eu faço para escrever um
arquivo PS sendo que eu so tenho os
visualizadores. E eu consegui fazer apenas o
problema 2 desta IMO(geometria cearense sem do
nem piedade.Estilo problema 1 da IMO da Coreia.
  

--- Ralph Teixeira [EMAIL PROTECTED] escreveu: 
Estah correto...
 
 Mas soh para voces terem uma ideia de como
 o pessoal lah era rigoroso,
 esta solucao valeria 6 pontos.
 
 O pequeno detalhe que estah faltando eh o
 seguinte. NO caso (2),
 dividimos a inducao em T_{k} e T_{n-k} e, por
 inducao acabou, certo? Bom,
 nao exatamente... Note que poderiamos ter k=0
 ou k=n, e um dos triangulos
 simplesmente nao teria ponto algum. Entao estah
 faltando uma das duas
 coisas:
 
 (i) Ou voce cita o caso T_{0}
 explicitamente e nota que tambem vale a
 tal proposicao (voce soh citou T_1 e T_2)...
 (ii) ...ou voce separa o caso 2 em 2(a)
 (que vira dois triangulos) e
 este caso especial (onde ha de fato um
 triangulo soh T_{n}).
 
 Eles nao queriam uma demonstracao
 complicada destas coisas, que sao de
 fato obvias. O que eles querem eh uma *mencao*
 de que este caso (o
 triangulo vazio) existia e nao se enquadrava
 perfeitamente na inducao. No
 criterio de correcao, nao fazer o caso T_0 era
 um erro mais ou menos
 semelhante a esquecer o caso inicial de uma
 inducao... e por isso perdia-se
 um ponto (o que explica a grande quantidade de
 6 desta questao).
 
 Abraco,
  Ralph
 
 
 -Original Message-
 From: Marcio
 To: [EMAIL PROTECTED]
 Sent: 7/27/02 9:18 AM
 Subject: [obm-l] IMO dia 1, Q1 (solucao?!)
 
 Me ajudem a detectar possiveis falhas nessa
 solucao!
 
 Traducao : Seja n  0 inteiro.  Seja T_n o
 conjunto dos ptos (x,y) do
 plano com x,y inteiros nao negativos e x+y  n.
 Cada pto de T eh pintado
 de
 R ou B. Se (x,y) eh R, entao tmb o serao tds os
 ptos (x',y') de Tcom x'
 = x
 e y'=y. Defina uma X-set como um conjunto de n
 ptos azuis com
 coordenadas x
 distintas, e uma
 Y-set como um conjunto de n ptos azuis com
 coordenadas y distintas.
 Prove
 que o nr de X-sets eh igual ao nr de Y-sets.
 
 Minha solucao foi por inducao na seguinte
 proposicao:
 Se a n-upla P = (p0, p1, ..., p_(n-1) ) da a
 qtd de ptos pintados de B
 nas
 retas x=0, x=1, ..., x=n-1 (respectivamente),
 entao a qtd de B's nas
 retas
 y=0, y=1, ..., y=n-1 nessa ordem eh dada por
 uma permutacao de P. (em
 particular nr de X-sets = nr de Y-sets =
 Produtorio de p_i).
 
 Em 1o lugar, note que se (x,y)=B, entao (x',
 y') = B sempre que x'=x ou
 y'=y.
 
 Para n=1, n=2 eh soh considerar todos os
 (poucos) casos possiveis e
 confirmar que eh verdade.
 Suponha valido para inteiros menores ou iguais
 a n, e consideremos o
 caso
 n+1.
 
 1) Se #X eh nao nulo, entao toda a diagonal
 externa x+y=n eh B (de fato,
 se
 (a,n-a) = R, entao todos abaixo dele sao R e
 nessa reta x=a nao existe
 nenhum pto B).
 Apagando essa diagonal, note que o que sobre eh
 uma configuracao valida
 em
 T_n e portanto, se nessa configuracao temos P =
 (p0, p1, ..., p_(n-1) )
 B's
 nas retas x=0,1,...,n-1, teremos /P =
 permutacao de P B's nas retas
 y=0,...
 Reescrevendo a diagonal soh de B's, teremos
 P'=(p0+1, p1+1, ..., p_(n-1)
 +
 1, 1) associada a qtd de ptos pintados de B nas
 retas x=0, x=1,... x=n e
 /P'
 = (elementos de /P somados de 1 unidade, com 1
 no final), donde /P' eh
 uma
 permutacao de P'.
 
 2) Se #X eh nulo, entao existe k tq a reta x=k
 soh tem R. Apagando o
 retangulo de vertices
 (0,0)-(k,0)-(k,n-k)-(0,n-k), ficamos com uma
 configuracao valida de
 T_(k)
 (considerada sobre um novo eixo transladado em
 relacao ao original e com
 centro em (0, n-k+1) e outra de T_(n-k)
 (...centro em (k+1,0) ) nas
 quais
 podemos aplicar a hipotese de inducao e
 proceder como em (1).
 
 Isso conclui a inducao e o problema.
 
 Abracos,
 Marcio
 
 PS: Tmb tentei o problema 3, mas o melhor que
 eu consegui foi verificar
 que
 se a divisao vale para infinitos inteiros,
 entao o polinomio do
 denominador
 (em a) deve dividir o polinomio do numerador..
 Depois devo tentar os
 problemas do 2o dia..
 


 =
 Instruções para entrar na lista, sair da lista
 e usar a lista em

http://www.mat.puc-rio.br/~nicolau/olimp/obm-l.html
 O administrador desta lista é
 [EMAIL PROTECTED]


 =

=
 Instruções para entrar na lista, sair da lista
 e usar a lista em

http://www.mat.puc-rio.br/~nicolau/olimp/obm-l.html
 O administrador desta lista é
 [EMAIL PROTECTED]

= 

___
Yahoo! PageBuilder
O super editor para criação de sites: é grátis, fácil e rápido.
http://br.geocities.yahoo.com/v/pb.html

Re: [obm-l] Re: [obm-l] Noticias da IMO

2002-07-29 Por tôpico Johann Dirichlet

E que ninguem discuta!!!Afinal,quando a
primeira garota a tirar medalha na IMO(leia-se
Larissa)vai ser entrevistada pela galera do
teorema?




 --- [EMAIL PROTECTED] escreveu:  
 
 A menina é féra  :o P
 
   
  |-=Rick-C.R.B.=- |
  |ICQ 124805654   |
  |e-mail [EMAIL PROTECTED]  |
   
 
 
 --
 Use o melhor sistema de busca da Internet
 Radar UOL - http://www.radaruol.com.br
 
 
 

=
 Instruções para entrar na lista, sair da lista
 e usar a lista em

http://www.mat.puc-rio.br/~nicolau/olimp/obm-l.html
 O administrador desta lista é
 [EMAIL PROTECTED]

= 

___
Yahoo! PageBuilder
O super editor para criação de sites: é grátis, fácil e rápido.
http://br.geocities.yahoo.com/v/pb.html
=
Instruções para entrar na lista, sair da lista e usar a lista em
http://www.mat.puc-rio.br/~nicolau/olimp/obm-l.html
O administrador desta lista é [EMAIL PROTECTED]
=



Re: [obm-l] IMO dia 1, Q2 (solucao)(comentario de JOHANN)

2002-07-29 Por tôpico Johann Dirichlet

Mas tu e um porre hein Cohen??
Pra que complexos se da pra fazer com Geometria
Cearense(marcar angulos ate se cansar)?
Essa soluçao e parecida com a do Daniel Uno para
a questao 1 da IMOP da Coreia(veja Eureka 9 no
site da OBM). 

Eu nao vou passar a soluçao integral que eu
fiz.Mas essas dicas ja dao conta do recado.
Prove que os triangulos AOF e AOE sao
equilateros.
Chame angBOE=4x e calcule todos os angulos em
funçao de x.Desenhe o ponto I incentro de CEF(que
deve ser o J,certo?).Provaremos que
angDAO=angAOI,o que acarreta o paralelismo. 
Veja que CA e bissetriz de angECF,logo I
e encontro de CA e a bissetriz de angEFC.
Agora faça o arrastao(marque tudo que e angulo)e 
prontoE so ver depois de infindaveis
contas que OAJ e isosceles.
Como brincadeira prove que EOJF e ciclico(olhe
para AO,AJ,...
 --- Marcio [EMAIL PROTECTED] escreveu:  Essa
eh para fortalecer os numeros complexos (o
 enunciado traduzido esta
 no final).
 Q2)  Vou usar a' para representar o conjugado
 de a. Os lemas abaixo sao
 usados toda hora em problemas de geometria, e
 por isso eu os coloquei em
 evidencia.
 
 1. Suponha, spg, q o circunraio de ABC eh 1.
 Ponha B=-1, C=1, A=a^2 =
 cis(2x); a = cis(x),
 com 30x90. Note que a' = 1/a, (a^2)'=1/(a^2).
 Lema1: Se x e y sao pontos do circulo unitario,
 a reta que os une tem eq. z
 + (xy)z' = x+y.
 Lema2: Os pontos medios dos arcos formados
 pelos complexos a^2 e b^2 sao ab
 e -ab (de fato, se m eh medio,
 arg(m)-arg(a)=arg(b)-arg(m) = m/a=b/m).
 
 2. Determinacao do ponto J:
 Ponto D: arg(D) = (180+2x)/2 = 90+x donde d =
 cis(90)*cisx = ia
 Reta AD: z + (ia^3)z' = ...
 Reta OJ (//AD passando pela origem):   z +
 (ia^3)z' = 0(1)
 Reta AC:   
z + (a^2)z' = a^2 + 1
 (2)
 Resolvendo as eqs (1) e (2), encontramos o
 ponto J: z = a(a-i)
 
 3. Determinando E,F:
 Temos |z-a^2|=|z| (esta na mediatriz) e zz' = 1
 (esta na circunferencia),
 logo
 (z-a^2)(z' - 1/a^2) = zz'
 Simplificando, z^2 - (a^2)z + a^4 = 0.
 Usando baskara ou multiplicando os 2 lados por
 z+a^2, obtemos p. ex:
 e = (a^2)cis(60)  e
 f = (a^2)cis(-60)
 
 4. Ponto medio do arco CF:
 m=acis(-30)
 (note que, como x30, esse ponto eh de fato o
 que esta entre C e F, pois
 argm = x-300).
 
 5. J esta na bissetriz EM:
 Eh soh ver que J verifica a eq. da reta EM: z +
 (a^3)cis(30)z' = (a^2)cis60
 + acis(-30) .
 Eh soh substituir z=a(a-i), z'=(1+ia)/(a^2) e
 ver que os coeficientes de a e
 de a^2 sao iguais dos dois lados.
 
 6. J esta na bissetriz de C:
 O pto medio do arco EF que nao contem C eh
 sqrt(e*f) = a^2 = A. Logo, a
 bissetriz de C eh exatamente a reta CA, donde J
 esta nessa bissetriz. (essa
 parte ateh eu consegui fazer por plana :)
 
 7. Logo, J pertence a duas bissetrizes, e
 portanto eh o incentro.
 
 Estive tentando fazer as questoes do primeiro
 dia da prova.. Comecei pela 2,
 que achei mais facil, e depois tentei a
 primeira.. parei de tentar a 3 agora
 pq nao estava produzindo muita coisa.. Nos
 proximos dois emails vou mandar
 minhas ideias/solucoes.. Mandem as suas tmb!
 
 Abracos,
 Marcio
 
 - Original Message -
 From: Ralph Teixeira [EMAIL PROTECTED]
 To: 'Rodrigo Villard Milet '
 [EMAIL PROTECTED]; 'Obm '
 [EMAIL PROTECTED]
 Sent: Friday, July 26, 2002 10:10 AM
 Subject: RE: [obm-l] IMO!?!?
 
 
  Let \ $BC$ be a diameter of the circle
 ${\Gamma}$ with
  centre $O$. \ Let $A$ be a point on $\Gamma$
 such that $0{{}^\circ
  }\angle AOB120{{}^\circ}$. \ Let $D$ be the
 midpoint of the
  arc $AB$ not containing $C$. \ The line
 through $O$ parallel
  to $DA$ meets the line $AC$ at $J$. \ The
 perpendicular bisector
  of $OA$ meets $\Gamma$ at $E$ and at $F$. \
 Prove that $J$ is
  the incentre of the triangle $CEF$.
 
 Traducao: Seja BC diametro de um
 circunferencia com centro O. Seja A um
 pto da circunferencia com AOB120 graus. Seja D
 medio do arco AB que nao
 contem C. A reta por O paralela a DA encontra
 AC em J. A mediatriz de OA
 encontra a circunferencia em E e F. Mostre que
 J eh incentro do triangulo
 CEF.
 
 

=
 Instruções para entrar na lista, sair da lista
 e usar a lista em

http://www.mat.puc-rio.br/~nicolau/olimp/obm-l.html
 O administrador desta lista é
 [EMAIL PROTECTED]

= 

___
Yahoo! PageBuilder
O super editor para criação de sites: é grátis, fácil e rápido.
http://br.geocities.yahoo.com/v/pb.html
=
Instruções para entrar na lista, sair da lista e usar a lista em
http://www.mat.puc-rio.br/~nicolau/olimp/obm-l.html
O administrador desta lista é [EMAIL PROTECTED]
=



RE: [obm-l] IMO dia 1, Q1 (solucao?!)

2002-07-29 Por tôpico Johann Dirichlet

 --- Johann Dirichlet
[EMAIL PROTECTED] escreveu:  Peguei
as provas em PS e PDF da IMO.Se alguem
 puder me dizercomo eu faço para escrever um
 arquivo PS sendo que eu so tenho os
 visualizadores. E eu consegui fazer apenas o
 problema 2 desta IMO(geometria cearense sem do
 nem piedade.Estilo problema 1 da IMO da Coreia.
   Ao contrario do Cohen.Mas afinal de onde e
que ele teve a ideia de tirar complexos ali?
 
 --- Ralph Teixeira [EMAIL PROTECTED] escreveu:   
  
 Estah correto...
  
  Mas soh para voces terem uma ideia de
 como
  o pessoal lah era rigoroso,
  esta solucao valeria 6 pontos.
  
  O pequeno detalhe que estah faltando eh o
  seguinte. NO caso (2),
  dividimos a inducao em T_{k} e T_{n-k} e, por
  inducao acabou, certo? Bom,
  nao exatamente... Note que poderiamos ter k=0
  ou k=n, e um dos triangulos
  simplesmente nao teria ponto algum. Entao
 estah
  faltando uma das duas
  coisas:
  
  (i) Ou voce cita o caso T_{0}
  explicitamente e nota que tambem vale a
  tal proposicao (voce soh citou T_1 e T_2)...
  (ii) ...ou voce separa o caso 2 em 2(a)
  (que vira dois triangulos) e
  este caso especial (onde ha de fato um
  triangulo soh T_{n}).
  
  Eles nao queriam uma demonstracao
  complicada destas coisas, que sao de
  fato obvias. O que eles querem eh uma
 *mencao*
  de que este caso (o
  triangulo vazio) existia e nao se
 enquadrava
  perfeitamente na inducao. No
  criterio de correcao, nao fazer o caso T_0
 era
  um erro mais ou menos
  semelhante a esquecer o caso inicial de uma
  inducao... e por isso perdia-se
  um ponto (o que explica a grande quantidade
 de
  6 desta questao).
  
  Abraco,
   Ralph
  
  
  -Original Message-
  From: Marcio
  To: [EMAIL PROTECTED]
  Sent: 7/27/02 9:18 AM
  Subject: [obm-l] IMO dia 1, Q1 (solucao?!)
  
  Me ajudem a detectar possiveis falhas nessa
  solucao!
  
  Traducao : Seja n  0 inteiro.  Seja T_n o
  conjunto dos ptos (x,y) do
  plano com x,y inteiros nao negativos e x+y 
 n.
  Cada pto de T eh pintado
  de
  R ou B. Se (x,y) eh R, entao tmb o serao tds
 os
  ptos (x',y') de Tcom x'
  = x
  e y'=y. Defina uma X-set como um conjunto de
 n
  ptos azuis com
  coordenadas x
  distintas, e uma
  Y-set como um conjunto de n ptos azuis com
  coordenadas y distintas.
  Prove
  que o nr de X-sets eh igual ao nr de Y-sets.
  
  Minha solucao foi por inducao na seguinte
  proposicao:
  Se a n-upla P = (p0, p1, ..., p_(n-1) ) da a
  qtd de ptos pintados de B
  nas
  retas x=0, x=1, ..., x=n-1 (respectivamente),
  entao a qtd de B's nas
  retas
  y=0, y=1, ..., y=n-1 nessa ordem eh dada por
  uma permutacao de P. (em
  particular nr de X-sets = nr de Y-sets =
  Produtorio de p_i).
  
  Em 1o lugar, note que se (x,y)=B, entao (x',
  y') = B sempre que x'=x ou
  y'=y.
  
  Para n=1, n=2 eh soh considerar todos os
  (poucos) casos possiveis e
  confirmar que eh verdade.
  Suponha valido para inteiros menores ou
 iguais
  a n, e consideremos o
  caso
  n+1.
  
  1) Se #X eh nao nulo, entao toda a diagonal
  externa x+y=n eh B (de fato,
  se
  (a,n-a) = R, entao todos abaixo dele sao R e
  nessa reta x=a nao existe
  nenhum pto B).
  Apagando essa diagonal, note que o que sobre
 eh
  uma configuracao valida
  em
  T_n e portanto, se nessa configuracao temos P
 =
  (p0, p1, ..., p_(n-1) )
  B's
  nas retas x=0,1,...,n-1, teremos /P =
  permutacao de P B's nas retas
  y=0,...
  Reescrevendo a diagonal soh de B's, teremos
  P'=(p0+1, p1+1, ..., p_(n-1)
  +
  1, 1) associada a qtd de ptos pintados de B
 nas
  retas x=0, x=1,... x=n e
  /P'
  = (elementos de /P somados de 1 unidade, com
 1
  no final), donde /P' eh
  uma
  permutacao de P'.
  
  2) Se #X eh nulo, entao existe k tq a reta
 x=k
  soh tem R. Apagando o
  retangulo de vertices
  (0,0)-(k,0)-(k,n-k)-(0,n-k), ficamos com uma
  configuracao valida de
  T_(k)
  (considerada sobre um novo eixo transladado
 em
  relacao ao original e com
  centro em (0, n-k+1) e outra de T_(n-k)
  (...centro em (k+1,0) ) nas
  quais
  podemos aplicar a hipotese de inducao e
  proceder como em (1).
  
  Isso conclui a inducao e o problema.
  
  Abracos,
  Marcio
  
  PS: Tmb tentei o problema 3, mas o melhor que
  eu consegui foi verificar
  que
  se a divisao vale para infinitos inteiros,
  entao o polinomio do
  denominador
  (em a) deve dividir o polinomio do
 numerador..
  Depois devo tentar os
  problemas do 2o dia..
  
 


  =
  Instruções para entrar na lista, sair da
 lista
  e usar a lista em
 

http://www.mat.puc-rio.br/~nicolau/olimp/obm-l.html
  O administrador desta lista é
  [EMAIL PROTECTED]
 


  =
 

=
  Instruções para entrar na lista, sair da
 lista
  e usar a lista em
 

http://www.mat.puc-rio.br/~nicolau/olimp/obm-l.html
  O administrador desta lista é

Re: [obm-l] Precisamos de 2 Solucoes!

2002-07-29 Por tôpico Johann Dirichlet

Oi NellyJa mandei a soluçao do problema
63,ta? --- Olimpiada Brasileira de Matematica
[EMAIL PROTECTED] escreveu:  Caros(as) amigos(as) da
lista,
 
 Estamos trabalhando na Revista Eureka No. 14.
 Para a secao de problemas propostos estao
 faltando
 as solucoes dos problemas 63 e 66 publicados na
 Revista Eureka! No. 12, se alguem quiser
 colaborar
 pode enviar as suas solucoes via e-mail para:
 [EMAIL PROTECTED](qualquer formato) ou via fax 
 para 21-25295023. 
 
 Abracos, Nelly.
 
 

=
 Instruções para entrar na lista, sair da lista
 e usar a lista em

http://www.mat.puc-rio.br/~nicolau/olimp/obm-l.html
 O administrador desta lista é
 [EMAIL PROTECTED]

= 

___
Yahoo! PageBuilder
O super editor para criação de sites: é grátis, fácil e rápido.
http://br.geocities.yahoo.com/v/pb.html
=
Instruções para entrar na lista, sair da lista e usar a lista em
http://www.mat.puc-rio.br/~nicolau/olimp/obm-l.html
O administrador desta lista é [EMAIL PROTECTED]
=



Re: [obm-l] Discussao dos problemas da IMO

2002-07-29 Por tôpico Johann Dirichlet

 --- Olimpiada Brasileira de Matematica
[EMAIL PROTECTED] escreveu: Caros colegas,
Por sugestao do Marcio vamos fazer uma
 reuniao informal na sexta-feira
 (2/8) as 14:00 no IMPA para discutir os
 problemas da IMO deste ano.Tragam
 suas solucoes...
Abracos,
Carlos Gustavo Moreira (Gugu)
  
 Mas gente,eu sou de Sao Paulo,ta?Se voces
puderem fazer algo com computadores,Internet e
tal,te que da.

=
 Instruções para entrar na lista, sair da lista
 e usar a lista em

http://www.mat.puc-rio.br/~nicolau/olimp/obm-l.html
 O administrador desta lista é
 [EMAIL PROTECTED]

= 

___
Yahoo! PageBuilder
O super editor para criação de sites: é grátis, fácil e rápido.
http://br.geocities.yahoo.com/v/pb.html
=
Instruções para entrar na lista, sair da lista e usar a lista em
http://www.mat.puc-rio.br/~nicolau/olimp/obm-l.html
O administrador desta lista é [EMAIL PROTECTED]
=



Re de uma re: [obm-l] Discussao dos problemas da IMO

2002-07-30 Por tôpico Johann Dirichlet

Que tal a gente fazer isso pela Internet?Da ate
pra colocar as melhores soluçoes na revista
Eureka!,mais ou menos como os problemas
propostos.Como a prova estara na revista 14,todos
se reunem pelas listas obm-l e pelo Teoremalista
pra discutir.E as melhores soluçoes vao pra
Eureka!Pensem um pouco nisso.
Te mais
Peterdirichlet
PS:Parabens a delegaçao brasileira da IMO e seus
123 pontos!!
:):):):):):):):):):):):):):):):):):):):):):):):):)

 Caros colegas,
 Por sugestao do Marcio vamos fazer uma
 reuniao informal na sexta-feira
 (2/8) as 14:00 no IMPA para discutir os
 problemas da IMO deste ano.Tragam
 suas solucoes...
 Abracos,
 Carlos Gustavo Moreira (Gugu)
 
 

=
 Instruções para entrar na lista, sair da lista
 e usar a lista em

http://www.mat.puc-rio.br/~nicolau/olimp/obm-l.html
 O administrador desta lista é
 [EMAIL PROTECTED]

=
 

=
 Instruções para entrar na lista, sair da lista
 e usar a lista em

http://www.mat.puc-rio.br/~nicolau/olimp/obm-l.html
 O administrador desta lista é
 [EMAIL PROTECTED]

= 

___
Yahoo! PageBuilder
O super editor para criação de sites: é grátis, fácil e rápido.
http://br.geocities.yahoo.com/v/pb.html
=
Instruções para entrar na lista, sair da lista e usar a lista em
http://www.mat.puc-rio.br/~nicolau/olimp/obm-l.html
O administrador desta lista é [EMAIL PROTECTED]
=



Re: [obm-l] Problema 5 da IMO 2002

2002-07-31 Por tôpico Johann Dirichlet

Oi turma da OBMEu vou enviar minha soluçao
para esse problema.Ela e bem parecida com a do
Gugu.O Villard obteve outra sem apelando pra
limites.Ela vai pro final deste e-mail.


 --- Carlos Gustavo Tamm de Araujo Moreira
[EMAIL PROTECTED] escreveu: Caros colegas,
Segue (depois de um bom espaco) uma solucao
 para o problema 5.
Abracos,
Gugu
 
Ache todas as funçoes f:R=R tais que para todos
os x,y,z,w reais,se tenha
 
 
 
 
 
 
 
 
 
 
 
 
 
 
 
 
 
 
 
 
 
 
 
 
 
 
 
 
 
 
 
 
 
 
 
 
 
 
 
 
 
 
 
 
 
 
 
 
 
 
 
 
 
 
 
 
 
 
 
 
 
 
 
 
 
 
 
 
 
 
 
 
 
 
 
 
 
 
Fazendo t=y=0 e x=z temos
 4f(0)f(x)=2f(0),donde f(0)=0 ou f(x)=1/2 para
 todo x (o que da' uma solucao).A seguir
 suporemos entao f(0)=0.
Fazendo z=y=t=x,temos
 4f(x)^2=f(0)+f(2.x^2),logo
 f(2.x^2)=4.f(x)^2.Assim,
 se u=0,f(u)=4.f(rquad(u/2))^2=0. [rquad
 denota raiz quadrada]
Fazendo x=t=0,temos f(y)f(z)=f(yz),e fazendo
 x=y=0,temos f(z)f(t)=f(-zt).
 Assim,f(u)=f(u.1)=f(u)f(1)=f(-u.1)=f(-u),ou
 seja,f e' uma funcao par,e,pela
 observacao anterior,f(u)=0 para todo u.Basta
 agora calcular f(u) para u=0.
Fazendo x=t=u e y=z=v,temos
 (f(u)+f(v))^2=f(u^2+v^2)=f(rquad(u^2+v^2))^2.
 Assim,se 0=x=y,fazendo z=rquad(y^2-x^2),temos

f(x)^2=(f(x)+f(z))^2=f(rquad(x^2+z^2))^2=f(y)^2,logo
 f(x)=f(y),ou seja,
 f e' monotona nao-decrescente em R+.
Temos f(1)=f(1.1)=f(1).f(1).Assim,f(1)=1 ou
 f(1)=0.No segundo caso,temos
 f(x)=f(x.1)=f(x).f(1)=0 para todo x,o que
 tambem da' uma solucao.Vamos supor
 entao que f(1)=1.Vamos mostrar que para  n
 natural temos f(nz)=n^2.f(z) para
 todo z,por inducao.De fato isso vale para n=0 e
 n=1.Fazendo y=t=1,temos

2(f(x)+f(z))=2f(1)(f(x)+f(z))=f(x-z)+f(x+z).Fazendo
 x=nz,temos,para n=1,

2(n^2.f(z)+f(z))=2(f(nz)+f(z))=f((n-1)z)+f((n+1)z)=(n-1)^2.f(z)+f((n+1)z),
 donde

f((n+1)z)=(2(n^2+1)-(n-1)^2).f(z)=(n+1)^2.f(z),cqd.
Fazendo z=1,temos f(n)=n^2.f(1)=n^2 para
 todo n natural,e fazendo z=p/q
 com p e q inteiros positivos temos
 p^2=f(p)=f(q.p/q)=q^2.f(p/q),donde
 f(p/q)=(p/q)^2.Assim,f(x)=x^2 para todo x
 racional positivo.Vamos mostrar
 que de fato f(x)=x^2 para todo x real positivo
 (e portanto para todo x real,
 pois se x e' negativo,-x e' positivo e teriamos
 f(x)=f(-x)=(-x)^2=x^2).
 Suponha que nao,i.e.,que existe x positivo tal
 que f(x) e' diferente de x^2.
 Vamos supor que f(x)  x^2 (o outro caso e'
 analogo).Seja p/q um racional
 tal que x  p/q  rquad (f(x)). Devemos ter
 f(x)=f(p/q),mas 
 f(p/q)=(p/q)^2f(x),absurdo.
Assim,as unicas funcoes f que satisfazem o
 enunciado sao:f(x)=1/2 para
 todo x,f(x)=0 para todo x e f(x)=x^2 para todo
 x.
Abracos,
Gugu
 
Segue a minha solução para a quinta questão dessa
IMO. Confiram :),( se alguém tiver paciência ).
(f(x)+f(z))*(f(y)+f(t)) = f(xy-zt) + f(xt+yz)
Primeiramente faça x=z=0 : 2f(0) * ( f(y) + f(t)
) = 2f(0), logo ou f(0)=0, ou f(y)+f(t) = 1, para
todos y,t reais e em particular quando y=t, temos
f(y)=1/2, para todo y real, o que é uma solução
particular. Assuma então f(0)=0.
Faça z=t=0 : f(xy)=f(x)*f(y). Então, fazendo y=1,
f(x)=f(x)*f(1), logo temos f(1)=1 ou f(x)=0, para
todo x, o que é outra solução particular. Faça
y=t=1 e x=0 : 2f(z) = f(-z)+f(z), logo f é par.
Então, precisamos nos preocupar apenas com a
parte positiva.
Na equação inicial, temos :
f(xy)+f(xt)+f(yz)+f(zt) = f(xy-zt) + f(xt+yz),
então f(a)+f(b)+f(c)+f(d)=f(a-d)+f(b+c), desde
que ad=bc. Colocando a=c=mx, b=d=x , temos
2f(mx)+2f(x)=f((m-1)x) + f((m+1)x), o que prova
por indução que f(mx)=m^2 * f(x), para todo m
inteiro. Logo f(m)=m^2, para todo m inteiro.
f(p/q)=f(p)/f(q), pois é multiplicativa, logo
f(p/q)=(p/q)^2, então f(m)=m^2, para todo m
racional. Vamos mostrar que f é monótona
crescente em R+. Faça y=x,t=z : (f(x)+f(z))^2 =
f(x^2-z^2)+f(2xy). Faça y=z,t=x :
(f(x)+f(z))^2=f(x^2+z^2). Juntando as duas
últimas, temos que f(a)=f(b)+f(c), quando a^2 =
b^2+c^2, logo f é crescente (pois se ab0, então
existe c0 tal que a^2=b^2+c^2, logo
f(a)=f(b)+f(c) f(b), pois f(c)=f(sqrt(c))^2=0.
Em particular f é positiva.).
Como f é crescente em R+ e f(m)=m^2 nos
racionais, então é fácil mostrar que f(x)=x^2
para todos x em R+, logo para todos x em R, pois
f é par. Sejam a(n) e b(n) duas sequências de
racionais convergindo para um irracional x0,
tais que 0a(n) x b(n). Então como f é monótona
crescente, a(n)^2 = f(x) = b(n)^2 e fazendo n
tender a infinito, temos a(n) e b(n) tendendo a
x, logo f(x)=x^2 também nos irracionais.
Resposta : f(x)=0 ; f(x)=1/2 ; f(x)=x^2 .
 
Abraços, 
 Villard


Esta e a minha soluçao.
A ideia de certos problemas funcionais e a
simetria.Vamos ver isso.
Pondo y=z,t=x,
(f(x)+f(z))^2=f(x^2+z^2).
Pondo x=y,z=t, temos 
(f(x)+f(z))^2=f(x^2-z^2)+f(2xz)
Com z=0,temos
4((f(x))^2)=f(2x^2)+f(0)
Na original,pondo x=z,t=y,
4f(x)f(y)=f(0)+f(2xy).
Pondo y=0,ja obtemos algumas coisas:
4f(x)f(0)=2f(0),logo ou f(0)=0 ou f(x)=1/2 para
todo x real. 
Suporemos f(0)=0.Assim sendo podemos continuar.
4f(x)f(y)=f(2xy)
A partir daqui podemos 

Re: [obm-l] Discussao dos problemas da IMO(para toda sao Paulo

2002-08-01 Por tôpico Johann Dirichlet

sera que voces do Rio nao pensam em nos de Sampa,
que nao tem nem como se encontrar pra isso?Se eu
pudesse sugeriria ao Edmilson que fizesse algo
assim no Etapa ou no IME-Usp.,E ai o que voces
acham?

Johann Peter Gustav Lejeune Dirichlet
 --- Rodrigo Villard Milet [EMAIL PROTECTED]
escreveu:  Essa idéia de fazer em dois dias é
boa, pois
 cada um tem sua disponibilidade
 de horários... eu só posso na sexta...
 Abraços,
  Villard
 -Mensagem original-
 De: Carlos Gustavo Tamm de Araujo Moreira
 [EMAIL PROTECTED]
 Para: [EMAIL PROTECTED]
 [EMAIL PROTECTED]
 Data: Terça-feira, 30 de Julho de 2002 15:41
 Assunto: Re: [obm-l] Discussao dos problemas da
 IMO
 
 
 Eu tinha proposto na sexta por sugestao do
 Marcio.O Marcelo estava no
 IMPA e disse que tambem preferia sexta.Eu nao
 tenho nenhum problema na
 segunda,entretanto.Talvez seja bom o pessoal
 do Rio se manifestar sobre que
 dia prefere.Por outro lado nao vejo problema
 em fazer uma reuniao na sexta
 e
 outra na segunda,e discutir tambem outros
 problemas,alem dos da IMO,para
 aproveitar a animacao do pessoal.O que voces
 acham ?
 Abracos,
 Gugu
 
 
 Eu posso participar se for na segunda-feira.
 Na sexta é mais difícil.
 
 Luciano.
 
 At 15:12 29/07/02 -0300, you wrote:
 Caros colegas,
 Por sugestao do Marcio vamos fazer uma
 reuniao informal na
 sexta-feira
 (2/8) as 14:00 no IMPA para discutir os
 problemas da IMO deste ano.Tragam
 suas solucoes...
 Abracos,
 Carlos Gustavo Moreira (Gugu)
 
 

=
 Instruções para entrar na lista, sair da
 lista e usar a lista em

http://www.mat.puc-rio.br/~nicolau/olimp/obm-l.html
 O administrador desta lista é
 [EMAIL PROTECTED]

=
 

=
 Instruções para entrar na lista, sair da
 lista e usar a lista em

http://www.mat.puc-rio.br/~nicolau/olimp/obm-l.html
 O administrador desta lista é
 [EMAIL PROTECTED]

=
 

=
 Instruções para entrar na lista, sair da lista
 e usar a lista em

http://www.mat.puc-rio.br/~nicolau/olimp/obm-l.html
 O administrador desta lista é
 [EMAIL PROTECTED]

=
 
 

=
 Instruções para entrar na lista, sair da lista
 e usar a lista em

http://www.mat.puc-rio.br/~nicolau/olimp/obm-l.html
 O administrador desta lista é
 [EMAIL PROTECTED]

=



___
Yahoo! PageBuilder
O super editor para criação de sites: é grátis, fácil e rápido.
http://br.geocities.yahoo.com/v/pb.html
=
Instruções para entrar na lista, sair da lista e usar a lista em
http://www.mat.puc-rio.br/~nicolau/olimp/obm-l.html
O administrador desta lista é [EMAIL PROTECTED]
=



[obm-l] Problema2,IMO2002

2002-08-01 Por tôpico Johann Dirichlet

Essa soluçao e do porre emocional
COMPLEXado:):):):) do Cohen.Ja que e assim,vamos
dar nossa contribuiçao ao Ceara.A minha resposta
esta no final. 

Seja BC diametro de um
 circunferencia com centro O. Seja A um
 pto da circunferencia com AOB120 graus. Seja D
 medio do arco AB que nao
 contem C. A reta por O paralela a DA encontra
 AC em J. A mediatriz de OA
 encontra a circunferencia em E e F. Mostre que
 J eh incentro do triangulo
 CEF.


























De Marcio Afonso Assad Cohen
  Vou usar a' para representar o conjugado
 de a. Os lemas abaixo sao
 usados toda hora em problemas de geometria, e
 por isso eu os coloquei em
 evidencia.
 
 1. Suponha, spg, q o circunraio de ABC eh 1.
 Ponha B=-1, C=1, A=a^2 =
 cis(2x); a = cis(x),
 com 30x90. Note que a' = 1/a, (a^2)'=1/(a^2).
 Lema1: Se x e y sao pontos do circulo unitario,
 a reta que os une tem eq. z
 + (xy)z' = x+y.
 Lema2: Os pontos medios dos arcos formados
 pelos complexos a^2 e b^2 sao ab
 e -ab (de fato, se m eh medio,
 arg(m)-arg(a)=arg(b)-arg(m) = m/a=b/m).
 
 2. Determinacao do ponto J:
 Ponto D: arg(D) = (180+2x)/2 = 90+x donde d =
 cis(90)*cisx = ia
 Reta AD: z + (ia^3)z' = ...
 Reta OJ (//AD passando pela origem):   z +
 (ia^3)z' = 0(1)
 Reta AC:   
z + (a^2)z' = a^2 + 1
 (2)
 Resolvendo as eqs (1) e (2), encontramos o
 ponto J: z = a(a-i)
 
 3. Determinando E,F:
 Temos |z-a^2|=|z| (esta na mediatriz) e zz' = 1
 (esta na circunferencia),
 logo
 (z-a^2)(z' - 1/a^2) = zz'
 Simplificando, z^2 - (a^2)z + a^4 = 0.
 Usando baskara ou multiplicando os 2 lados por
 z+a^2, obtemos p. ex:
 e = (a^2)cis(60)  e
 f = (a^2)cis(-60)
 
 4. Ponto medio do arco CF:
 m=acis(-30)
 (note que, como x30, esse ponto eh de fato o
 que esta entre C e F, pois
 argm = x-300).
 
 5. J esta na bissetriz EM:
 Eh soh ver que J verifica a eq. da reta EM: z +
 (a^3)cis(30)z' = (a^2)cis60
 + acis(-30) .
 Eh soh substituir z=a(a-i), z'=(1+ia)/(a^2) e
 ver que os coeficientes de a e
 de a^2 sao iguais dos dois lados.
 
 6. J esta na bissetriz de C:
 O pto medio do arco EF que nao contem C eh
 sqrt(e*f) = a^2 = A. Logo, a
 bissetriz de C eh exatamente a reta CA, donde J
 esta nessa bissetriz. (essa
 parte ateh eu consegui fazer por plana :)
 
 7. Logo, J pertence a duas bissetrizes, e
 portanto eh o incentro.
 
 
Soluçao de peterdirichlet.

Ceara na cabeça

Para se divertir:prove que EOJF e ciclico.
Os triangulos AOF e AOE sao
equilateros(oras,AO=FO=EO porque sao raios,e
AF=FO,AE=EO(mediatriz).
Provaremos o seguinte:se J for mesmo esse desdito
incentro,entao AD e OJ sao paralelos.
CA e bissetriz de angECF(oras,AE e AF medem a
mesma coisa).Logo J e o encontro da bissetriz de
angCFE e CA.Seja angEOB=4q.Esse 4q veio porque
tive que codificar esse e-mail decentemente.
Entao angEAB=angECB=angOEC=2q(oras,EOC e
isosceles,angulos inscritos e so).
arcoAD=arcoDB,logo angAOD=angDOB,logo
AOD=30+2q,angDOE=30-2q.
60=angEAO=angEAB+angBAO=2q+angBAO,
logo angBAO=60-2q.BC e diametro,logo angBAC=90,e
angOAC=30+2q.Como angOAF=60,angCAF=30-2q,e
angFEC=30-2q(mesmo arco).E angEFC=angEAC=90+22.
FJ e bissetriz,angJFC=angEFJ=45+q,e
angOFJ=15+q.Por Angulo
Externo,angAJF=angJFC+angFCJ=75+q.Logo os
triangulos JAF e OAJ sao isosceles.Como
angOAJ=30+2q,entao angAOJ=75-q.
Como angDOE=30-2q,angEAD=15-q.E angDAO=75-q,que
por acaso e angAOJ.Logo por paralelismo(AO
transversal),acabou!
Mais umna do paulista do Ceara,Peter Gustav
Lejeune Dirichlet.

=
TRANSIRE SVVM PECTVS MVNDOQVE POTIRE
CONGREGATI EX TOTO ORBE MATHEMATICI OB SCRIPTA INSIGNIA TRIBUERE
Fields Medal(john Charles Fields)

___
Yahoo! PageBuilder
O super editor para criação de sites: é grátis, fácil e rápido.
http://br.geocities.yahoo.com/v/pb.html
=
Instruções para entrar na lista, sair da lista e usar a lista em
http://www.mat.puc-rio.br/~nicolau/olimp/obm-l.html
O administrador desta lista é [EMAIL PROTECTED]
=



Re: [obm-l] Artigo de Curiosidades

2010-02-02 Por tôpico Johann Dirichlet
Uma dica? Eis:
http://mathworld.wolfram.com

Em 1 de fevereiro de 2010 17:52, Albert Bouskela bousk...@msn.com escreveu:
 Olá!



 Sei que nesta Lista e, também, na Internet, você encontrará inúmeras
 curiosidades bem legais. Eu, particularmente, vou lhe sugerir duas:



 1)    A Lei de Benford (Primazia do 1º Dígito), cuja demonstração posso lhe
 enviar; e

 2)    A Hipótese do Continuum de Cantor.



 Adicionalmente, o Número Áureo tem propriedades bastante interessantes.



 Albert Bouskela

 bousk...@msn.com



 De: owner-ob...@mat.puc-rio.br [mailto:owner-ob...@mat.puc-rio.br] Em nome
 de Thiago Tarraf Varella
 Enviada em: segunda-feira, 1 de fevereiro de 2010 14:14
 Para: OBM Lista
 Assunto: [obm-l] Artigo de Curiosidades



 Muito obrigado a todos! Eu estou escrevendo um artigo de curiosidades
 matemáticas, e estou pesquisando essas curiosidades na internet, para
 prová-las no meu artigo. Se souberem de alguma coisa legal para eu incluir,
 eu ia agradecer!
 Se vocês quiserem, posso mandar quando ficar pronto para voces verem...
 Abraços,
 Thiago

 

 Quer navegar na Internet sem medo? Instale grátis o Internet Explorer 8.



-- 
/**/
Quadrinista e Taverneiro!

http://tavernadofimdomundo.blogspot.com  Histórias, Poemas, Quadrinhos e Afins
http://baratoeletrico.blogspot.com / Ativismo Digital (?)
http://bridget-torres.blogspot.com/  Personal! Do not edit!

=
Instru��es para entrar na lista, sair da lista e usar a lista em
http://www.mat.puc-rio.br/~obmlistas/obm-l.html
=


Re: [obm-l] Desigualdade de Cauchy e um problema

2010-02-05 Por tôpico Johann Dirichlet
Em 5 de fevereiro de 2010 11:27, marcone augusto araújo borges
marconeborge...@hotmail.com escreveu:
 Tentei usar a desiguldade de Cauchy para resolver o seguinte problema:sejam
 x,y,z números reais positivos satisfazendo x+y+z=raiz(xyz).Prove q xy+yz+xz
= 9(x+y+z).
LaTeX-mode

(\sqrt{x/yz}+\sqrt{y/xz}+\sqrt{z/xy}) *
(\sqrt{yz/x}+\sqrt{xz/y}+\sqrt{xy/z}) = (1+1+1)^2=9

Como x+y+z=raiz(xyz), o primeiro produto vale 1:
(\sqrt{yz/x}+\sqrt{xz/y}+\sqrt{xy/z}) = 9
(yz+xz+xy) = 9 \sqrt{xyz} = 9(x+y+z), com igualdade se e só se x=y=z=9


 
 Faça compras on-line com mais segurança. Instale grátis o Internet Explorer
 8.



-- 
/**/
Quadrinista e Taverneiro!

http://tavernadofimdomundo.blogspot.com  Histórias, Poemas, Quadrinhos e Afins
http://baratoeletrico.blogspot.com / Ativismo Digital (?)
http://bridget-torres.blogspot.com/  Personal! Do not edit!

=
Instru��es para entrar na lista, sair da lista e usar a lista em
http://www.mat.puc-rio.br/~obmlistas/obm-l.html
=


Re: [obm-l] Desigualdade de Cauchy e um problema

2010-02-06 Por tôpico Johann Dirichlet
Em 5 de fevereiro de 2010 16:22, Francisco Barreto
fcostabarr...@gmail.com escreveu:
 Dá pra ver tex no e-mail?
Nope. Não até onde sei.
Estou usando apenas a notação matemática do TeX. Bem, é apenas um modo
de se escrever as fórmulas. Eu acho mais prárico que outras formas de
se escrever.
Caso precise urgentemente verificar, tente o LaTeX Previewer:
http://www.tlhiv.org/ltxpreview/
As fórmulas são, colocadas entre sinais de cifrão: $2+2=2^2$.

Caso precise aprender LaTeX, tem uns tutoriaos no MathLinks.ro

 Em 5 de fevereiro de 2010 14:13, Johann Dirichlet peterdirich...@gmail.com
 escreveu:

 Em 5 de fevereiro de 2010 11:27, marcone augusto araújo borges
 marconeborge...@hotmail.com escreveu:
  Tentei usar a desiguldade de Cauchy para resolver o seguinte
  problema:sejam
  x,y,z números reais positivos satisfazendo x+y+z=raiz(xyz).Prove q
  xy+yz+xz
 = 9(x+y+z).
 LaTeX-mode

 (\sqrt{x/yz}+\sqrt{y/xz}+\sqrt{z/xy}) *
 (\sqrt{yz/x}+\sqrt{xz/y}+\sqrt{xy/z}) = (1+1+1)^2=9

 Como x+y+z=raiz(xyz), o primeiro produto vale 1:
 (\sqrt{yz/x}+\sqrt{xz/y}+\sqrt{xy/z}) = 9
 (yz+xz+xy) = 9 \sqrt{xyz} = 9(x+y+z), com igualdade se e só se x=y=z=9


  
  Faça compras on-line com mais segurança. Instale grátis o Internet
  Explorer
  8.



 --
 /**/
 Quadrinista e Taverneiro!

 http://tavernadofimdomundo.blogspot.com  Histórias, Poemas, Quadrinhos e
 Afins
 http://baratoeletrico.blogspot.com / Ativismo Digital (?)
 http://bridget-torres.blogspot.com/  Personal! Do not edit!

 =
 Instru�ões para entrar na lista, sair da lista e usar a lista em
 http://www.mat.puc-rio.br/~obmlistas/obm-l.html
 =





-- 
/**/
Quadrinista e Taverneiro!

http://tavernadofimdomundo.blogspot.com  Histórias, Poemas, Quadrinhos e Afins
http://baratoeletrico.blogspot.com / Ativismo Digital (?)
http://bridget-torres.blogspot.com/  Personal! Do not edit!

=
Instru��es para entrar na lista, sair da lista e usar a lista em
http://www.mat.puc-rio.br/~obmlistas/obm-l.html
=


[obm-l] Re: [obm-l] RE: [obm-l] Como provar que PI é irraci onal?

2010-04-02 Por tôpico Johann Dirichlet
A melhor referência, IMHO, é o infame Proofs from THE BOOK, de
martin Aigner e Günter Ziegler. Tem um capítulo dedicado aos
irracionais, em especial pi e exp(n) para todo n (e alguns outros).

Sei que tem como folhear no Google Books, e que tem em algumas
faculdades de Matemática de São Paulo.

Em 25 de março de 2010 16:54, Maikel Andril Marcelino
maikinho0...@hotmail.com escreveu:
 DÊ UMA LIDA NESSE ARTIGO

 http://pt.wikipedia.org/wiki/Prova_da_irracionalidade_de_%CF%80

 From: fftone...@uol.com.br
 Subject: [obm-l] Como provar que PI é irracional?
 Date: Wed, 24 Mar 2010 13:37:08 -0300
 To: obm-l@mat.puc-rio.br

 Pessoal,
 Sou novo aqui. Meu nome é Felipe Tonello e faço matemática bacharelado na
 UNIFEI, Universidade Federal de Itajubá.

 Um professor veio com essa questão. Prove que PI é irracional.
 Eu tentei e tentei, tentei provar que a constante e é irracional também,
 mas sem sucesso.
 Acredito que falta alguma prática pra mim =/

 Alguém pode me ajudar?

 Obrigado

 Felipe Ferreri Tonello
 fftone...@uol.com.br
 http://felipetonello.com


 =
 Instruções para entrar na lista, sair da lista e usar a lista em
 http://www.mat.puc-rio.br/~obmlistas/obm-l.html
 =

 
 Fale com seus amigos do Messenger direto da Caixa de Entrada do Hotmail.
 Clique aqui



-- 
/**/
Quadrinista e Taverneiro!

http://tavernadofimdomundo.blogspot.com  Histórias, Poemas, Quadrinhos e Afins
http://baratoeletrico.blogspot.com / Ativismo Digital (?)
http://bridget-torres.blogspot.com/  Personal! Do not edit!

=
Instru��es para entrar na lista, sair da lista e usar a lista em
http://www.mat.puc-rio.br/~obmlistas/obm-l.html
=


[obm-l] Re: [obm-l] Números Quadrados

2010-04-10 Por tôpico Johann Dirichlet
N=x^2-y^2=(x-y)(x+y)
Creio que daqui dá pra colocar algumas restrições na fatoração. Por exemplo,
ambos os fatores devem ser de mesma paridade.

Em 9 de abril de 2010 09:42, luiz silva luizfelipec...@yahoo.com.brescreveu:

 Prezados,

 Estou precisando de uma ajuda no seguinte problema :

 De quantas maneiras um determinado número inteiros pode ser escrito como
 diferença de dois quadrados (inteiros tb)? Ou seja, De quantos ternos
 pitagóricos um número inteiro pode ser elemento, de modo que nunca seja o
 maior número do terno ?








-- 
/**/
Quadrinista e Taverneiro!

http://tavernadofimdomundo.blogspot.com  Histórias, Poemas, Quadrinhos e
Afins
http://baratoeletrico.blogspot.com / Ativismo Digital (?)
http://bridget-torres.blogspot.com/  Personal! Do not edit!


[obm-l] Re: [obm-l] RES: [obm-l] Números Primos

2010-04-24 Por tôpico Johann Dirichlet
Acho que van der Corput disse que existem infinitas PAs de três termos
todos primos. Mas epa!, o termo do meio é média aritmética dos outros
dois!

E creio que recentemente conseguirm melhorar: existem PAs
arbitrariamente grandes formadas somente de números primos.

Fuçando na Internet, achei esse site (não espico ínglichi):
http://mathworld.wolfram.com/PrimeArithmeticProgression.html

Ass.: Johann

Em 11 de abril de 2010 16:46, Artur Costa Steiner
steinerar...@gmail.com escreveu:
 Há vários exemplos de pares de números primos cuja média é um número primo.
 (17 + 5)/2 = 11
 (23 + 3)/2 = 13
 (11 + 23)/2 = 17
 (19 + 7)/2 = 13

 Um problema interessante é deduzir se o número de pares de primos (p1, p2)
 cuja média seja um número primo é finito ou infinito.

 Artur


 -Mensagem original-
 De: owner-ob...@mat.puc-rio.br [mailto:owner-ob...@mat.puc-rio.br] Em nome
 de Carlos Watanabe
 Enviada em: domingo, 11 de abril de 2010 01:51
 Para: obm-l@mat.puc-rio.br
 Assunto: Re: [obm-l] Números Primos

 Se for, eu preciso rever meus conceitos, pois 3, 5 e 7 são primos e
 (3+7)/2=5.
 Abraços,
 Carlos

 vitor alves escreveu:
 Como provar que a média aritmética de dois números primos nunca é um
 número primo?
 
 Quer ver seus e-mails de todas as contas num lugar só? Junte todas
 elas no Hotmail.

 http://www.windowslive.com.br/public/tip.aspx/view/16?product=1ocid=CRM-Wi
 ndowsLive:dicaPopAggregator:Tagline:WLCRM:On:WL:pt-BR:Hotmail

 __
 Fale com seus amigos  de graça com o novo Yahoo! Messenger
 http://br.messenger.yahoo.com/
 =
 Instruções para entrar na lista, sair da lista e usar a lista em
 http://www.mat.puc-rio.br/~obmlistas/obm-l.html
 =


 =
 Instruções para entrar na lista, sair da lista e usar a lista em
 http://www.mat.puc-rio.br/~obmlistas/obm-l.html
 =




-- 
/**/
Quadrinista e Taverneiro!

http://tavernadofimdomundo.blogspot.com  Histórias, Poemas, Quadrinhos e Afins
http://baratoeletrico.blogspot.com / Ativismo Digital (?)
http://bridget-torres.blogspot.com/  Personal! Do not edit!

=
Instru��es para entrar na lista, sair da lista e usar a lista em
http://www.mat.puc-rio.br/~obmlistas/obm-l.html
=


Re: [obm-l] Fatorial

2010-04-24 Por tôpico Johann Dirichlet
Puxa! Acho que já resolvi!

Seja P o maior primo menor que N (se N for primo, ninguém menor que
ele é múltiplo, logo seu expoente é 1 e o problema acaba).
Pelo Postulado de Bertrand, PN=2P. Portanto, P só aparece uma vez
entre os fatores de N!
E é isso!

Em 24 de abril de 2010 17:09, Johann Dirichlet
peterdirich...@gmail.com escreveu:
 Deixa eu ver se entendi: você quer todos os fatoriais que são
 quadrados perfeitos?

 Se for isso, apesar da minha preguiça extrema, darei uma dica:  é
 necessário que todos os fatores primos de n! tenham expoente par, e é
 fácil calcular tais expoentes. Se acharmos um ímpar, acabou.

 (Divagação: podemos tentar provar que existe pelo menos um primo entre
 1,2,3,...,n que aparece apenas uma vez (expoente 1). Por exemplo, de 1
 a 13, o 7 aparece só uma vez.
 Isso me faria pensar no Postulado de Bertrand, que diz que existe um
 primo entre n e 2n)

 Em 24 de abril de 2010 14:29, Felippe Coulbert Balbi
 felippeba...@hotmail.com escreveu:
 Olá.  Tenho um problema que eu empaquei em uma parte, espero que me ajudem.

 Prove que não existe solucoes inteiras para sqrt n! (raiz quadrada de n!)
 para n E Z / n1
 Obrigado
 Coulbert.
 
 Transforme-se em personagens engraçados e coloque no Messenger. Clique e
 veja como.



 --
 /**/
 Quadrinista e Taverneiro!

 http://tavernadofimdomundo.blogspot.com  Histórias, Poemas, Quadrinhos e 
 Afins
 http://baratoeletrico.blogspot.com / Ativismo Digital (?)
 http://bridget-torres.blogspot.com/  Personal! Do not edit!




-- 
/**/
Quadrinista e Taverneiro!

http://tavernadofimdomundo.blogspot.com  Histórias, Poemas, Quadrinhos e Afins
http://baratoeletrico.blogspot.com / Ativismo Digital (?)
http://bridget-torres.blogspot.com/  Personal! Do not edit!

=
Instru��es para entrar na lista, sair da lista e usar a lista em
http://www.mat.puc-rio.br/~obmlistas/obm-l.html
=


Re: [obm-l] Problema de geometria euclidiana

2010-04-24 Por tôpico Johann Dirichlet
Eu pensaria em uma rotação. Se girarmos esta figura de 120graus em
relação ao centro do triângulo maior, a imagem se encaixa. Usando
Complexos daria para formalizar melhor.

Em 24 de abril de 2010 10:05, Tiago hit0...@gmail.com escreveu:
 Olá!, obrigado pela resposta. Mas não entendi porque o ângulo EBC (vc
 escreveu EBD, mas acho que você quis dizer EBC) tem que ser igual ao BAD? O
 que você tá usando explicitamente pra chegar a esta conclusão?

 2010/4/23 Walter Tadeu Nogueira da Silveira wtade...@gmail.com

 Oi, Tiago

 Pensei assim. Chamando de x e 60º-x os ângulos menor e maior
 respectivamente ao vértice B, e se AD = BE, então o ângulo x EBD é igual
 ao BAD. (Lembre que AB = AC = BC)
 O ângulo FDE é externo a ABD e vale (x) + (60º - x) = 60º.

 Pela mesma razão saem ângulo ACF = BAD e DFE (externo vale 60º) e
 finalmente DEF é o suplementar.

 Creio ser isso.





 --
 Tiago J. Fonseca
 http://legauss.blogspot.com




-- 
/**/
Quadrinista e Taverneiro!

http://tavernadofimdomundo.blogspot.com  Histórias, Poemas, Quadrinhos e Afins
http://baratoeletrico.blogspot.com / Ativismo Digital (?)
http://bridget-torres.blogspot.com/  Personal! Do not edit!

=
Instru��es para entrar na lista, sair da lista e usar a lista em
http://www.mat.puc-rio.br/~obmlistas/obm-l.html
=


[obm-l] Que tal um grupo de resolucao de problemas?

2010-05-07 Por tôpico Johann Dirichlet
Ola povo!

Eu estou pensando em algo que pode ser interessante para a lista.
Que tal montar um grupo de resolucao de problemas de matematica? Por
exemplo, a Eureka! 31 tem uns problemas bem divertidos (eu resolvi
alguns deles).

A ideia e simples: postamos um problema, no seguinte formato:

Assunto do e-mail: Resolucao de problemas[Problema 255, Revista de
matematica olimpica de Krugerrandia no. 21]

Corpo da mensagem: o enunciado do problema (caso se precise, usar
LaTeX nas partes de matematica)

A partir dai, quem tiver ideias para uma solucao vai postando. Assim,
podemos testar mais ideias e teorias mais rapidamente. Caso se chegue
a uma solucao comprovada (100% sem furos), podemos escrever um ultimo
post, no seguinte formato:

Assunto do e-mail: Resolucao de problemas[Problema 255, Revista de
matematica olimpica de Krugerrandia no. 21][FECHADO]

Corpo da mensagem: o enunciado do problema e a sua solucao

Dai, alguem poderia simplesmente enviar para a revista; para as que
tem e-mail isso seria bem facil, mas pode-se pensatr em mandar
solucoes por carta tambem (isso ficaria para bem mais tarde)
Para o remetente, poderiamos por o nome de todos os contribuintes, ou
apenas algo como Grupo de Resolucao de problemas da OBM-L.

Pretendo comecar um teste assim que possivel, colocando um problema da
Eureka! 31.

E entao, o que acham?

-- 
/**/
Quadrinista e Taverneiro!

http://tavernadofimdomundo.blogspot.com  Histórias, Poemas, Quadrinhos e Afins
http://baratoeletrico.blogspot.com / Ativismo Digital (?)
http://bridget-torres.blogspot.com/  Personal! Do not edit!

=
Instru��es para entrar na lista, sair da lista e usar a lista em
http://www.mat.puc-rio.br/~obmlistas/obm-l.html
=


Re: [obm-l] Ajuda

2010-05-07 Por tôpico Johann Dirichlet
Vamos la: se r+c/100 e o preco, temos que z=104/100*(r+c/100) e inteiro.

Abrindo os denominadores, temos

1*z = 104*(100r+c)
Fatorando,
2^4*5^4*z = 8*13(100r+c) ou
2*5^4*z = 13*(100r+c)

Logo z, o preco com imposto, e multiplo de 13. Vamos colocar z=13a
1250*a = 100r+c ou 1250a-100r=c. Logo c e multiplo de 50. Escrevendo c =
50b, temos 25a-2r=b.

Como c e menor que 100 , podemos testar b=1 e ver r minimo:

25a-2r=1
Pondo a=1, teremos r=12 e ai o produto custaria 12,50. E e isso!


Em 7 de maio de 2010 10:02, Bruno Carvalho brunomos...@yahoo.com.brescreveu:

 *

 Peço uma pequena ajuda para a solução desses problemas.Grato,pessoal.

 abraço



 Bruno



 1)Na multiplicação abaixo, onde os algarismos a, b e c são desconhecidos, o
 valor da soma a+b+c é 14.

 (1abc)x 3 = abc4



 2)Um cidadão fixa o preço de um objeto em reais e centavos de real,de tal
 modo que, quando ele acrescenta 4% de imposto, o resultado é um número
 inteiro de reais.Qual o menor valor que esse objeto pode ter?
 *





-- 
/**/
Quadrinista e Taverneiro!

http://tavernadofimdomundo.blogspot.com  Histórias, Poemas, Quadrinhos e
Afins
http://baratoeletrico.blogspot.com / Ativismo Digital (?)
http://bridget-torres.blogspot.com/  Personal! Do not edit!


[obm-l] Resolucao de problemas[Problema 134, Eureka! 31]

2010-05-10 Por tôpico Johann Dirichlet
Considere  a operação . entre dois vetores do R^3 definida por:
(x,y,z) . (a,b,c) = (xa+yc+zb,xc+yb+za,xb+ya+zc)

Demonstre que para todo K0, se (x,y,z)^k=(0,0,0) entao (x,y,z)=(0,0,0)


-- 
/**/
Quadrinista e Taverneiro!

http://tavernadofimdomundo.blogspot.com  Histórias, Poemas, Quadrinhos e Afins
http://baratoeletrico.blogspot.com / Ativismo Digital (?)
http://bridget-torres.blogspot.com/  Personal! Do not edit!

=
Instru��es para entrar na lista, sair da lista e usar a lista em
http://www.mat.puc-rio.br/~obmlistas/obm-l.html
=


[obm-l] Resolucao de problemas[Problema 137, Eureka! 31]

2010-05-10 Por tôpico Johann Dirichlet
Seja A um conjunto de quinze pontos do plano, tais que dois deles não
se alinham com a origem e cuja distância à origenm seja no máximo 1

Demonstre que existem dois pontos tais que a área formada pelo
triângulo cujos vértices são estes dois pontos e a origem é menor que
1/4.

-- 
/**/
Quadrinista e Taverneiro!

http://tavernadofimdomundo.blogspot.com  Histórias, Poemas, Quadrinhos e Afins
http://baratoeletrico.blogspot.com / Ativismo Digital (?)
http://bridget-torres.blogspot.com/  Personal! Do not edit!

=
Instru��es para entrar na lista, sair da lista e usar a lista em
http://www.mat.puc-rio.br/~obmlistas/obm-l.html
=


Re: [obm-l] Resolucao de problemas[Problema 137, Eureka! 31]

2010-05-12 Por tôpico Johann Dirichlet
Em 12 de maio de 2010 09:29, Bernardo Freitas Paulo da Costa
bernardo...@gmail.com escreveu:
 2010/5/10 Johann Dirichlet peterdirich...@gmail.com:
 Seja A um conjunto de quinze pontos do plano, tais que dois deles não
 se alinham com a origem e cuja distância à origenm seja no máximo 1

 Demonstre que existem dois pontos tais que a área formada pelo
 triângulo cujos vértices são estes dois pontos e a origem é menor que
 1/4.

 Eu acho que deveria dar

 1/2 * sin(2*pi/15) = 0.2033683216  0.25

Consegui melhorar esta estimativa pela metade. Se pudermos refletir
cada ponto na origem, são 30 pontos. A ideia e que  area nao muda se
eu trocar um ponto refletido pelo ponto original. Entao a estimativa
fica em sin(pi/29) por Gavetas(aliás, por que chamam isso de Casa dos
Pombos, já que pigeonhole significa escaninho??).


 --
 Bernardo Freitas Paulo da Costa

 =
 Instruções para entrar na lista, sair da lista e usar a lista em
 http://www.mat.puc-rio.br/~obmlistas/obm-l.html
 =




-- 
/**/
Quadrinista e Taverneiro!

http://tavernadofimdomundo.blogspot.com  Histórias, Poemas, Quadrinhos e Afins
http://baratoeletrico.blogspot.com / Ativismo Digital (?)
http://bridget-torres.blogspot.com/  Personal! Do not edit!

=
Instru��es para entrar na lista, sair da lista e usar a lista em
http://www.mat.puc-rio.br/~obmlistas/obm-l.html
=


Re: [obm-l] vetores e baricentro

2010-05-12 Por tôpico Johann Dirichlet
Uma definição mais física seria: o baricentro é o centro de gravidade
de uma figura, supondo que ela fose feita de um material homogeneo.

Em 11 de maio de 2010 23:20, Ralph Teixeira ralp...@gmail.com escreveu:
 Bom, a minha definicao de baricentro eh vetorial: o baricentro do poligono
 A1A2...An eh o ponto correspondente ao vetor (A1+A2+A3+...+An)/n. Seria o
 centro de massa de um conjunto de n particulas de mesma massa colocadas nos
 vertices.

 Infelizmente (ou felizmente?), esta definicao eh virtualmente equivalente ao
 seu problema, pois as seguintes linhas sao equivalentes:

 SUM (G-Ai)=0
 SUM G = SUM Ai
 nG = SUM Ai
 G= (SUM Ai)/n
 (SUM eh somatorio, i=1 a n)

 Ajudou?

 Abraco,
     Ralph
 2010/5/11 Hermann ilhadepaqu...@bol.com.br

 Boa noite.

 Existe baricentro de um polígono?
 Se não. Perdoem minha ignorância.
 Se sim.
 Eis um exercício que gostaria de uma ajuda:

 Dado um polígono formado pelos pontos A1, A2, An. Provar que o Somatório
 dos vetores GAi = vetor nulo. Onde G é o baricentro do polígono.

 Muito obrigado
 Hermann




-- 
/**/
Quadrinista e Taverneiro!

http://tavernadofimdomundo.blogspot.com  Histórias, Poemas, Quadrinhos e Afins
http://baratoeletrico.blogspot.com / Ativismo Digital (?)
http://bridget-torres.blogspot.com/  Personal! Do not edit!

=
Instru��es para entrar na lista, sair da lista e usar a lista em
http://www.mat.puc-rio.br/~obmlistas/obm-l.html
=


[obm-l] Resolução de Problemas [Problema 138, Eureka! 31]

2010-05-12 Por tôpico Johann Dirichlet
Determine o maior divisor comum de todo os números da forma xyz, em
que x,y,z satisfazem a equação diofantina x^2+y^2=z^2.

-- 
/**/
Quadrinista e Taverneiro!

http://tavernadofimdomundo.blogspot.com  Histórias, Poemas, Quadrinhos e Afins
http://baratoeletrico.blogspot.com / Ativismo Digital (?)
http://bridget-torres.blogspot.com/  Personal! Do not edit!

=
Instru��es para entrar na lista, sair da lista e usar a lista em
http://www.mat.puc-rio.br/~obmlistas/obm-l.html
=


[obm-l] Re: [obm-l] Re: [obm-l] Resolução de Problemas [Pr oblema 138, Eureka! 31]

2010-05-17 Por tôpico Johann Dirichlet
Em 13 de maio de 2010 18:59, Willy George do Amaral Petrenko
wgapetre...@gmail.com escreveu:
 Tem aquela história de que os termos primitivos são da forma
 x = m^2 - n^2
 y = 2*m*n
 z = m^2 + n^2

Algumas correções:

1 - Multiplique todos por uma constante d arbitrária;
2 - MDC(m,n)=1 e um dos dois m,n é par

Isto produz todas as trincas pitagóricas. Mas o restante da sua
demonstração é independente deste detalhe.


 Que tinha uma demonstração que eu esqueci.
 Mas daí é fácil: x = (m-n)*(m+n).
 Um dos números m-n, m, m+n é múltiplo de 3.
 Se n ou m for par então y é múltiplo de 4, caso contrário x é multiplo de 4.
 Se m ou n for múltiplo de 5 então y é múltiplo de 5, se tiverem a mesma
 congruência módulo 5 ou simétricas (1 e -1, 2 e -2...) então x é múltiplo de
 4, e caso contrário (analisando os casos) z é múltiplo de 5.
 Como existe uma terna que é (3,4,5) então MMC é 3*4*5 = 60

 2010/5/12 Johann Dirichlet peterdirich...@gmail.com

 Determine o maior divisor comum de todo os números da forma xyz, em
 que x,y,z satisfazem a equação diofantina x^2+y^2=z^2.

 --
 /**/
 Quadrinista e Taverneiro!

 http://tavernadofimdomundo.blogspot.com  Histórias, Poemas, Quadrinhos
 e Afins
 http://baratoeletrico.blogspot.com / Ativismo Digital (?)
 http://bridget-torres.blogspot.com/  Personal! Do not edit!

 =
 Instruções para entrar na lista, sair da lista e usar a lista em
 http://www.mat.puc-rio.br/~obmlistas/obm-l.html
 =





-- 
/**/
Quadrinista e Taverneiro!

http://tavernadofimdomundo.blogspot.com  Histórias, Poemas, Quadrinhos e Afins
http://baratoeletrico.blogspot.com / Ativismo Digital (?)
http://bridget-torres.blogspot.com/  Personal! Do not edit!

=
Instru��es para entrar na lista, sair da lista e usar a lista em
http://www.mat.puc-rio.br/~obmlistas/obm-l.html
=


[obm-l] Resolução de Problemas [Problema 141, Eureka! 31]

2010-05-17 Por tôpico Johann Dirichlet
Dado a um dígito de 0 a 9, seja X um conjunto finito de naturais não
nulos tal que nenhum deles contenha o dígito a na sua representação
decimal. Demonstre que a soma dos inversos dos elementos de X é menor
que 80.

-- 
/**/
Quadrinista e Taverneiro!

http://tavernadofimdomundo.blogspot.com  Histórias, Poemas, Quadrinhos e Afins
http://baratoeletrico.blogspot.com / Ativismo Digital (?)
http://bridget-torres.blogspot.com/  Personal! Do not edit!

=
Instru��es para entrar na lista, sair da lista e usar a lista em
http://www.mat.puc-rio.br/~obmlistas/obm-l.html
=


[obm-l] Resolução de Problemas [Problema 139, Eureka! 31]

2010-05-17 Por tôpico Johann Dirichlet
Determine todos os inteiros positivos x,y,z tais que

y é primo
3 não é divisor de z
y não é divisor de z
x^3-y^3=z^2

-- 
/**/
Quadrinista e Taverneiro!

http://tavernadofimdomundo.blogspot.com  Histórias, Poemas, Quadrinhos e Afins
http://baratoeletrico.blogspot.com / Ativismo Digital (?)
http://bridget-torres.blogspot.com/  Personal! Do not edit!

=
Instru��es para entrar na lista, sair da lista e usar a lista em
http://www.mat.puc-rio.br/~obmlistas/obm-l.html
=


[obm-l] Re: Resolução de Problemas [Problema 141, Eureka! 31]

2010-05-17 Por tôpico Johann Dirichlet
Em 18 de maio de 2010 01:32, Johann Dirichlet
peterdirich...@gmail.com escreveu:
 Dado a um dígito de 0 a 9, seja X um conjunto finito de naturais não
 nulos tal que nenhum deles contenha o dígito a na sua representação
 decimal. Demonstre que a soma dos inversos dos elementos de X é menor
 que 80.

Bem, este problema é mais ou menos fácil, se você pegar a ideia certa.
O que se quer provar, no fim das contas, é que a série harmônica dos
inversos dos naturais sem um certo dígito converge. E convergência de
séries quase sempre é provada por comparação com uma PG.

O dígito em si pouco importa, o raciocínio não muda (eu tive alguns
problemas com o zero, mas nada de arrancar os cabelos...). Vou mostrar
com o dígito 9.

Bem, eu fiz mais ou menos assim (para facilitar a escrita, [a]=1/a:
[2]=1/2=0,5):

[1]+[2]+...+[8]=C (eu vou deixar assim, calculo depois)

[10]+...+[18] 9*[10]
[20]+...+[28] 9*[20]
[30]+...+[38] 9*[30]
[40]+...+[48] 9*[40]
.
.
.
[80]+...+[88] 8*[80]

Se eu somar estas desigualdades, obtenho um limitante de
9/10*([1]+[2]+...+[8])=9C/10.

Daqui, eu tive uma ideia: e se calcularmos a soma dos inversos dos
caras com exatamente K dígitos e sem o dígito 9? Depois é só somar
estas parciais e provar que elas convergem.

Veja com o 3 como fica:

[100]+...+[108] 9*[100]
[110]+...+[118] 9*[110]
[120]+...+[128] 9*[120]
.
.
.
[880]+...+[888] 9*[880]

Somando, obtemos um limitante de
9/10*(a soma dos inversos dos caras de 2 dígitos sem o 9), o que daria
81C/100=(9/10)^2C

Analogamente, obtemos um limitante da forma (9/10)^n*C para a soma dos
inversos dos caras com n dígitos.

Se somarmos, temos algo como
C*(1+9/10+(9/10)^2+...)=10*C

Basta provar que 11C80. Mas aí é só calcular e correr pro abraço!

Fazendo a conta, obtemos C3.


 --
 /**/
 Quadrinista e Taverneiro!

 http://tavernadofimdomundo.blogspot.com  Histórias, Poemas, Quadrinhos e 
 Afins
 http://baratoeletrico.blogspot.com / Ativismo Digital (?)
 http://bridget-torres.blogspot.com/  Personal! Do not edit!




-- 
/**/
Quadrinista e Taverneiro!

http://tavernadofimdomundo.blogspot.com  Histórias, Poemas, Quadrinhos e Afins
http://baratoeletrico.blogspot.com / Ativismo Digital (?)
http://bridget-torres.blogspot.com/  Personal! Do not edit!

=
Instru��es para entrar na lista, sair da lista e usar a lista em
http://www.mat.puc-rio.br/~obmlistas/obm-l.html
=


[obm-l] Resolução de Problemas [Problema 133, Eureka! 31]

2010-05-17 Por tôpico Johann Dirichlet
133) Considere um n–ágono regular inscrito em um círculo unitário, fixe um
vértice i e denote por d_j a distância entre este vértice i e o
vértice j. Prove que

(produtório de j=0 até j=n-1, j diferente de i) (5-d_j^2) = F_n^2

F_1 = 0, F_1 = 1 e F_n = F_(n−1)+F_(n−2) se n ≥ 2.


-- 
/**/
Quadrinista e Taverneiro!

http://tavernadofimdomundo.blogspot.com  Histórias, Poemas, Quadrinhos e Afins
http://baratoeletrico.blogspot.com / Ativismo Digital (?)
http://bridget-torres.blogspot.com/  Personal! Do not edit!

=
Instru��es para entrar na lista, sair da lista e usar a lista em
http://www.mat.puc-rio.br/~obmlistas/obm-l.html
=


[obm-l] Re: Resolução de Problemas [Problema 133, Eureka! 31] (esboço de tentativa)

2010-05-27 Por tôpico Johann Dirichlet
Em 17 de maio de 2010 21:00, Johann Dirichlet
peterdirich...@gmail.com escreveu:
 133) Considere um n–ágono regular inscrito em um círculo unitário, fixe um
 vértice i e denote por d_j a distância entre este vértice i e o
 vértice j. Prove que

 (produtório de j=0 até j=n-1, j diferente de i) (5-d_j^2) = F_n^2

 F_1 = 0, F_1 = 1 e F_n = F_(n−1)+F_(n−2) se n ≥ 2.

Bem, eu vou mostrar uma parte da minha ideia:

1- d_j^2=4sen^2(j*theta_n) em que theta_n=2pi/n é o ângulo central do poligono.
Assim, o que temos é (5-d_j^2)=(3+2cos(2j*theta_n)).

O problema se resume a acahar um polinomio cujas raizes sao os
cossenos acima, e depois calcular este polinomio no ponto (-3/2).

Eu tentei escrever o polinomio de maneira recursiva: ele é
cos(n*theta) escrito em função de cos \theta. Mas tô numa preguiça
insana de continuar



 --
 /**/
 Quadrinista e Taverneiro!

 http://tavernadofimdomundo.blogspot.com  Histórias, Poemas, Quadrinhos e 
 Afins
 http://baratoeletrico.blogspot.com / Ativismo Digital (?)
 http://bridget-torres.blogspot.com/  Personal! Do not edit!




-- 
/**/
Quadrinista e Taverneiro!

http://tavernadofimdomundo.blogspot.com  Histórias, Poemas, Quadrinhos e Afins
http://baratoeletrico.blogspot.com / Ativismo Digital (?)
http://bridget-torres.blogspot.com/  Personal! Do not edit!

=
Instru��es para entrar na lista, sair da lista e usar a lista em
http://www.mat.puc-rio.br/~obmlistas/obm-l.html
=


[obm-l] Resolucao de Problemas - Enviando...

2010-06-02 Por tôpico Johann Dirichlet
Olá people!
Bem, eu vou enviar esta primeira remessa de problemas por carta. Eu já
comecei a escrever e transcrever algumas delas, então não pretendo
fazer uma versão em LaTeX tão cedo... Até porque meu computador não tá
100%, mas logo eu resolvo isso.

Ao que parece este grupo fez um certo sucesso, mas ainda vou deixar em
fase de testes.


Qualquer coisa, eu aviso.

Johann
-- 
/**/
Quadrinista e Taverneiro!

http://tavernadofimdomundo.blogspot.com  Histórias, Poemas, Quadrinhos e Afins
http://baratoeletrico.blogspot.com / Ativismo Digital (?)
http://bridget-torres.blogspot.com/  Personal! Do not edit!

=
Instru��es para entrar na lista, sair da lista e usar a lista em
http://www.mat.puc-rio.br/~obmlistas/obm-l.html
=


Re: [obm-l] Divisores

2010-06-20 Por tôpico Johann Dirichlet
Bem, existe muita coisa sobre a soma dos divisores de um inteiro.
Primeiro, existe uma fórmula mais ou menos conhecida e fácil de
demonstrar.

Para comecar, que tal o MathWorld?
http://mathworld.wolfram.com/Divisor.html

Lá você pode procurar algumas curiosidades e mais referências.


Em 20 de junho de 2010 07:41, marcone augusto araújo borges
marconeborge...@hotmail.com escreveu:
 Se n é um inteiro positivo primo,a soma dos seus divisores é igual a
 n+1.Caso contrarioexiste algo sobre soma(e produto)dos divisores de um
 inteiro positivo?Onde eu poderia ler sobre isso?
 
 TRANSFORME-SE EM PERSONAGENS ENGRAÇADOS COM O SITE DE I LOVE MESSENGER. VEJA
 COMO.



-- 
/**/
Quadrinista e Taverneiro!

http://tavernadofimdomundo.blogspot.com  Histórias, Poemas, Quadrinhos e Afins
http://baratoeletrico.blogspot.com / Ativismo Digital (?)
http://bridget-torres.blogspot.com/  Personal! Do not edit!

=
Instru��es para entrar na lista, sair da lista e usar a lista em
http://www.mat.puc-rio.br/~obmlistas/obm-l.html
=


Re: [obm-l] IMO Polinomio irredutivel

2010-06-24 Por tôpico Johann Dirichlet
O site do Scholes morreu :(
Tente pelo Archive.org.

A solucao que eu conheco e mais ou menos essa:
Este polinomio nao tem raizes racionais (é só testar 1,3,-1 e -3 que
seriam as possibilidades).

Modulo 3, esse polinomio fatora como x^(n-1)(x+5).

Se pudermos escrever isto como P(x)Q(x), teremos
P(x)Q(x)=x^(n-1)(x+5). Logo P=x^k(x+5)+3X e Q=x^l+3Y com k+l=n-1 e X,Y
polinomios. Daí dá para obter uam contradicao simplesmente
multiplicando os polinomios.

Em 24 de junho de 2010 11:39, Luís Lopes qed_te...@hotmail.com escreveu:
 Sauda,c~oes,

 Na página 27 do livro 21 Aulas de Mat. Olímp. do C. Y. Shine
 encontro o seguinte problema:

 Prove que o polinômio x^n + 5x^{n-1} + 3 é irredutível em Q(Z).

 Gostaria de ver a solução baseada com o que foi mostrado no
 livro e as referências (fonte e solução) da página do kalva.

 Aliás o link para a homepage do kalva do site da Olimp. Paulista de Mat.
 estava quebrado quando tentei entrar.

 Obrigado.

 []'s
 Luís


 
 O SEU NAVEGADOR PODE TE PROTEGER DE FRAUDES NA WEB. VEJA DICAS DE INTERNET
 EXPLORER 8



-- 
/**/
Quadrinista e Taverneiro!

http://tavernadofimdomundo.blogspot.com  Histórias, Poemas, Quadrinhos e Afins
http://baratoeletrico.blogspot.com / Ativismo Digital (?)
http://bridget-torres.blogspot.com/  Personal! Do not edit!

=
Instru��es para entrar na lista, sair da lista e usar a lista em
http://www.mat.puc-rio.br/~obmlistas/obm-l.html
=


Re: [obm-l] IMO Polinomio irredutivel

2010-06-24 Por tôpico Johann Dirichlet
Ah, o site:
http://www.cs.cornell.edu/~asdas/IMO/imo.html

Uma versao antiga.

Em 24 de junho de 2010 12:24, Johann Dirichlet
peterdirich...@gmail.com escreveu:
 O site do Scholes morreu :(
 Tente pelo Archive.org.

 A solucao que eu conheco e mais ou menos essa:
 Este polinomio nao tem raizes racionais (é só testar 1,3,-1 e -3 que
 seriam as possibilidades).

 Modulo 3, esse polinomio fatora como x^(n-1)(x+5).

 Se pudermos escrever isto como P(x)Q(x), teremos
 P(x)Q(x)=x^(n-1)(x+5). Logo P=x^k(x+5)+3X e Q=x^l+3Y com k+l=n-1 e X,Y
 polinomios. Daí dá para obter uam contradicao simplesmente
 multiplicando os polinomios.

 Em 24 de junho de 2010 11:39, Luís Lopes qed_te...@hotmail.com escreveu:
 Sauda,c~oes,

 Na página 27 do livro 21 Aulas de Mat. Olímp. do C. Y. Shine
 encontro o seguinte problema:

 Prove que o polinômio x^n + 5x^{n-1} + 3 é irredutível em Q(Z).

 Gostaria de ver a solução baseada com o que foi mostrado no
 livro e as referências (fonte e solução) da página do kalva.

 Aliás o link para a homepage do kalva do site da Olimp. Paulista de Mat.
 estava quebrado quando tentei entrar.

 Obrigado.

 []'s
 Luís


 
 O SEU NAVEGADOR PODE TE PROTEGER DE FRAUDES NA WEB. VEJA DICAS DE INTERNET
 EXPLORER 8



 --
 /**/
 Quadrinista e Taverneiro!

 http://tavernadofimdomundo.blogspot.com  Histórias, Poemas, Quadrinhos e 
 Afins
 http://baratoeletrico.blogspot.com / Ativismo Digital (?)
 http://bridget-torres.blogspot.com/  Personal! Do not edit!




-- 
/**/
Quadrinista e Taverneiro!

http://tavernadofimdomundo.blogspot.com  Histórias, Poemas, Quadrinhos e Afins
http://baratoeletrico.blogspot.com / Ativismo Digital (?)
http://bridget-torres.blogspot.com/  Personal! Do not edit!

=
Instru��es para entrar na lista, sair da lista e usar a lista em
http://www.mat.puc-rio.br/~obmlistas/obm-l.html
=


[obm-l] Olimpíada Matemática do Cone Sul - 2010 (enunciado s)

2010-06-24 Por tôpico Johann Dirichlet
Como já foi tradição nesta lista, vou colocar os enunciados da
Olimpíada do Cone Sul deste ano.

***

Problema 1
Pedro tem que escolher duas frações irredutíveis, cada uma com
numerador e denominador
positivos, tais que:
• A soma das duas frações seja igual a 2.
• A soma dos numeradores das duas frações seja igual a 1000.

De quantas maneiras Pedro pode fazer isso?

Problema 2
Marcam-se em uma reta 44 pontos, numerados 1, 2, 3, . . . , 44 da
esquerda para a
direita. Vários grilos saltam na reta. Cada grilo parte do ponto 1,
salta por pontos marcados e
termina no ponto 44. Além disso, cada grilo sempre salta de um ponto
marcado a outro marcado
com um número maior.
Quando todos os grilos terminaram de saltar, notou-se que para cada
par i, j, com 1 = i  j = 44,
há um grilo que saltou diretamente do ponto i para o ponto j, sem
pousar em nenhum dos pontos
entre eles.
Determine a menor quantidade de grilos para que isso seja possível.

Problema 3
Recortar um polígono convexo de n lados significa escolher um par de
lados consecutivos
AB,BC do polígono e substitui-los por três segmentos AM, MN e NC,
sendo M o ponto
médio de AB e N o ponto médio de BC. Em outras palavras, corta-se o
triângulo MBN e obtém-se
um polígono convexo de n + 1 lados.
Seja P6 um hexágono regular de área 1. Recorta-se P6 e obt´em-se o
polígono P7. Então recorta-se
P7, de uma das sete maneiras possíveis, e obtém-se o polígono P8, e
assim sucessivamente. Prove
que, independentemente de como sejam feitos os recortes, a área de Pn
é sempre maior do que 2/3.

Problema 4
Pablo e Sílvia jogam em um tabuleiro 2010×2010. Primeiro Pablo escreve um número
inteiro em cada casa. Feito isso, Sílvia repete tantas vezes quanto
quiser a seguinte operação: escolher
três casas que formem um L (um triminó em formato de L) e somar 1 a
cada número dessas três casas.
Sílvia ganha se fizer com que todos os números do tabuleiro sejam
múltiplos de 10.
Demonstre que Sílvia sempre pode escolher uma sequência de operações
com as quais ela ganha
o jogo.

Problema 5
O incírculo do triângulo ABC toca os lados BC, CA e AB em D, E e F,
respectivamente.
Sejam Wa, Wb e Wc os circuncírculos dos triângulos EAF, DBF e DCE,
respectivamente.
As retas DE e DF cortam Wa em Ea != E e Fa != F, respectivamente. Seja
rA a reta EaFa. Defina
rB e rC de modo analogo. Prove que as retas rA, rB e rC determinam um
triângulo cujos vértices
pertencem aos lados do triângulo ABC.

Problema 6
Determine se existe uma sequência infinita a0, a1, a2, a3, . . . de
inteiros nao negativos
que satisfaz as seguintes condições:
(i) Todos os numeros inteiros nao negativos aparecem na sequência uma única vez;
(ii) A sequˆencia

bn = an + n, n ≥ 0,
é formada por todos os numeros primos, cada um aparecendo uma única vez.

***


-- 
/**/
Quadrinista e Taverneiro!

http://tavernadofimdomundo.blogspot.com  Histórias, Poemas, Quadrinhos e Afins
http://baratoeletrico.blogspot.com / Ativismo Digital (?)
http://bridget-torres.blogspot.com/  Personal! Do not edit!

=
Instru��es para entrar na lista, sair da lista e usar a lista em
http://www.mat.puc-rio.br/~obmlistas/obm-l.html
=


[obm-l] Re: [obm-l] Re: [obm-l] Re: [obm-l] Re: [obm-l] Re: [obm-l] Hierarquia das Operações

2010-07-29 Por tôpico Johann Dirichlet
Bem, posso dizer que eu nunca pensei nesta convenção. No fim das
contas, quando entrei no Ensino Médio, eu usava só parênteses por ser
nais prático, e pelo fato de colchetes nunca aparecerem dentro de
parênteses (só fui entender o porquê quando passei a programar).

Logo que passei pela Computação (aliás tô meio enferrujado) eu aprendi
algo sobre notação infixa e posfixa. Apesar de serem boas para contas
em computador, são horríveis para o raciocínio. Por exemplo, como
expressar uma equação de segundo grau, ou um sistema de equações, em
notação não-convencional? Mas na hora de programar a notação prefixa
é mais útil (quem já viu código LISP sabe do que estou falando, hehe!)
e a posfixa tem seu lugar cativo em Compiladores (quase toda linguagem
de programação pode ser evaluada em uma pilha modificada).

Hoje em dia, eu me confundo completamente quando não leio a primeira
página das convenções adotadas neste livro: em alguns livros, {x} é
o inteiro mais próximo de x, em outros [x] é o conjunto dos naturais
abaixo de x...

Uma curiosidade meio que no off-topic: tem um programa muito comum em
máquinas Unix, o dc, que roda um RPN Calculator. Ele tem mais
operações além das 4 básicas. Dá até pra calcular raízes quadradas e
programar funções.

Em 29/07/10, Luiz Rodriguesrodrigue...@gmail.com escreveu:
 Olá pessoal!!!
 Tudo bem???
 Agradeço a todos vocês pelas respostas.
 Tenho bastante material para poder pensar...
 Um grande abraço para todos!!!
 Luiz

 2010/7/28 Adalberto Dornelles aadornell...@gmail.com
  Olá turma,
 
 
  Falando em notação RPN, lembrei do termo quando comprei uma
  calculadora HP 15 C (há muito tempo). Lembro que custei a pegar o
  jeito, mas depois  gostei muito. A resolução de expressões usando a
  pilha operacional e pensando de dentro pada fora me facilitou a
  vida. Até hoje, penso ser a melhor maneira de operar. Pena que meus
  alunos da engenharia tenham muita dificuldade com a calculadora...
 
  Abraço,
 
  Adalberto
 
 
  Em 28 de julho de 2010 15:05, Bernardo Freitas Paulo da Costa
  bernardo...@gmail.com escreveu:
 
 
 
   2010/7/28 Alessandro Madruga Correia amcorr...@viaconnect.com.br:
   Em 28-07-2010 09:32, Bernardo Freitas Paulo da Costa escreveu:
  
   E agora, uma charada:
  
   +*+*123--45--678 = ?
  
  
   +  ((1*2)+3) *   [(4-5) - ((6-7) - 8)] = 40
  
   É isso?
   Essa é uma solução que sai do problema :) Note que você interpretou
   o primeiro + sem somar ninguém, o que é um problema !!! Mas é mais
   ou menos por aí...
  
   Enfim, pra dar uma dica: como há 8 operandos e 8 operações, não é
   possível que elas sejam todas binárias... senão, fica faltando um
   operando.
  
  
   Abraço.
  
   abraços,
   --
   Bernardo Freitas Paulo da Costa
  
  
 =
   Instruções para entrar na lista, sair da lista e usar a lista em
   http://www.mat.puc-rio.br/~obmlistas/obm-l.html
  
 =
  
 
 
 =
  Instruções para entrar na lista, sair da lista e usar a lista em
  http://www.mat.puc-rio.br/~obmlistas/obm-l.html
 
 =
 




-- 
/**/
Quadrinista e Taverneiro!

http://tavernadofimdomundo.blogspot.com  Histórias, Poemas, Quadrinhos e Afins
http://baratoeletrico.blogspot.com / Ativismo Digital (?)
http://bridget-torres.blogspot.com/  Personal! Do not edit!

=
Instru��es para entrar na lista, sair da lista e usar a lista em
http://www.mat.puc-rio.br/~obmlistas/obm-l.html
=


Re: [obm-l] Ajuda Urgente!!!

2010-08-12 Por tôpico Johann Dirichlet
Outra ideia e tentar uma induçao. Se voce faz casinhos pequenos, o
padrao surge rapidamente. Bem, como e de praxe, nao vou exibir nada
escrito.

Em 10/08/10, Fabio Bernardoprof_fabioberna...@yahoo.com.br escreveu:
 1/1.2 + 1/2.3 + ... + 1/(n-1).n



 = 1 – 1/2 + 1/2 – 1/3 + 1/3 – 1/4 + . . . + 1/(n-1) – 1/n = 1 – 1/n = (n –
 1)/n







 De: owner-ob...@mat.puc-rio.br [mailto:owner-ob...@mat.puc-rio.br] Em nome
 de warley ferreira
 Enviada em: terça-feira, 10 de agosto de 2010 00:04
 Para: Lista de Discussão
 Assunto: [obm-l] Ajuda Urgente!!!



 Oá Pessoal, td bom?

 Como calcular a soma abaixo?

 1/1.2 + 1/2.3 + ... + 1/(n-1).n

 Desde já agradeço,

 Abraços

 Warley F Souza




 No virus found in this incoming message.
 Checked by AVG - www.avg.com
 Version: 9.0.851 / Virus Database: 271.1.1/3060 - Release Date: 08/09/10
 03:35:00




-- 
/**/
Quadrinista e Taverneiro!

http://tavernadofimdomundo.blogspot.com  Histórias, Poemas, Quadrinhos e Afins
http://baratoeletrico.blogspot.com / Ativismo Digital (?)
http://bridget-torres.blogspot.com/  Personal! Do not edit!

=
Instru��es para entrar na lista, sair da lista e usar a lista em
http://www.mat.puc-rio.br/~obmlistas/obm-l.html
=


[obm-l] Re: [obm-l] Números

2010-08-18 Por tôpico Johann Dirichlet
O que voce quer dizer com fatores?
Se for fatores em geral, isso so acontece se ambos forem iguais.
Por exemplo, 18 e 12 tem o fator 3 em comum mas nao o fator 4.

Se forem fatores primos, fica mais interessante Por exemplo, ambos os
caras acima tem os fatores 2 e 3 em comum.


Em 18/08/10, luiz silvaluizfelipec...@yahoo.com.br escreveu:
 Pessoal,

 Dados a,b,u e v com mdc(a,b)=1 e mdc(u,v) =1,   ba e vu, quais as
 condições para que

 todos os fatores de bu-av , sejam fatores de bv-au ?

 Abs
 Felipe





-- 
/**/
Quadrinista e Taverneiro!

http://tavernadofimdomundo.blogspot.com  Histórias, Poemas, Quadrinhos e Afins
http://baratoeletrico.blogspot.com / Ativismo Digital (?)
http://bridget-torres.blogspot.com/  Personal! Do not edit!

=
Instru��es para entrar na lista, sair da lista e usar a lista em
http://www.mat.puc-rio.br/~obmlistas/obm-l.html
=


[obm-l] Re: [obm-l] Re: [obm-l] combinatória

2010-08-20 Por tôpico Johann Dirichlet
Outra solucao, mais direta IMHO, usa o teorema de Turan:

Dado um grafo (G,V), se não existem subgrafos k-completos nele, entao
o numero de arestas maximo e obtido em uma configuracao desta forma:
k-1 grupos de vertices, cada um contendo o mesmo numero de vertices
(ou o mais proximo disso, usando uma divisao euclidiana). Cada vertice
conecta-se a todos os vertices que nao sejam de seu grupo.

De certo modo, isto demonstra que a solucao que o WGAP mostrou para o
maximo e unica.

Bem, depois eu envio uma solucao do teorema de turan. Nao e dificil
demonstra-lo para o caso acima (o grafo nao contem triangulos). Para
quem for curioso, o famoso livro Proofs from THE BOOK contem tres ou
quatro provas.

Em 18/08/10, Willy George do Amaral Petrenkowgapetre...@gmail.com escreveu:
 *O planeta Walrus possui 20 países. Sabe-se que, dentre quaisquer três
 desses países, existem dois sem relações diplomáticas. Prove que Walrus
 possui no Maximo 200 embaixadas*.

 O número mínimo de embaixadas é zero. O enunciado diz existem 2 sem
 relações, e não existem EXATAMENTE 2 sem relações. De fato, se nenhum
 país faz diplomacia com ninguém, o enunciado é satisfeito.

 Mas vamos à idéia da demonstração:

 Notação: Chamarei de amigos os países que tiverem relações diplomáticas e de
 inimigos os que não tiverem.

 Repare que se um país A é amigo de B e C, então B é inimigo de C.
 Repare também que se um país é amigo de todos os outros, então todos os
 outros são inimigos entre si, fazendo um total de 38 embaixadas.
 Agora, veja que se nenhum país tem mais de 10 amigos então o enunciado é
 satisfeito.

 Suponha agora por absurdo que existam + de 200 embaixadas e que algum país A
 tenha 11 amigos.
 Então esses 11 amigos de A serão inimigos mútuos, fazendo com que cada um
 deles tenha um máximo de 9 amigos.
 Esses 12 países (A + os 11) terão um máximo de 11 + 9*11 = 110 embaixadas.
 Os outros 8 países então devem ter mais de 90 embaixadas (para satisfazer o
 mais de 200). Pelo princípio da casa dos pombos algum terá 12 amigos.

 Provamos então que se o total de embaixadas é maior que 200 e alguém tem 11
 amigos então alguém tem 12 amigos.
 *A idéia agora é provar que se o total de embaixadas é maior que 200 e
 alguém tem 10+n amigos, onde n natural pertencente a [1,8], então alguém tem
 10+n+1 amigos. [é fácil, a mesma idéia, só trabalhar com letrinhas].*
 Com isso completa-se a indução.

 A indução mostra que se existem +de 200 embaixadas e alguém tem 11 amigos
 então alguém tem 19 amigos e logo o número de embaixadas é 38, absurdo.
 E se ninguém tem +de 10 amigos então o número de embaixadas é = 200.

 Isso completa a prova.

 Dá para dar um exemplo onde existem 200 embaixadas (embora o enunciado não
 peça):
 Países de 1 a 10 inimigos entre si e amigos de todos os países de 11 a 20.
 Países de 11 a 20 inimigos entre si e amigos de todos os países de 1 a 10.

 Espero que tenha entendido a idéia. Se vc não conseguir completar a prova
 avisa que eu escrevo.



-- 
/**/
Quadrinista e Taverneiro!

http://tavernadofimdomundo.blogspot.com  Quadrinhos, histórioas e afins
http://baratoeletrico.blogspot.com / Um pouco sobre elétrons em movimento
http://bridget-torres.blogspot.com/  Personal! Do not edit!

=
Instru��es para entrar na lista, sair da lista e usar a lista em
http://www.mat.puc-rio.br/~obmlistas/obm-l.html
=


[obm-l] Re: [obm-l] combinatória

2010-08-25 Por tôpico Johann Dirichlet
Este e patrecido com um problema da primeira fase da OBM de uns 2 ou 3
anos atras.
Como tem tres As repetidos, chame eles de A1, A2, A3 (A1 e uma coisa
so, nao duas. Pense como se fossem indices numericos)

Primeiro, os caras BTHL ficam nesta ordem. Veja o esquema
_B_T_L_H_

Escolha aonde o O vai ficar nestes espacos. Sao 5 possibilidades. Veja
um exemplo:
_B_T_L_H_O_

Sempre que um cara entra ele gera um espaco. Continuando, onde o A1
quer ir? Sao 6 possibilidades:
_B_A1_T_L_H_O_

E assim por diante.

Agora, lembre-se que nos diferenciamos os As. Como eles sao 3, eles
vem nestas ordens: 123,132,213,231,312,321 (ou o famoso 3!). Basta
dividir o numero obtido antes por este fator.



Em 25/08/10, Fabio Silvacacar...@yahoo.com escreveu:
 Quantos anagramas da palavra BATALHÃO (desconsidere o til como diferença)
 tem as consoantes em ordem alfabética?

 Como pensar?






-- 
/**/
Quadrinista e Taverneiro!

http://tavernadofimdomundo.blogspot.com  Quadrinhos, histórioas e afins
http://baratoeletrico.blogspot.com / Um pouco sobre elétrons em movimento
http://bridget-torres.blogspot.com/  Personal! Do not edit!

=
Instru��es para entrar na lista, sair da lista e usar a lista em
http://www.mat.puc-rio.br/~obmlistas/obm-l.html
=


[obm-l] Re: Propriedades dos divisores

2010-09-03 Por tôpico Johann Dirichlet
Ué, mas ela já está aí!

Se você quer algo com linguagem chata, tá bom:

Seja D(N) o conjunto dos divisores de N

1) d pertence a D(N)
acarreta
N/d pertence a D(N)

2) d1d2 acarreta N/d1  N/d2

3) Sejam os conjuntos D(N,d) = {d,N/d}.
Para cada d pertencente a D(N), o conjunto D(N,d) está contido em D(N).
Ademais, D(N,d1)=D(N,d2) se e so se d1=d2 ou d1=N/d2.
Além disso, se d=N/d (ou d^2=N) o conjunto D(N,d) tem somente um elemento.
De quebra, o produto dos elementos de cada D(N,d) é N (excetuando o caso d^2=N).

Portanto, os conjuntos D(N,d), com d variando entre todos os divisores
de N menores que a raiz quadrada de N, particionam o conjunto D(N) (em
outras palavras, são dois a dois disjuntos e a uniao deles é D(n)).

Agora a parte lúdica:
Se colocarmos os divisores de N em uma fila indiana, e a cada um
colocarmos uma etiqueta eu sou elemento do conjunto D(N,d) (em que
escolhemos o d tal que d^2 = N), veremos que os números das pontas
terao etiquetas iguais, e o produto de ambos será N. É justamente isto
o que eu demonstrei acima, ó pá!
Se o numero de divisores for impar, tera um cara que nao tem etiqueta
repetida, justamente a raiz quadrada exata de N.É justamente isto o
que eu demonstrei acima, ó pá!

Esta é a demonstração mais formal que eu consigo escrever, sem ficar enjoado...


Em 02/09/10, enniusenn...@bol.com.br escreveu:

 Caro Torres,

 É exatamente a formalização que desejo obter.

 Um abraço!
 Ennius



-- 
/**/
Quadrinista e Taverneiro!

http://tavernadofimdomundo.blogspot.com  Quadrinhos, histórioas e afins
http://baratoeletrico.blogspot.com / Um pouco sobre elétrons em movimento
http://bridget-torres.blogspot.com/  Personal! Do not edit!

=
Instru��es para entrar na lista, sair da lista e usar a lista em
http://www.mat.puc-rio.br/~obmlistas/obm-l.html
=


[obm-l] Re: [obm-l] Equação algébrica

2010-09-13 Por tôpico Johann Dirichlet
A maneira que me vem à cabeça é usar o teorema do valor intermediario.

Podemos fazer algumas suposições:
|r|  1. De fato, se |r|1, troque r por R=1/r e x por X=1/x. Assim,
teremos X^n=R, com |R|1, e resolver essa equacao é equivalente
resolver a original.

Caso n ímpar:
Se r  0, podemos trocar x por -x e r por -r. Vamops entao supor r1.

Enfim, existe um valor de x tal que x^n-r0. Isso e relativamente
facil de demonstrar usando limites ou algo que valha.
Igualmente, existe outro valor de x tal que x^n-r0.

Pelo teorema do valor intermediario, existe um cara entre estes dois
extremos tal que x^n=r=0.

O caso par fica por sua conta :)


Em 11/09/10, Guilherme Vieirarjguilhermevie...@hotmail.com escreveu:

 Solicito aos amigos uma demonstração do teorema enunciado a seguir.
 Obviamente, a propriedade é muito conhecida. A demonstração, entretanto,
 parece-me muito difícil.

 Teorema: Se x é uma variável real, n é um número natural (não nulo) e r é
 uma constante real, a equação algébrica x^n = r admite uma única solução
 real quando n é ímpar e admite duas soluções reais quando n é par e r0.


 Obrigado!!!
 Guilherme 


-- 
/**/
Quadrinista e Taverneiro!

http://tavernadofimdomundo.blogspot.com  Quadrinhos, histórioas e afins
http://baratoeletrico.blogspot.com / Um pouco sobre elétrons em movimento
http://bridget-torres.blogspot.com/  Personal! Do not edit!

=
Instru��es para entrar na lista, sair da lista e usar a lista em
http://www.mat.puc-rio.br/~obmlistas/obm-l.html
=


Re: [obm-l] ajuda

2010-09-14 Por tôpico Johann Dirichlet
Não é nenhuma das coisas.O zero é uma espécie de múltiplo universal:
todo número é múlrtiplo de zero.

Um numero, para ser primo, não pode ser escrito como o produto de dois
fatores maiores que 1.
Já um composto é, necessariamente, um produto de dois ou mais naturais
menores que ele (e maiores que 1).

O zero e o um caem fora destes dois casos.

P.S.: estamos falando dos naturais. Existem estruturas matemáticas nas
quais o produto de dois não-nulos pode ser nulo.


Em 14/09/10, Adalberto Dornellesaadornell...@gmail.com escreveu:
 Olá turma,

 Perguntinha rápida:

 O zero é primo? é composto? ou nem uma coisa nem outra?

 Tenho 98,6544% de certeza que a resposta é nem uma coisa nem outra, mas
 ...

 Abraço,
 Adalberto

 =
 Instruções para entrar na lista, sair da lista e usar a lista em
 http://www.mat.puc-rio.br/~obmlistas/obm-l.html
 =



-- 
/**/
Quadrinista e Taverneiro!

http://tavernadofimdomundo.blogspot.com  Quadrinhos, histórioas e afins
http://baratoeletrico.blogspot.com / Um pouco sobre elétrons em movimento
http://bridget-torres.blogspot.com/  Personal! Do not edit!

=
Instru��es para entrar na lista, sair da lista e usar a lista em
http://www.mat.puc-rio.br/~obmlistas/obm-l.html
=


Re: [obm-l] ajuda

2010-09-15 Por tôpico Johann Dirichlet
Em 14/09/10, Bernardo Freitas Paulo da Costabernardo...@gmail.com escreveu:
 2010/9/14 Johann Dirichlet peterdirich...@gmail.com:
 Não é nenhuma das coisas.O zero é uma espécie de múltiplo universal:
 todo número é múlrtiplo de zero.
 Cuidado, Johann! Além de escrever quase escrever múrtiplo, nenhum
 número, exceto zero, é múltiplo de zero. Ele é realmente um múltiplo
 universal, logo é também divisível por todo número inteiro.


Bem lembrado! Eu sempre esqueço o que é o que: se a | b então a é
divisor de b e b é múltiplo de a.

 Um numero, para ser primo, não pode ser escrito como o produto de dois
 fatores maiores que 1.
 Já um composto é, necessariamente, um produto de dois ou mais naturais
 menores que ele (e maiores que 1).

 O zero e o um caem fora destes dois casos.
 Em geral, você exclui o zero da decomposição porque ele não faz parte
 do grupo multiplicativo (ele não tem inverso). Só tem sentido falar de
 composto e indecomponível (a primeira noção de primo, lá dos
 gregos) dentro de um grupo. Já o um não é primo porque é inversível, e
 a gente também exclui os inversíveis das definições porque isso
 complicaria muito a decomposição (fatoração) única.

 Enfim, dica pro Alberto e todos mais: o importante de entender as
 definições é ver porque elas foram escolhidas assim. Uma resposta boa
 (para mim) é que neste caso a gente tem um enunciado bem simples do
 teorema de decomposição dos números inteiros:
  Todo número inteiro composto n possui uma única decomposição em
 fatores primos, a menos de
 - mudança na ordem dos fatores
 - multiplicações por inversíveis

Esse é semelhante a uma definição do artigo do Guilherme Issao sobre
inteiros de Eisenstein: fatoração única a menos da ordem e a menos de
multiplicação por unidades.


 Note que eu excluí os primos, os irredutíveis (e o zero) do teorema
 porque para eles o resultado é imediato das definições, portanto não é
 tão interessante como o que vale para os compostos.

 Note que esse teorema fica mais simples ainda (mas perdemos um pouco a
 abstração que vale em vários casos) se falarmos de naturais: Todo
 inteiro estritamente positivo pode ser escrito de forma única como
 produto de primos positivos em ordem crescente

Melhor escrever não-decrescente (para deixar mais claro que pode
haver repetições).


 Abraços
 --
 Bernardo Freitas Paulo da Costa

 P.S.: estamos falando dos naturais. Existem estruturas matemáticas nas
 quais o produto de dois não-nulos pode ser nulo.


 Em 14/09/10, Adalberto Dornellesaadornell...@gmail.com escreveu:
 Olá turma,

 Perguntinha rápida:

 O zero é primo? é composto? ou nem uma coisa nem outra?

 Tenho 98,6544% de certeza que a resposta é nem uma coisa nem outra, mas
 ...

 Abraço,
 Adalberto

 =
 Instruções para entrar na lista, sair da lista e usar a lista em
 http://www.mat.puc-rio.br/~obmlistas/obm-l.html
 =



-- 
/**/
Quadrinista e Taverneiro!

http://tavernadofimdomundo.blogspot.com  Quadrinhos, histórioas e afins
http://baratoeletrico.blogspot.com / Um pouco sobre elétrons em movimento
http://bridget-torres.blogspot.com/  Personal! Do not edit!

=
Instru��es para entrar na lista, sair da lista e usar a lista em
http://www.mat.puc-rio.br/~obmlistas/obm-l.html
=


[obm-l] Re: [obm-l] RE: [obm-l] Fatorial via Stirling (confi rmação)

2010-09-17 Por tôpico Johann Dirichlet
Bem, vou azedar um pouco a coisa: que tal se pudéssemos isolar o r?
n! = [(2.n.pi)^(1/2)].[(n/e)^n].(e^r) se e somente se
n!/((2.n.pi)^(1/2).(n/e)^n)=(e^r)
Passa o log, temos uma expressão em r.
Se pudermos provar a existência deste monstrinho, fechou

Em 17/09/10, Guilherme Vieirarjguilhermevie...@hotmail.com escreveu:

 Caro Paulo,
 Continuo pensando que não há possibilidade de se obter demonstração por
 indução finita, pois r depende de n.
 Não sei se há outro modo de confirmar a validade da fórmula.
 Continuemos tentando!
 Um abraço do Guilherme!



 From: argolopa...@hotmail.com
 To: obm-l@mat.puc-rio.br
 Subject: [obm-l] Fatorial via Stirling (confirmação)
 Date: Thu, 16 Sep 2010 20:55:27 +





 Caros amigos,
 Repito a questão a que propus.
 Não sei se as respostas já dadas tratam efetivamente da mesma questão.
 Fiquei em dúvida.

 Gostaria de obter uma demonstração (pode ser por indução finita) do fato
 abaixo, proveniente da fórmula de Stirling.

 Fato:
 Para todo número inteiro positivo n, existe um número real r, com 1/(12n+1)
  r
  1/(12n), de modo que seja válida a igualdade:
 n! = [(2.n.pi)^(1/2)].[(n/e)^n].(e^r)

 Muito obrigado!
 Paulo Argolo







   


-- 
/**/
Quadrinista e Taverneiro!

http://tavernadofimdomundo.blogspot.com  Quadrinhos, histórioas e afins
http://baratoeletrico.blogspot.com / Um pouco sobre elétrons em movimento
http://bridget-torres.blogspot.com/  Personal! Do not edit!

=
Instru��es para entrar na lista, sair da lista e usar a lista em
http://www.mat.puc-rio.br/~obmlistas/obm-l.html
=


Re: [obm-l] Qual a melhor mailing list internacional de Mathematics

2010-09-20 Por tôpico Johann Dirichlet
Mailing list eu não sei, mas se você aceita um fórum, tem o www.mathlinks.ro.

Em 19/09/10, Rafaelapolo_hiperbo...@terra.com.br escreveu:
 Olá, pessoal.


 Qual a melhor mailing list internacional de Matemática ?



 Regards,
 Rafael


-- 
/**/
Quadrinista e Taverneiro!

http://tavernadofimdomundo.blogspot.com  Quadrinhos, histórioas e afins
http://baratoeletrico.blogspot.com / Um pouco sobre elétrons em movimento
http://bridget-torres.blogspot.com/  Personal! Do not edit!

=
Instru��es para entrar na lista, sair da lista e usar a lista em
http://www.mat.puc-rio.br/~obmlistas/obm-l.html
=


[obm-l] Re: [obm-l] Re: [obm-l] Re: [obm-l] RE: [obm-l] Fato rial via Stirling (confirmação)

2010-09-20 Por tôpico Johann Dirichlet
Em 18/09/10, Bernardo Freitas Paulo da Costabernardo...@gmail.com escreveu:
 2010/9/17 Johann Dirichlet peterdirich...@gmail.com:
 Bem, vou azedar um pouco a coisa: que tal se pudéssemos isolar o r?
 n! = [(2.n.pi)^(1/2)].[(n/e)^n].(e^r) se e somente se
 n!/((2.n.pi)^(1/2).(n/e)^n)=(e^r)
 Passa o log, temos uma expressão em r.
 Se pudermos provar a existência deste monstrinho, fechou

 Eu acho que a fórmula de Euler-MacLaurin é realmente o que é mais
 adaptado para provar esse tipo de horror (expansão assintótica de
 somas finitas, quando a gente passa aos logs). Tem que estudar, mas
 enfim, você não pode querer demonstrar tudo a partir de nada: a
 matemática se constrói passo a passo...

 Enfim, esta observação chata é mais porque, de memória, obter o raiz
 de 2*pi na fórmula do fatorial é bem difícil. Se você dispensar
 essa exatidão toda, acho que até dá, inclusive por indução (Johann: já
 achou como corrigir a tua?). Daí, a fórmula fica
 n! = (n/e)^n*raiz(n) * erro(n)

Na verdade eu nem tentei :)
Creio que você esteja certo no erro da fórmula. No fim das contas
essa constante é difícil de se obter por indução. A bem da verdade não
conheço nenhum problema de limites que use indução.


 onde 0  min  erro(n)  MAX para duas constantes min e MAX (que a
 gente não calculou)

 Em 17/09/10, Guilherme Vieirarjguilhermevie...@hotmail.com escreveu:

 Caro Paulo,
 Continuo pensando que não há possibilidade de se obter demonstração por
 indução finita, pois r depende de n.
 Não sei se há outro modo de confirmar a validade da fórmula.
 Continuemos tentando!
 Um abraço do Guilherme!



 From: argolopa...@hotmail.com
 To: obm-l@mat.puc-rio.br
 Subject: [obm-l] Fatorial via Stirling (confirmação)
 Date: Thu, 16 Sep 2010 20:55:27 +





 Caros amigos,
 Repito a questão a que propus.
 Não sei se as respostas já dadas tratam efetivamente da mesma questão.
 Fiquei em dúvida.

 Gostaria de obter uma demonstração (pode ser por indução finita) do fato
 abaixo, proveniente da fórmula de Stirling.

 Fato:
 Para todo número inteiro positivo n, existe um número real r, com
 1/(12n+1)
  r
  1/(12n), de modo que seja válida a igualdade:
 n! = [(2.n.pi)^(1/2)].[(n/e)^n].(e^r)

 Muito obrigado!
 Paulo Argolo










 --
 /**/
 Quadrinista e Taverneiro!

 http://tavernadofimdomundo.blogspot.com  Quadrinhos, histórioas e afins
 http://baratoeletrico.blogspot.com / Um pouco sobre elétrons em
 movimento
 http://bridget-torres.blogspot.com/  Personal! Do not edit!

 =
 Instru�ões para entrar na lista, sair da lista e usar a lista em
 http://www.mat.puc-rio.br/~obmlistas/obm-l.html
 =




 --
 Bernardo Freitas Paulo da Costa

 =
 Instru�ões para entrar na lista, sair da lista e usar a lista em
 http://www.mat.puc-rio.br/~obmlistas/obm-l.html
 =



-- 
/**/
Quadrinista e Taverneiro!

http://tavernadofimdomundo.blogspot.com  Quadrinhos, histórioas e afins
http://baratoeletrico.blogspot.com / Um pouco sobre elétrons em movimento
http://bridget-torres.blogspot.com/  Personal! Do not edit!

=
Instru��es para entrar na lista, sair da lista e usar a lista em
http://www.mat.puc-rio.br/~obmlistas/obm-l.html
=


Re: [obm-l] Para mim, 0^0=1

2010-09-22 Por tôpico Johann Dirichlet
Nessas horas eu me pergunto: por que existem tantas arestas
não-aparadas na matemática?

A aresta mais pontuda, na minha opinião, é o paradoxo de
Banach-Tarski: é possível desmontar uma bolinha de gude e juntar os
pedaços de modo a se obter uma bola do tamanho do sol.

Em 16/09/10, Ralph Teixeiraralp...@gmail.com escreveu:
 Eu sou um dos defensores de 0^0=1. Apresento dois motivos:

 i) Se f(x) e g(x) sao analiticas em 0 com f(x),g(x)-0 quando x-a, entao
 f^g - 1 quando x- a (bom, desde que f^g faca sentido em volta de x=a). A
 *unica* excecao a esta regra eh o caso em que f eh identicamente nula,
 quando o limite dah 0 (se f^g faz sentido) ou nao existe (se g0 ali por
 perto de x=a).
 Isto explica porque 99.9% dos exercicios de limite que ficam da forma 0^0
 acabam dando 1 como resposta!

 Acho que isto tambem explica porque eu nao faria 0/0=1 ou algo assim -- nao
 ha teorema semelhante para 0/0.

 ii) Como escrever um polinomio generico de grau 17 usando somatorios? Acho
 que muita gente concorda que uma boa representacao eh:
 p(x) = SUM (n=0 a 17) a_n x^n
 onde os a_n sao coeficientes arbitrarios. Agora eu pergunto -- quanto vale
 p(0)?

 Com a convencao 0^0=1, nada especial precisa ser feito, eh soh substituir
 x=0 no somatorio.

 Com a convencao 0^0 nao existe bom, ai a nossa representacao por
 somatorio ficaria tecnicamente errada. Teriamos que escrever:

 p(x) = SUM (n=0 a 17) a_n x^n, se x0
 p(x) = a_0, se x=0

 ou entao tirar o x^0 do somatorio:

 p(x) = a_0 + SUM (n=1 a 17) a_n x^n

 (e se voce acha que esta ultima eh bem razoavel -- escreva p'(x). Separou o
 a1? Argh!)

 Como eu nao tenho paciencia de ficar escrevendo este a_0 separado toda hora,
 prefiro logo pensar que 0^0=1 e resolvo meus problemas com um somatorio soh.
 :)

 Isto tudo dito, claro que eh soh uma convencao, questao de gosto. Mas eu
 *gosto* de 0^0=1. :)

 Abraco,
  Ralph

 2010/9/16 Jorge Luis Rodrigues e Silva Luis jorgelrs1...@hotmail.com

 Olá, Pessoal! Vale lembrar que o símbolo do nada está entre as mais
 importantes descobertas feita pelo homem. É difícil acreditar que os
 homens
 levaram 5 mil anos entre escrever números e conceber o nosso sistema de
 numeração posicional, ponto crucial num desenvolvimento sem o qual o
 progresso da ciência moderna seria inconcebível. Hoje parece simples, mas
 a
 mentalidade concreta dos antigos gregos, não podia conceber o vazio, o
 nada,
 como um número. Apreciaremos ainda mais a grandeza dessa conquista se
 lembrarmo-nos de que ela escapou ao gênio de Arquimedes e Apolônio, dois
 dos
 maiores homens da antiguidade.

 Existem situações em Análise Combinatória onde há uma certa conveniência
 em
 adotar a regra 0^0=1, a fim de estender um pouco mais o campo de validez
 de
 algumas fórmulas. Nem por isso 0^0 deixa de ser uma expressão
 indeterminada.
 Um caso parecido acontece na Teoria da Medida e da Integral, onde às vezes
 é
 conveniente escrever 0*...=0, a fim de que a fórmula da área de um
 retângulo
 continue válida quando a base do retângulo é toda uma reta e a altura se
 reduz a um ponto. O curioso é que os defensores de 0^0=1 não reivindiquem
 o
 mesmo direito para 0/0. Algum colega saberia o motivo?

 Afinal! Qual das medidas é a mais precisa? E a mais exata?  a)5,6m
 b)560m
 (com aproximação de 10m)   c) 0,056m   d)5600m (com aproximação de 100m)

 Quantos algarismos significativos temos nesta medida? X=(0,009050 + -
 0,02)

 A propósito! Como se escreve zero em algarismos romanos?


 Abraços!




-- 
/**/
Quadrinista e Taverneiro!

http://tavernadofimdomundo.blogspot.com  Quadrinhos, histórioas e afins
http://baratoeletrico.blogspot.com / Um pouco sobre elétrons em movimento
http://bridget-torres.blogspot.com/  Personal! Do not edit!

=
Instru��es para entrar na lista, sair da lista e usar a lista em
http://www.mat.puc-rio.br/~obmlistas/obm-l.html
=


[obm-l] Re: [obm-l] Teoria dos números

2010-09-24 Por tôpico Johann Dirichlet
1)
Basta demonstrar que (n^8+1)(n^8-1) é múltiplo de 17.
Mais isso sai direto de Euler-Fermat: 17 divide n^16-1 se n não é
múltiplo de 17.

2)

(2y+1)^2-4=x^3
Escrevendo z=2y-1:
(z-2)(z+2)=x^3
Veja que z-2 e z+2 não tem fatores comuns (ambos são ímpares
consecutivos), logo ambos são cubos perfeitos. Daqui fica fácil
seguir...

Em 23/09/10, warley ferreiralulu...@yahoo.com.br escreveu:
 Pessoal seria possível ajuda nestas questoes,

 Questão 1)
 Seja n um número natural não divisível por 17. Prove que n^8 + 1 ou n^8 -1 é
 divisível por 17.
 Questão 2)
 Determine, caso existam, as soluções inteiras da equação x^3+3 = 4y(y+1).

 Desde já agradeço,
  Warley Souza





-- 
/**/
Quadrinista e Taverneiro!

http://tavernadofimdomundo.blogspot.com  Quadrinhos, histórioas e afins
http://baratoeletrico.blogspot.com / Um pouco sobre elétrons em movimento
http://bridget-torres.blogspot.com/  Personal! Do not edit!

=
Instru��es para entrar na lista, sair da lista e usar a lista em
http://www.mat.puc-rio.br/~obmlistas/obm-l.html
=


Re: [obm-l] REPASSE COM URGENCIA, URGENCIA

2010-10-05 Por tôpico Johann Dirichlet
Legal, agora temos spam político na OBM-L...

Em 05/10/10, Luís Juniorjrcarped...@gmail.com escreveu:
 Concordo com todos os projetos.

 2010/10/5 Aline Rosane aline.ace...@hotmail.com




  *PENSE BEM ANTES DE VOTAR NA DONA DILMA E SUA GANG !!!
 *



 Cordialmente,
 Camargo Júnior
 69-3421-3061



 --
 *Resumo de projetos que podem virar lei após as eleição: ENTÃO VOTE NA
 DILMA*

 **

 *Fica proibido fazer:*
 ·   Cultos ou evangelismo na rua (Reforma Constitucional)
 ·   Programas evangélicos na televisão por mais de uma hora por dia.
 ·   Programa de rádio ou televisão, quem não possuir faculdade
 de'jornalismo'.
 ·   Pregar sobre dízimos e ofertas, havendo reclamações, obreiros serão
 presos.

 *Quanto aos cultos:*
 - Cultos somente com portas fechadas (Reforma Constitucional)
 - As igrejas serão obrigadas a pagarem impostos sobre dízimos,
 ofertas e contribuições,as católicas também.
 - Será considerado crime pregar sobre espiritismo, feitiçaria e idolatria,
 e também veicular mensagem no rádio, televisão, jornais e internet,
 sobre essas práticas contrárias a Palavra de Deus.
 - Pastores e Padres que forem presos por pregar sobre práticas condenadas
 pela
 Bíblia Sagrada (homossexualismo, idolatria e espiritismo), não terão
 direito a se defender por meio de ação judicial.

 *Se estabeleça:*
 -O dia do “Orgulho Gay” e que seja oficializado em todas as cidades
 brasileiras e comemorado nas Instituições de Ensino Fundamental
 (primeira a 8.a série), público e particular.
 -Que as Igrejas que se negarem a realização das solenidades
 dos casamentos de homem com homem e de mulher com mulher,
 estarão fazendo “discriminação”, seja multadas e seus pastores e padres
 processados criminalmente por discriminação e desobediência civil.


 *Projeto nº 4.720/03 - Altera a legislação constitucional** **

 -Projeto nº 3.331/04 –* Altera o artigo 12 da Lei nº 9.250/95, que trata
 da legislação do imposto de renda das 'pessoas físicas' Se convertidos em
 Lei, os dois projetos obrigariam as igrejas a recolherem impostos sobre
 dízimos, ofertas e contribuições.


 *-**Projeto nº 299/99 –* Altera o código brasileiro de telecomunicações
 (Lei 4.117/62).
 Se aprovado, reduziria programas evangélicos
 no rádio e televisão a apenas uma hora.
 **

 *-Projeto nº 6.398/05 –* Regulamenta a profissão de Jornalista
 Contém artigos que estabelecem que só poderão fazer programas
 de rádio e televisão, pessoas com formação em JORNALISMO, Significa que
 pastores,padres e ouros sem a formação em jornalismo não
 poderão fazer programas através desses meios.


 *-Projeto nº 1.154/03 –* Proíbe veiculação de programas  em que o teor
 seja considerado preconceito religioso. Se aprovado, será considerado
 crime
 pregar sobre idolatria, feitiçaria e rituais satânicos. Será proibido que
 mensagens sobre essas práticas sejam veiculadas no rádio, televisão,
 jornais
 e internet. A verdade sobre esses atos contrários
 a Palavra de Deus, não poderá mais ser mostrada.


 *-Projeto nº 952/03 – *Estabelece que é crime atos religiosos que possam
 ser considerados abusivos a boa-fé das pessoas. Convertido em Lei, pelo
 número de reclamações, pastores serão considerados 'criminosos' por
 pregarem
 sobre dízimos e ofertas.


 *-Projeto nº 4.270/04[/b] –** *Determina que comentários feitos contra
 ações praticadas por grupos religiosos possam ser passíveis de ação civil.
 Se convertido em Lei, as Igrejas Evangélicas ficariam proibidas de pregar
 sobre práticas condenadas pela Bíblia Sagrada, como espiritismo,
 feitiçaria,
 idolatria e outras. Se o fizerem, não terão direito a se defender por meio
 de ação judicial.


 *-**Projeto de nº 216/04[/b] –* Torna inelegível a função religiosa com a
 governamental. Significa que todo pastor ou líder religioso lançado a
 candidaturas para qualquer cargo político, não poderá de forma alguma
 exercer trabalhos na igreja.





-- 
/**/
Quadrinista e Taverneiro!

http://tavernadofimdomundo.blogspot.com  Quadrinhos, histórioas e afins
http://baratoeletrico.blogspot.com / Um pouco sobre elétrons em movimento
http://bridget-torres.blogspot.com/  Personal! Do not edit!

=
Instru��es para entrar na lista, sair da lista e usar a lista em
http://www.mat.puc-rio.br/~obmlistas/obm-l.html
=


[obm-l] Spam político e terrorismo, aqui não!! (era: REPAS SE COM URGENCIA, URGENCIA)

2010-10-05 Por tôpico Johann Dirichlet
Olá pessoas!
Bem, não sei se estou sendo redundandte ou mesmo se o que eu direi irá
ofender a inteligência dos presentes desta lista:
Pesquise as fontes sempre que surgir um e-mail ou mensagem destas por
esta lista (e de qualquer outro meio de comunicação). Como diziam na
Internet, O Google é seu amigo.

Enfim, é isso. Tem algum problema bom de Matemática aí?

Em 05/10/10, Saon Crispim Vieirasao...@gmail.com escreveu:
 Olá pessoal, boa tarde.
 Isto é pura fantasia! É inaceitável ouvir tais acusações sem indícios e/ou
 provas!
 Sou espírita e não eleitor da Dilma, mas este não é o nível de debate que
 espero desta lista.
 Segue anexa uma matéria interessante, de uma fonte confiável e com tema mais
 relacionado à proposta desta lista.

 http://www.nature.com/news/2010/100929/full/467511b.html?s=news_rss

 2010/10/5 Aline Rosane aline.ace...@hotmail.com




  *PENSE BEM ANTES DE VOTAR NA DONA DILMA E SUA GANG !!!
 *



 Cordialmente,
 Camargo Júnior
 69-3421-3061



 --
 *Resumo de projetos que podem virar lei após as eleição: ENTÃO VOTE NA
 DILMA*

 **

 *Fica proibido fazer:*
 ·   Cultos ou evangelismo na rua (Reforma Constitucional)
 ·   Programas evangélicos na televisão por mais de uma hora por dia.
 ·   Programa de rádio ou televisão, quem não possuir faculdade
 de'jornalismo'.
 ·   Pregar sobre dízimos e ofertas, havendo reclamações, obreiros serão
 presos.

 *Quanto aos cultos:*
 - Cultos somente com portas fechadas (Reforma Constitucional)
 - As igrejas serão obrigadas a pagarem impostos sobre dízimos,
 ofertas e contribuições,as católicas também.
 - Será considerado crime pregar sobre espiritismo, feitiçaria e idolatria,
 e também veicular mensagem no rádio, televisão, jornais e internet,
 sobre essas práticas contrárias a Palavra de Deus.
 - Pastores e Padres que forem presos por pregar sobre práticas condenadas
 pela
 Bíblia Sagrada (homossexualismo, idolatria e espiritismo), não terão
 direito a se defender por meio de ação judicial.

 *Se estabeleça:*
 -O dia do “Orgulho Gay” e que seja oficializado em todas as cidades
 brasileiras e comemorado nas Instituições de Ensino Fundamental
 (primeira a 8.a série), público e particular.
 -Que as Igrejas que se negarem a realização das solenidades
 dos casamentos de homem com homem e de mulher com mulher,
 estarão fazendo “discriminação”, seja multadas e seus pastores e padres
 processados criminalmente por discriminação e desobediência civil.


 *Projeto nº 4.720/03 - Altera a legislação constitucional** **

 -Projeto nº 3.331/04 –* Altera o artigo 12 da Lei nº 9.250/95, que trata
 da legislação do imposto de renda das 'pessoas físicas' Se convertidos em
 Lei, os dois projetos obrigariam as igrejas a recolherem impostos sobre
 dízimos, ofertas e contribuições.


 *-**Projeto nº 299/99 –* Altera o código brasileiro de telecomunicações
 (Lei 4.117/62).
 Se aprovado, reduziria programas evangélicos
 no rádio e televisão a apenas uma hora.
 **

 *-Projeto nº 6.398/05 –* Regulamenta a profissão de Jornalista
 Contém artigos que estabelecem que só poderão fazer programas
 de rádio e televisão, pessoas com formação em JORNALISMO, Significa que
 pastores,padres e ouros sem a formação em jornalismo não
 poderão fazer programas através desses meios.


 *-Projeto nº 1.154/03 –* Proíbe veiculação de programas  em que o teor
 seja considerado preconceito religioso. Se aprovado, será considerado
 crime
 pregar sobre idolatria, feitiçaria e rituais satânicos. Será proibido que
 mensagens sobre essas práticas sejam veiculadas no rádio, televisão,
 jornais
 e internet. A verdade sobre esses atos contrários
 a Palavra de Deus, não poderá mais ser mostrada.


 *-Projeto nº 952/03 – *Estabelece que é crime atos religiosos que possam
 ser considerados abusivos a boa-fé das pessoas. Convertido em Lei, pelo
 número de reclamações, pastores serão considerados 'criminosos' por
 pregarem
 sobre dízimos e ofertas.


 *-Projeto nº 4.270/04[/b] –** *Determina que comentários feitos contra
 ações praticadas por grupos religiosos possam ser passíveis de ação civil.
 Se convertido em Lei, as Igrejas Evangélicas ficariam proibidas de pregar
 sobre práticas condenadas pela Bíblia Sagrada, como espiritismo,
 feitiçaria,
 idolatria e outras. Se o fizerem, não terão direito a se defender por meio
 de ação judicial.


 *-**Projeto de nº 216/04[/b] –* Torna inelegível a função religiosa com a
 governamental. Significa que todo pastor ou líder religioso lançado a
 candidaturas para qualquer cargo político, não poderá de forma alguma
 exercer trabalhos na igreja.




 --
 Saon Crispim Vieira



-- 
/**/
Quadrinista e Taverneiro!

http://tavernadofimdomundo.blogspot.com  Quadrinhos, histórioas e afins
http://baratoeletrico.blogspot.com / Um pouco sobre elétrons em movimento
http://bridget-torres.blogspot.com/  Personal! Do not edit!

=
Instru��es para entrar 

Re: [obm-l] Semelhantes ou iguais?

2010-10-07 Por tôpico Johann Dirichlet
Pense no mapa de uma cidade. Os  Ãngulos são iguais, mas as distâncias não.

Em 07/10/10, Nathália Santosnathalia...@hotmail.com escreveu:
 Serão sempre semelhantes, mas não necessariamente iguais, já que ângulos
 iguais não determinam sempre lados iguais.

 From: rhilbert1...@hotmail.com
 To: obm-l@mat.puc-rio.br
 Subject: [obm-l] Semelhantes ou iguais?
 Date: Wed, 6 Oct 2010 22:25:19 +





 Colegas, uma discussão sem solução, acontenceu por conta da seguinte dúvida.

 Dois triângulos com os seus ângulos, respectivamente, de mesma medida, são
 iguais (lados  respectivos de mesma medida) ou semelhantes (lados
 respectivos proporcionais)?

 Exemplo: Triângulos ABC e A'B'C'  com   ângulos A=A, B=B e C=C  = AB=A'B',
 AC=A'C' e BC=B'C' ?










   
   


-- 
/**/
Quadrinista e Taverneiro!

http://tavernadofimdomundo.blogspot.com  Quadrinhos, histórioas e afins
http://baratoeletrico.blogspot.com / Um pouco sobre elétrons em movimento
http://bridget-torres.blogspot.com/  Personal! Do not edit!

=
Instru��es para entrar na lista, sair da lista e usar a lista em
http://www.mat.puc-rio.br/~obmlistas/obm-l.html
=


Re: [obm-l] Algoritmo de Euclides estendido

2010-10-20 Por tôpico Johann Dirichlet
Suponha que p é divisor de ab, mas não seja de a.
Então a e p serão primos entre si, e assim podemos achar x e y tais que
xa+yp=1
Multiplicando por b, temos
xab+ybp=b
Como xab e ybp são múltiplos de p, a soma também será. É isso!


Em 15/10/10, luizluizvalve...@globo.com escreveu:

 Alguem pode me ajudar.?





 O algoritmo de Euclides estendido é o seguinte:

 Dados a e b inteiros, seja d = mdc(a,b) então existem r e s inteiros tais
 que sa+rb=d.

 Usando o algoritmo de Euclides estendido mostre que se p é primo e a e b são
 inteiros tais que p é divisor de ab, então p é divisor de a ou p é divisor
 de b.



-- 
/**/
Quadrinista e Taverneiro!

http://tavernadofimdomundo.blogspot.com  Quadrinhos, histórioas e afins
http://baratoeletrico.blogspot.com / Um pouco sobre elétrons em movimento
http://bridget-torres.blogspot.com/  Personal! Do not edit!

=
Instru��es para entrar na lista, sair da lista e usar a lista em
http://www.mat.puc-rio.br/~obmlistas/obm-l.html
=


[obm-l] Re: [obm-l] Quadrados mágicos: problema da Eureka 0 1:

2010-10-24 Por tôpico Johann Dirichlet
O unico pre-requisito para se ler uma Eureka! e ler as anteriores.
Desculpe falar algo tao obvio, mas e que nao tem bem um pre-Eureka! no
Brasil, ate onde eu sei. Se voce encara uma leitura em ingles, a
melhor referencia que conheco e o site mathlinks.ro. La tem tutoriais
e artigos de todos os niveis. Tambem tem um arquivo de problemas de
olimpiadas de todo o mundo.
Por ora, esta e minha recomendacao: o site da OBM e as Eureka!s, todas elas.

Bem, eu pretendo lancar um arquivo pessoal contendo solucoes da
Eureka! e de alguns problemas que eu fiz ha milenios em papel, mas
ainda não posso garantir nada...

Quanto ao problema, tente resolver o sistema de equacoes gerado pelas
somas, e voce descobrira que o numero central e igual a 5. Depois eu
posto algo competo.

Em 23/10/10, Rafaelapolo_hiperbo...@terra.com.br escreveu:
 Olá, pessoal.

 Antes de comentar sobre um problema da Eureka 01, uma pergunta:

 Alguém aqui costuma resolver todos os problemas sem solução da revista
 Eureka e deixar em um arquivo no word, por exemplo ? Se sim, gostaria muito
 de um arquivo com esses problemas que contém apenas o gabarito e não a
 solução.

 Uma vez enviaram aqui um arquivo com questões resolvidas do IME, inclusive
 bem antigas. Alguém aqui tem arquivos de questões resolvidas assim também,
 mas olímpicas ? Seja da Eureka ou não. No site Excalibur, há muitos
 problemas assim, mas o nível é bem alto. Gostaria de um arquivo com
 problemas resolvidos de forma preparatória à leitura das Eurekas.

 Comecei a ler as Eurekas. Veja este problema da Eureka 01:

 Você já conhece o quadrado mágico de ordem 3: a soma dos números das linhas,
 das colunas e das diagonais é 15. A figura a seguir mostra uma das oito
 possibilidades de escrever os números no quadrado:

 a11 = 8; a12 = 1; a13 = 6
 a21 = 3; a22 = 5; a23 = 7
 a31 = 4; a32 = 9; a33 = 2

 O único número que não pode mudar de posição em todos esses quadrados
 mágicos é:



 a) 1  b) 3  c) 5  d) 7  e) 9


 Eu percebi que a correta é a C (gabarito), pois se girarmos o quadrado no
 sentido horário ou anti-horário, teremos 4 quadrados (incluindo o original)
 e em todos eles não houve mudança do número 5 em relação ao quadrado do
 enunciado. Eu gostaria de uma solução mais formal e por que são 8
 possibilidades e não 4.

 Obs1: Saber todo o conteúdo do ensino médio já é o suficiente para ler e
 entender as Revistas Eurekas OU deve haver uma outra condição prévia, como
 ler algum livro específico ou estudar por problemas de outros sites ?
 Pergunto isso, pois abri aleatoriamente algumas revistas e li alguns termos
 matemáticos não abordados em livros normais do ensino médio, daí pensei: -
 OU os elaboradores das Eureka estão partindo do pressuposto que os leitores
 já saibam determinadas coisas (mesmo que não estejam em livros regulares do
 ensino médio); OU há uma gradação de conhecimentos nas revistas, ou seja, se
 não entendeu algum termo ou conceito OLÍMPICOS em alguma revista, então é
 provável que haja um explicação em alguma das revistas anteriores.


 Regards,
 Rafael


-- 
/**/
Quadrinista e Taverneiro!

http://tavernadofimdomundo.blogspot.com  Quadrinhos, histórioas e afins
http://baratoeletrico.blogspot.com / Um pouco sobre elétrons em movimento
http://bridget-torres.blogspot.com/  Personal! Do not edit!

=
Instru��es para entrar na lista, sair da lista e usar a lista em
http://www.mat.puc-rio.br/~obmlistas/obm-l.html
=


[obm-l] Re: [obm-l] Parte fracionária = parte decimal?

2010-10-24 Por tôpico Johann Dirichlet
Com certeza! (eu acho...)

Por definicao, a parte inteira de um real e o maior inteiro que fica
abaixo deste real. Por exemplo, 7 e a parte inteira de 7,1234.

A parte fracionaria e esta diferenca entre o numero e sua parte
inteira. No caso, 0,1234.

O lance e que as vezes voce tem um numero feio. Por exemplo,
pi=3,141592..., e e mais pratico escrever pi-3 do que 0,141592...

Um exemplo: qual a parte inteira de  -3,1415?

Em 22/10/10, Guilherme Vieirarjguilhermevie...@hotmail.com escreveu:

 Caros Colegas,

 Dado o número decimal 7,1234, pode-se dizer que sua parte fracionária é
 0,1234?  Isto é, a parte fracionária é a parte decimal?

 Um abração!
 Guilherme

   


-- 
/**/
Quadrinista e Taverneiro!

http://tavernadofimdomundo.blogspot.com  Quadrinhos, histórioas e afins
http://baratoeletrico.blogspot.com / Um pouco sobre elétrons em movimento
http://bridget-torres.blogspot.com/  Personal! Do not edit!

=
Instru��es para entrar na lista, sair da lista e usar a lista em
http://www.mat.puc-rio.br/~obmlistas/obm-l.html
=


[obm-l] Re: [obm-l] Re: [obm-l] Fibonacci e Razão Áurea

2010-10-28 Por tôpico Johann Dirichlet
Poxa, alguém tem um exemplo de uma sequencia x_n que sempre é positiva
mas o limite não é?
Eu acho que 1/n tende a zero sempre sendo maior que zero, mas tem que
tomar cuidado com o estritamente positivo.

P.S.: um treco legal sobre racionais tendendo a irracionais é o artigo
do Gugu na Eureka! 3, sobre frações contínuas. Se eu não me engano os
F/F são reduzidas da fração contínua da razão áurea.


Em 27/10/10, Ralph Teixeiraralp...@gmail.com escreveu:
 Como pode uma razão de números inteiros convergir para um número irracional
 ?

 Bom, como ilustração, pi é irracional, e é o limite da sequencia:

 3
 3,1=31/10
 3,14=314/100
 3,141=3141/1000
 3,1415=31415/1
 ...

 Acho que este exemplo deve te convencer que qualquer número irracional é
 limite de uma sequencia de racionais (razões entre inteiros).

 ---///---

 Para ponderar: raciocínios do tipo: se cada x_n tem a propriedade P, então
 lim(x_n) tem a propriedade P são muito naturais. Infelizmente, este tipo de
 raciocínio está frequentemente errado! Por exemplo, seu espanto acima seria
 representado pela frase:

 se cada x_n é racional (quociente de inteiros), então lim(x_n), se existir,
 também será.
 (FALSO!)

 Outras frases FALSAS do mesmo tipo (todos os limites são quando n-+Inf):
 se cada x_n é positivo, então lim(x_n) é positivo.
 se cada x_n é menor que 1, então lim(x_n) é menor que 1 (que, no fundo no
 fundo, é o problema que o pessoal tem com 0,9...=1)
 se cada uma das funções f_n(x) é contínua, então f(x)=lim f_n(x) é
 contínua
 se cada uma das funções f_n(x) é derivável, então f(x)=lim f_n(x) é
 derivável

 Bom, e assim por diante. O que eu quero dizer é que passar um raciocínio
 ao limite é perigoso (mas, quando funciona, é bem legal)

 Abraço,

 Ralph


 2010/10/27 luiz silva luizfelipec...@yahoo.com.br

   Pessoal,

 Pelo que lembro, a razão entre dois números consecutivos(an+1/an), da
 sequência Fibonacci converge para 1,61834 quando n tende a infinito.
 Porém, pelo que lembro, tb, este número é um número irracional.

 Como pode uma razão de números inteiros convergir para um número
 irracional
 ?

 Abs
 Felipe





-- 
/**/
Quadrinista e Taverneiro!

http://tavernadofimdomundo.blogspot.com  Quadrinhos, histórioas e afins
http://baratoeletrico.blogspot.com / Um pouco sobre elétrons em movimento
http://bridget-torres.blogspot.com/  Personal! Do not edit!

=
Instru��es para entrar na lista, sair da lista e usar a lista em
http://www.mat.puc-rio.br/~obmlistas/obm-l.html
=


[obm-l] Re: [obm-l] Re: [obm-l] Determinar parte inteira e p arte fracionária

2010-10-28 Por tôpico Johann Dirichlet
Eu creio que a resposta é o famigerado -9.

Melhor ser mais preciso nas definições: parte inteira de x ou maior
inteiro que não supera x?

Em 27/10/10, Adalberto Dornellesaadornell...@gmail.com escreveu:
 Olá Pedro,

 A resposta depende de como você define parte fracionária. Parece que não
 há uma definição universalmente aceita. Veja por exemplo:
 http://mathworld.wolfram.com/FractionalPart.html

 http://mathworld.wolfram.com/FractionalPart.htmlTalvez a definição mais
 usual seja

 Parte fracionaria de x = x - arredondar para baixo x

 ou, em notação (também não muito universal),

 {x} = x - [x]

 Assim,
 {8,25} = 8,25 - [8,25] = 8,25 - 8 = 0,25
 e
 {-8,25} = -8,25 - [-8,25] = -8,25 - (-9) = 0,75

 Agora, observe que arredondar para baixo significa encontrar o inteiro n
 à esquerda de x ou arredondar na direção de - infinito. No entanto, outra
 definição de arredondar para baixo pode ser arredondar na direção de 0,
 assim

 {8,25} = 8,25 - [8,25] = 8,25 - 8 = 0,25
 e
 {-8,25} = -8,25 - [-8,25] = -8,25 - (-8) = -0,25

 Em computação, geralmente as linguagens [Matlab, Mathematica, etc.] tem
 comandos relacionados a isso (floor, ceil, fix, round, mod, ...). cuja
 definição não é muito homogênea. Sempre recomendo leia o manual para saber
 como a linguagem define cada coisa.

 Veja também http://en.wikipedia.org/wiki/Floor_and_ceiling_functions

 Abraço,
 Adalberto

 Em 27 de outubro de 2010 08:09, Pedro Chaves brped...@hotmail.comescreveu:

  Caros Colegas,

 Qual é a parte inteira e a parte fracionária do número real negativo
 -8,25?

 Obrigado!
 Pedro Chaves




-- 
/**/
Quadrinista e Taverneiro!

http://tavernadofimdomundo.blogspot.com  Quadrinhos, histórioas e afins
http://baratoeletrico.blogspot.com / Um pouco sobre elétrons em movimento
http://bridget-torres.blogspot.com/  Personal! Do not edit!

=
Instru��es para entrar na lista, sair da lista e usar a lista em
http://www.mat.puc-rio.br/~obmlistas/obm-l.html
=


[obm-l] Re: [obm-l] Conjuntos Enumeráveis

2010-10-30 Por tôpico Johann Dirichlet
A ideia não é difícil, e o mais importante é o caso 2: X x Yé
enumerável se X,Y são.
Faz assim: os elementos de X são x1,x2,... e os de Y são y1,y2,y3...
(ambos são enumeráveis, então eu posso colocar índices)

Então podemos fazer assim:
Para cada natural N = 1,2,3,4,5...
liste os pares (xi,yj) tal que i+j=N

Teremos algo assim:
(x1,y1)
(x1,y2),(x2,y1)
(x1,y3),(x2,y2),(x3,y1)
E por aí vai...

Aí, basta aplicar o caso n=2 fazendo X=A1 x A2 x ... x An e Y=A(n+1)

Sem indução é mais fácil ainda: basta utilizar o algoritmo acima.

Em 30/10/10, Luiz Neto Netouizn...@yahoo.com.br escreveu:
 Sejam A1,An conjuntos enumeráveis, então A1xxAn é enumerável(Use
 Indução)






-- 
/**/
Quadrinista e Taverneiro!

http://tavernadofimdomundo.blogspot.com  Quadrinhos, histórioas e afins
http://baratoeletrico.blogspot.com / Um pouco sobre elétrons em movimento
http://bridget-torres.blogspot.com/  Personal! Do not edit!

=
Instru��es para entrar na lista, sair da lista e usar a lista em
http://www.mat.puc-rio.br/~obmlistas/obm-l.html
=


[obm-l] Olimpíadas ao Redor do Mundo

2010-10-30 Por tôpico Johann Dirichlet
Eis um problema legal:

Temos três caixas, cada uma com pelo menos uma bolinha dentro.
Podemos dobrar o total de bolinhas de uma das caixas, tirando as
bolinhas de uma das outras caixas para tal.
É possível esvaziar uma das caixas, fazendo uma escolha acertada de
operações permitidas?

-- 
/**/
Quadrinista e Taverneiro!

http://tavernadofimdomundo.blogspot.com  Quadrinhos, histórioas e afins
http://baratoeletrico.blogspot.com / Um pouco sobre elétrons em movimento
http://bridget-torres.blogspot.com/  Personal! Do not edit!

=
Instru��es para entrar na lista, sair da lista e usar a lista em
http://www.mat.puc-rio.br/~obmlistas/obm-l.html
=


Re: [obm-l] Teorema sobre logaritmos irracionais

2010-11-09 Por tôpico Johann Dirichlet
log_b a= x é o mesmo que a^x=b.
Usando o lema da fatoração única, vemos que se x fosse racional então
a e b teriam os mesmos fatores primos e com os expoentes múltiplos.

Em 06/11/10, Pedro Chavesbrped...@hotmail.com escreveu:

 Estou reapresentando o teorema sobre logaritmos, pois não consegui ainda uma
 demonstração completa. Peço, mais uma vez, a colaboração dos colegas.

 Teorema:
 Sendo a e b números inteiros maiores do que 1, que não podem ser
 representados como potências de mesma base (inteira), com expoente inteiro,
 então o logaritmo de a, na base b, é um número irracional.


 Um abraço do Pedro!   


-- 
/**/
Quadrinista e Taverneiro!

http://tavernadofimdomundo.blogspot.com  Quadrinhos, histórioas e afins
http://baratoeletrico.blogspot.com / Um pouco sobre elétrons em movimento
http://bridget-torres.blogspot.com/  Personal! Do not edit!

=
Instru��es para entrar na lista, sair da lista e usar a lista em
http://www.mat.puc-rio.br/~obmlistas/obm-l.html
=


[obm-l] Re: [obm-l] Alguém conseguirá provar?

2010-11-21 Por tôpico Johann Dirichlet
O que significa limitada primorialmente?


Em 05/11/10, Marco Bivarmarco.bi...@gmail.com escreveu:
 Prove isto:

 Em toda sucessão (c_1, c_2, ..., c_w) de números compostos limitada
 primorialmente, se c_i = z_i . x_i, i=1,2,..., w, onde z_i é um primo ou
 produto de primos tal que z_i=x_i e z_1z_2...z_w, então x_1x_2...x_w,
 onde x_i é um primo ou produto de primos.

 Obs.: Tomar apenas números primos positivos.


 --
 Marco A. B. C. Jr.



-- 
/**/
Quadrinista e Taverneiro!

http://tavernadofimdomundo.blogspot.com  Quadrinhos, histórioas e afins
http://baratoeletrico.blogspot.com / Um pouco sobre elétrons em movimento
http://bridget-torres.blogspot.com/  Personal! Do not edit!

=
Instru��es para entrar na lista, sair da lista e usar a lista em
http://www.mat.puc-rio.br/~obmlistas/obm-l.html
=


[obm-l] Re: [obm-l] mdc (a^x – 1, a^y – 1, a^z – 1, .. .......) = [a^mdc(x, y, z,...)] – 1

2010-11-23 Por tôpico Johann Dirichlet
Para dois caras, é fácil demonstrar na raça, usando Euclides:
MDC(a^x-1,a^y-1)= MDC(a^x-1,a^(x-y)-1). Daí se faz por indução no
número de variáveis.

Em 23/11/10, Paulo  Argolopauloarg...@bol.com.br escreveu:
 Caros Colegas,
 Estou refazendo o enunciado da questão.

 Como provar o teorema seguinte sobre máximo divisor comum?

 TEOREMA:

 O máximo divisor comum (mdc) dos números do tipo
 a^x -1 , onde a e x são números inteiros maiores do que 1, é dado pela
 expressão abaixo:
 mdc(a^x - 1, a^y - 1, a^z - 1, ... ) = [a^mdc(x, y, z,...)] -1

 Grato,

 Paulo Argolo
 =
 Instru�ões para entrar na lista, sair da lista e usar a lista em
 http://www.mat.puc-rio.br/~obmlistas/obm-l.html
 =



-- 
/**/
Quadrinista e Taverneiro!

http://tavernadofimdomundo.blogspot.com  Quadrinhos, histórioas e afins
http://baratoeletrico.blogspot.com / Um pouco sobre elétrons em movimento
http://bridget-torres.blogspot.com/  Personal! Do not edit!

=
Instru��es para entrar na lista, sair da lista e usar a lista em
http://www.mat.puc-rio.br/~obmlistas/obm-l.html
=


[obm-l] Re: [obm-l] Como provar que C(n,p) é número natura l?

2010-11-24 Por tôpico Johann Dirichlet
A melhor que eu posso imaginar e simplesmente pensar assim:

1 - Determine, para cada primo p, a maior potencia de p que divide n!
(ou seja, descubra na raça a fatoração de n!).
E facil: basta contar quanto cada p, 2p, 3p, ... (p-1)p, p^2, etc vai
contribuir (voce vai obter um somatorio).
Isso tem mais a ver com teoria dos conjuntos que com teoria dos números.

2 - A partir dai, fica facil provar que a potencia de p que aparece no
numerador não é menor que no denominador.



Em 23/11/10, Pedro Chavesbrped...@hotmail.com escreveu:

 Amigos da Lista,


 Como posso provar que C(n,p) é um número natural, usando apenas a definição
 C(n,p) = n! / [p! (n-p)!]?
 (p e n são números naturais, com p menor ou igual a n)
 Meu objetivo é obter uma prova direta, isto é, que não recorra à Análise
 Combinatória, nem às  propriedades dos números binomiais.

 Muito grato!

 Pedro Chaves
   


-- 
/**/
Quadrinista e Taverneiro!

http://tavernadofimdomundo.blogspot.com  Quadrinhos, histórioas e afins
http://baratoeletrico.blogspot.com / Um pouco sobre elétrons em movimento
http://bridget-torres.blogspot.com/  Personal! Do not edit!

=
Instru��es para entrar na lista, sair da lista e usar a lista em
http://www.mat.puc-rio.br/~obmlistas/obm-l.html
=


Re: [obm-l] geometria com 20 graus

2010-11-24 Por tôpico Johann Dirichlet
Se não me engano este problema foi proposto numa Eureka! Assim que der
eu vejo qual o número, mas é recente (entre as últimas 8 ou 10).

Em 16/11/10, Luís Lopesqed_te...@hotmail.com escreveu:

 Sauda,c~oes,

 Pediram-me a solução do problema abaixo. Como muito provavelmente
 tal problema já apareceu por aqui, pergunto se alguém teria a solução
 dele à mão.

 Obrigado.

 []'s
 Luís




  Prezado Luís mais uma vez venho pedir a sua ajuda na solução do exercicio
 abaixo.
 Dado o triângulo ABC ,isósceles, com BÂC medindo 20 graus ,onde AC = AB.
 Pelo vértice B traça-se até AC, BD tal que AD = BC. Calcule o valor do
 ângulo B^DC.


   


-- 
/**/
Quadrinista e Taverneiro!

http://tavernadofimdomundo.blogspot.com  Quadrinhos, histórioas e afins
http://baratoeletrico.blogspot.com / Um pouco sobre elétrons em movimento
http://bridget-torres.blogspot.com/  Personal! Do not edit!

=
Instru��es para entrar na lista, sair da lista e usar a lista em
http://www.mat.puc-rio.br/~obmlistas/obm-l.html
=


[obm-l] Re: [obm-l] Re: [obm-l] Re: [obm-l] O produto de n i nteiros consecutivos é múltiplo do fatorial de n

2010-11-28 Por tôpico Johann Dirichlet
Por que este povo tem tanto pavor de uma prova que não use outros
conceitos alem do enunciado?
Eu mesmo conheço vários problemas que são resolvidos usando outras
técnicas. Na IMO de Glasgow teve um problema de Teoria dos Números com
uma solução que usava polinômios. E tem um monte de problemas de
teoria dos números que se resolvem usando técnicas de combinatória (o
teorema de Euler-Fermat, por exemplo).

De todo modo, só pra não perder o propósito da mensagem:

Uma maneira seria observar que f(n,k)=(k+1)(k+2)...(k+n)/n! é um
polinômio de grau n em k.
Ele é completamnte determinado se eu utilizar (n+1) valores de k.

Para k de -1 até -n, este polinômio é igual a zero, e para k=n+1 ele vale 1.
A partir daí, usando a fórmula de interpolação de Newton (ou uma
modificação do triângulo de Pascal), este polinômio é inteiro para
todo n inteiro.


Em 27/11/10, Carlos Alberto da Silva Victorvictorcar...@globo.com escreveu:
 Olá Paulo,
 Verifique se esta ideia satisfaz o que desejas .

  Por indução :

 1) para n=1,2 e 3 é fácil de observar tal fato .
 2) hipótese : válida para  n fatores consecutivos.

 3) Tomemos (n+1) fatores consecutivos :P =  k(k+1)(k+n-1).(k+n) .Por
 hipótese k(k+1)(k+n-1) é divisível por n! . Não é difícil mostrar que o
 produto de n fatores consecutivos é divisível por n .Como P possui (n+1)
 fatores, temos que o valor (n+1) está em um dos fatores(ou divisor de um dos
 fatores) de P e, já que n e (n+1) são primos entre si , P será divisível por
 n! e (n+1) , ou seja, divisível por (n+1)! , ok ?

 Abraços

 Carlos  Victor





 Em 27 de novembro de 2010 18:29, Paulo Argolo
 argolopa...@hotmail.comescreveu:

  Obrigado, Tiago.

 O que desejo, na verdade, é obter uma demonstração que não use
 propriedades
 dos coeficientes binomiais, nem recorra à Análise Combinatória. Em suma:
 gostaria de ver uma prova puramente aritmética.

 Abraços do Paulo!






-- 
/**/
Quadrinista e Taverneiro!

http://tavernadofimdomundo.blogspot.com  Quadrinhos, histórioas e afins
http://baratoeletrico.blogspot.com / Um pouco sobre elétrons em movimento
http://bridget-torres.blogspot.com/  Personal! Do not edit!

=
Instru��es para entrar na lista, sair da lista e usar a lista em
http://www.mat.puc-rio.br/~obmlistas/obm-l.html
=


[obm-l] Re: [obm-l] Re: [obm-l] Re: [obm-l] Re: [obm-l] Re: [obm-l] O produto de n inteiros consecutivos é múltiplo do fatorial de n

2010-12-09 Por tôpico Johann Dirichlet
Bem, respondendo:
1 - Errei: para k=0 o valor é 1
2 - Tem uma especie de dispositivo pratico, que funciona na mesma
ideia do triangulo de Pascal:

0 0 0 0 0 ... 0 1
 0 0 0 0 ... 0 1
  0 0 0 ... 0 1
   0 0 ... 0 1
 0 ... 1

  1

Este e o triangulo das diferenças de f(n,k).
Depois de um numero finito de passos (n+1, se nao me engano) a ultima
linha fica constante (neste caso igual a 1).
Ai e so reverter...

Existe uma formula pronta, mas eu quase nao decoro...

Em 09/12/10, Henrique Rennóhenrique.re...@gmail.com escreveu:
 Em 28/11/10, Johann Dirichletpeterdirich...@gmail.com escreveu:
 Por que este povo tem tanto pavor de uma prova que não use outros
 conceitos alem do enunciado?
 Eu mesmo conheço vários problemas que são resolvidos usando outras
 técnicas. Na IMO de Glasgow teve um problema de Teoria dos Números com
 uma solução que usava polinômios. E tem um monte de problemas de
 teoria dos números que se resolvem usando técnicas de combinatória (o
 teorema de Euler-Fermat, por exemplo).

 De todo modo, só pra não perder o propósito da mensagem:

 Uma maneira seria observar que f(n,k)=(k+1)(k+2)...(k+n)/n! é um
 polinômio de grau n em k.
 Ele é completamnte determinado se eu utilizar (n+1) valores de k.

 Para k de -1 até -n, este polinômio é igual a zero, e para k=n+1 ele vale
 1.
 A partir daí, usando a fórmula de interpolação de Newton (ou uma
 modificação do triângulo de Pascal), este polinômio é inteiro para
 todo n inteiro.

 Como isso pode ser verificado?



 Em 27/11/10, Carlos Alberto da Silva Victorvictorcar...@globo.com
 escreveu:
 Olá Paulo,
 Verifique se esta ideia satisfaz o que desejas .

  Por indução :

 1) para n=1,2 e 3 é fácil de observar tal fato .
 2) hipótese : válida para  n fatores consecutivos.

 3) Tomemos (n+1) fatores consecutivos :P =  k(k+1)(k+n-1).(k+n) .Por
 hipótese k(k+1)(k+n-1) é divisível por n! . Não é difícil mostrar que
 o
 produto de n fatores consecutivos é divisível por n .Como P possui (n+1)
 fatores, temos que o valor (n+1) está em um dos fatores(ou divisor de um
 dos
 fatores) de P e, já que n e (n+1) são primos entre si , P será divisível
 por
 n! e (n+1) , ou seja, divisível por (n+1)! , ok ?

 Abraços

 Carlos  Victor





 Em 27 de novembro de 2010 18:29, Paulo Argolo
 argolopa...@hotmail.comescreveu:

  Obrigado, Tiago.

 O que desejo, na verdade, é obter uma demonstração que não use
 propriedades
 dos coeficientes binomiais, nem recorra à Análise Combinatória. Em suma:
 gostaria de ver uma prova puramente aritmética.

 Abraços do Paulo!






 --
 /**/
 Quadrinista e Taverneiro!

 http://tavernadofimdomundo.blogspot.com  Quadrinhos, histórioas e afins
 http://baratoeletrico.blogspot.com / Um pouco sobre elétrons em
 movimento
 http://bridget-torres.blogspot.com/  Personal! Do not edit!

 =
 Instru�ões para entrar na lista, sair da lista e usar a lista em
 http://www.mat.puc-rio.br/~obmlistas/obm-l.html
 =



 --
 Henrique

 =
 Instru�ões para entrar na lista, sair da lista e usar a lista em
 http://www.mat.puc-rio.br/~obmlistas/obm-l.html
 =



-- 
/**/
Quadrinista e Taverneiro!

http://tavernadofimdomundo.blogspot.com  Quadrinhos, histórioas e afins
http://baratoeletrico.blogspot.com / Um pouco sobre elétrons em movimento
http://bridget-torres.blogspot.com/  Personal! Do not edit!

=
Instru��es para entrar na lista, sair da lista e usar a lista em
http://www.mat.puc-rio.br/~obmlistas/obm-l.html
=


[obm-l] Re: [obm-l] Campeões da Matemática - Resultados da 32a. Olimpíada Brasileira de Matemática - OBM

2010-12-19 Por tôpico Johann Dirichlet
faltou o resultado do Torneio das Cidades :)
Apesar de eu nao saber se ele e realizado pela SBM ou OBM, seria bom
ter alguma info.


Em 16/12/10, Olimpiada Brasileira de Matematicao...@impa.br escreveu:

 Campeões da Matemática - Resultados da 32a. Olimpíada Brasileira de
 Matemática - OBM

 Por mais um ano consecutivo estamos finalizando a realização da
 Olimpíada Brasileira de Matemática. Conheça a listagem de premiados da
 OBM -- 2010 no endereço: www.obm.org.br http://www.obm.org.br/

 A Olimpíada Brasileira de Matemática tem crescido nos últimos anos,
 contando, este ano, com a adesão ao Programa de mais de 4.651 escolas,
 sendo 2.770 da rede pública e 1.881 da rede privada de ensino, o que
 implica em uma participação na Olimpíada Brasileira de Matemática de
 cerca de 350.000 jovens estudantes e seus professores.Além disso, o
 Programa Nacional de Olimpíadas de Matemática conta com a colaboração de
 professores universitários em 155 instituições de ensino superior: eles
 participam de todas as atividades da Olimpíada Brasileira de Matemática,
 inclusive aquelas referentes à OBM Nível Universitário em atividades de
 coordenação, divulgação, treinamento de alunos, aperfeiçoamento de
 professores e aplicação das distintas fases da Olimpíada Brasileira de
 Matemática.

 No que se refere à participação em competições internacionais, os
 resultados são excelentes:

 * Olimpíada de Matemática do Cone Sul (Águas de São Pedro -- São
   Paulo): uma medalha de Ouro, duas de Prata e uma de Bronze.
 * Olimpíada Internacional de Matemática -- IMO (Astana --
   Cazaquistão): duas medalhas de Prata, uma medalha de Bronze.
 * Olimpíada Internacional de Matemática para Estudantes
   Universitários -- IMC (Blagoevgrad -- Bulgária): uma medalha de
   Ouro, duas medalhas de Prata, oito medalhas de Bronze e oito
   Menções Honrosas.
 * Olimpíada Iberoamericana de Matemática (Assunção -- Paraguai.):
   três medalhas de Ouro e uma medalha de Prata.
 * Competição Interuniversitária Iberoamericana de Matemática -- CIIM
   (Rio de Janeiro -- RJ): seis medalhas de Ouro (sendo duas de ouro
   especial), cinco medalhas de Prata e oito medalhas de Bronze.
 * Olimpíada de Maio (Organizada pela Argentina): duas medalhas de
   Ouro, quatro medalhas de Prata e oito medalhas de Bronze
   distribuídas entre os dois níveis da competição.
 * Asian Pacific Mathematical Olympiad (APMO): três medalhas de
   Prata, quatro medalhas de Bronze distribuídas entre os dois níveis
   da competição.
 * Romanian Máster in Mathematics (RMM) (Bucarest -- Romênia): três
   medalhas de Bronze e uma menção honrosa.
 * Este ano também realizamos o Concurso Canguru Matemático Sem
   Fronteiras, concurso que reúne participantes de 42 países.

 Durante 2009 a CAPES e o CNPq lançaram o Programa de Iniciação
 Científica -- Mestrado (PICME) para medalhistas da OBMEP e OBM, com o
 objetivo de aumentar o número de matemáticos no país, e oferecer uma
 formação matemática mais sólida a jovens profissionais de outras áreas
 científicas e tecnológicas.

 Todos estes resultados nacionais e internacionais demonstram que, além
 de influenciar positivamente o ensino da Matemática nas instituições de
 ensino fundamental, médio e superior, conseguimos detectar jovens muito
 talentosos que são estimulados a seguir uma carreira científica, o que é
 fundamental para o crescimento da Ciência e Tecnologia no país.

 A Olimpíada Brasileira de Matemática é um projeto conjunto da Sociedade
 Brasileira de Matemática (SBM), do Instituto Nacional de Matemática Pura
 e Aplicada (IMPA) e conta com o apoio do Conselho Nacional de
 Desenvolvimento Científico e Tecnológico (CNPq) e do Instituto Nacional
 de Ciência e Tecnologia de Matemática (INCT -- Mat).

 *Informações:*

 --
 Secretaria da Olimpíada Brasileira de Matemática
 Estrada Dona Castorina, 110 Jd. Botânico,
 Rio de Janeiro - RJ, 22460-320, Brasil
 Tel: 55-21-25295077 Fax:55-21-25295023
 e-mail:o...@impa.br
 web site:www.obm.org.br




-- 
/**/
Quadrinista e Taverneiro!

http://tavernadofimdomundo.blogspot.com  Quadrinhos, histórioas e afins
http://baratoeletrico.blogspot.com / Um pouco sobre elétrons em movimento
http://bridget-torres.blogspot.com/  Personal! Do not edit!

=
Instru��es para entrar na lista, sair da lista e usar a lista em
http://www.mat.puc-rio.br/~obmlistas/obm-l.html
=


[obm-l] Demonstrar Frações Parciais com Álgebra Linear

2010-12-19 Por tôpico Johann Dirichlet
Olá pessoas!

Faz algum tempo atrás, eu tinha um livro de Cálculo 1 + Álgebra
Linear. Entre outras coisas, ele ensinava a calcular integrais de
funcoes racionais (aquelas que estao ficando famosas na lista:
integral de (P(x)/Q(x)), em que P e Q são polinômios).

Nisto, ele tinha um apêndice em que demonstrava, usando Álgebra
Linear, que e possivel quebrar P/Q em fracoes parciais.

Mais precisamente, todos devem conhecer o resultado: se o grau de P e
menor que o grau de Q, e Q se fatora como (x-r)^m, as fracoes parciais
tem a forma
C/(x-r)^(1)+C/(x-r)^(2)+...+C/(x-r)^(m)

Pois bem, eu nao tenho mais o livro :(
Portanto, eu queria uma demonstração usando Álgebra Linear do fato acima.
Eu lembro que era algo corriqueiro: demonstrar que as fracoes acima
formavam um espaco vetorial de dimensao K, e depois achar uma K-base.
Mas os detalhes me fogem...

Desde já, agradeço!

-- 
/**/
Quadrinista e Taverneiro!

http://tavernadofimdomundo.blogspot.com  Quadrinhos, histórioas e afins
http://baratoeletrico.blogspot.com / Um pouco sobre elétrons em movimento
http://bridget-torres.blogspot.com/  Personal! Do not edit!

=
Instru��es para entrar na lista, sair da lista e usar a lista em
http://www.mat.puc-rio.br/~obmlistas/obm-l.html
=


[obm-l] Re: [obm-l] Re: [obm-l] Demonstrar Frações Parciai s com Álgebra Linear

2010-12-19 Por tôpico Johann Dirichlet
O titulo era simplesmente O Calculo com Algebra Linear. Nao sei nem
os autores direito... Ele versava sobre Calculo e bem pouco sobre
AlgeLin, A mais marcante aplicação foi justamente esta.

Em 19/12/10, João Luís Gomes Guimarãesjoaolui...@uol.com.br escreveu:

 Olá Johann,

 Não se lembra qual era o livro?

 JL

 -Mensagem Original-
 From: Johann Dirichlet
 Sent: Sunday, December 19, 2010 3:05 PM
 To: obm-l
 Subject: [obm-l] Demonstrar Frações Parciais com Álgebra Linear

 Olá pessoas!

 Faz algum tempo atrás, eu tinha um livro de Cálculo 1 + Álgebra
 Linear. Entre outras coisas, ele ensinava a calcular integrais de
 funcoes racionais (aquelas que estao ficando famosas na lista:
 integral de (P(x)/Q(x)), em que P e Q são polinômios).

 Nisto, ele tinha um apêndice em que demonstrava, usando Álgebra
 Linear, que e possivel quebrar P/Q em fracoes parciais.

 Mais precisamente, todos devem conhecer o resultado: se o grau de P e
 menor que o grau de Q, e Q se fatora como (x-r)^m, as fracoes parciais
 tem a forma
 C/(x-r)^(1)+C/(x-r)^(2)+...+C/(x-r)^(m)

 Pois bem, eu nao tenho mais o livro :(
 Portanto, eu queria uma demonstração usando Álgebra Linear do fato acima.
 Eu lembro que era algo corriqueiro: demonstrar que as fracoes acima
 formavam um espaco vetorial de dimensao K, e depois achar uma K-base.
 Mas os detalhes me fogem...

 Desde já, agradeço!

 --
 /**/
 Quadrinista e Taverneiro!

 http://tavernadofimdomundo.blogspot.com  Quadrinhos, histórioas e afins
 http://baratoeletrico.blogspot.com / Um pouco sobre elétrons em movimento
 http://bridget-torres.blogspot.com/  Personal! Do not edit!

 =
 Instru��es para entrar na lista, sair da lista e usar a lista em
 http://www.mat.puc-rio.br/~obmlistas/obm-l.html
 =


 =
 Instru�ões para entrar na lista, sair da lista e usar a lista em
 http://www.mat.puc-rio.br/~obmlistas/obm-l.html
 =



-- 
/**/
Quadrinista e Taverneiro!

http://tavernadofimdomundo.blogspot.com  Quadrinhos, histórioas e afins
http://baratoeletrico.blogspot.com / Um pouco sobre elétrons em movimento
http://bridget-torres.blogspot.com/  Personal! Do not edit!

=
Instru��es para entrar na lista, sair da lista e usar a lista em
http://www.mat.puc-rio.br/~obmlistas/obm-l.html
=


Re: [obm-l] Teoria dos Conjuntos

2011-01-01 Por tôpico Johann Dirichlet
Bem, isto me parece um pouco com um daqueles paradoxos (acho que o de
Russel). Acho que nao e muito facil construir uma coisa dessas.
Antes de mais nada, isto vai incorrer em perguntas do tipo mas isto é
um axioma da teoria dos conjuntos?. Como eu conheço bem pouco, eu
prefiro dar uma referência: Paul Halmos, Naive Set Theory (Teoria
Ingenua dos Conjuntos),. aliás um livro nada ingênuo :P

P.S.:  Um errinho de portugues: este consertando e com s. Mas acho que
se eu repetir isto provocarei uma flame war...

Em 22/12/10, Vinícius Harlockcortes...@gmail.com escreveu:
 Concertando:

 Agora eu só quero um exemplo.
 Seja R o conjunto dos números reais, diga-me um elemento que pertença ao
 conjunto A, tal que A={x∈R|x∉x} e outro elemento que pertença a B, tal  B={x
 ∈R|x∈x}. Se por acaso R não servir como conjunto universo, troque-o por um
 que sirva.


 Agradeço muito a atenção de todos =8^B
 a[b]'s



-- 
/**/
Quadrinista e Taverneiro!

http://tavernadofimdomundo.blogspot.com  Quadrinhos, histórioas e afins
http://baratoeletrico.blogspot.com / Um pouco sobre elétrons em movimento
http://bridget-torres.blogspot.com/  Personal! Do not edit!

=
Instru��es para entrar na lista, sair da lista e usar a lista em
http://www.mat.puc-rio.br/~obmlistas/obm-l.html
=


[obm-l] Re: [obm-l] SEMANA OLÍMPICA

2011-01-01 Por tôpico Johann Dirichlet
Bem, eu direi em carater nao-oficial: acho que ate os medalhistas de
prata sao bancados pela OBM. Ja o bronze, e uma especie de
50-50(passagem, mas nao os dias de hospedagem), e os menção honrosa e
100-0.
De todo modo, se voce foi agraciado com um premio, voce sera melhor informado.

Em 28/12/10, charles9char...@gmail.com escreveu:
 Os premiados com medalha do nível universitário recebem medalha como nos
 outros níveis?
 E quanto a semana olímpica, devemos pagar para participar?

 Obrigado.



-- 
/**/
Quadrinista e Taverneiro!

http://tavernadofimdomundo.blogspot.com  Quadrinhos, histórioas e afins
http://baratoeletrico.blogspot.com / Um pouco sobre elétrons em movimento
http://bridget-torres.blogspot.com/  Personal! Do not edit!

=
Instru��es para entrar na lista, sair da lista e usar a lista em
http://www.mat.puc-rio.br/~obmlistas/obm-l.html
=


[obm-l] Prova da OBM-nivel U (problema 6)

2011-01-01 Por tôpico Johann Dirichlet
Ola povo!
Alguem resolveu o problema 6 da OBMU?
Se p e um primo da forma 60k+7, e p divide 10^(2n)+8*10^n+1, então n
e k são ambos pares.

Por ora, não estou conseguindo ter nenhuma ideia... Por ora, pensei em
hensel, mas nao testei ainda.

-- 
/**/
Quadrinista e Taverneiro!

http://tavernadofimdomundo.blogspot.com  Quadrinhos, histórioas e afins
http://baratoeletrico.blogspot.com / Um pouco sobre elétrons em movimento
http://bridget-torres.blogspot.com/  Personal! Do not edit!

=
Instru��es para entrar na lista, sair da lista e usar a lista em
http://www.mat.puc-rio.br/~obmlistas/obm-l.html
=


[obm-l] Re: [obm-l] Re: [obm-l] RE: [obm-l] Infinitas soluç ões(números inteiros)

2011-01-10 Por tôpico Johann Dirichlet
Eu propus este problema (na verdade uma versao) na Eureka!
Bem, ele ja foi resolvido, no numero 30 se nao me engano.

Em 09/01/11, Ralph Teixeiraralp...@gmail.com escreveu:
 Aprendi esta ideia num problema de uma IMO:

 -- (1,1,1) eh solucao.
 -- Pense na equacao como uma quadratica em x: x^2-(3yz)x+(y^2+z^2)=0.
 A soma das raizes eh 3yz. Entao, se x=a eh uma solucao, a outra eh
 x=3yz-a.
 -- Em outras palavras, o que mostramos eh que se (x,y,z) eh solucao,
 entao (3yz-x,y,z) tambem eh (o que poderia ser verificado
 diretamente).
 -- Por simetria, (x,3xz-y,z) e (x,y,3xy-z) tambem servem.
 -- Isso gera uma famila de solucoes:
 (1,1,1) - (3.1.1-1,1,1)=(2,1,1) - (2,3.1.1-1,1)=(2,5,1) -
 (2,5,3.2.5-1)=(2,5,29) - (3.5.29-2,5,29) - ...-
 -- Note que na construcao eu escolhi sempre trocar o MENOR dos numeros
 (x,y,z). Entao a cada passo a soma da terna passou de x+y+z para
 (3yz-x)+(y+z), onde x eh o menor dos tres numeros. Como claramente
 3yz-xx (pois 3yz=3.x.12x), entao a soma eh estritamente crescente,
 e as ternas sao todas diferentes.

 Abraco, Ralph.

 2011/1/9 marcone augusto araújo borges marconeborge...@hotmail.com:
 corrigindo: x^2 + y^2 + z^2 = 3xyz

 
 From: marconeborge...@hotmail.com
 To: obm-l@mat.puc-rio.br
 Subject: [obm-l] Infinitas soluções(números inteiros)
 Date: Sun, 9 Jan 2011 02:10:07 +

 mostre q a equação x^2 + y^2 +z^2 = xyz tem infinitas soluções onde x,y,z
 são números inteiros.
 Agradeço a todos q ajudarem.


 =
 Instruções para entrar na lista, sair da lista e usar a lista em
 http://www.mat.puc-rio.br/~obmlistas/obm-l.html
 =



-- 
/**/
Quadrinista e Taverneiro!

http://tavernadofimdomundo.blogspot.com  Quadrinhos, histórioas e afins
http://baratoeletrico.blogspot.com / Um pouco sobre elétrons em movimento
http://bridget-torres.blogspot.com/  Personal! Do not edit!

=
Instru��es para entrar na lista, sair da lista e usar a lista em
http://www.mat.puc-rio.br/~obmlistas/obm-l.html
=


Re: [obm-l] Prova da OBM-nivel U (problema 6)

2011-01-10 Por tôpico Johann Dirichlet
Aonde eu acho esse cara??


Em 01/01/11, charles9char...@gmail.com escreveu:
 O Leandro Farias fez!



-- 
/**/
Quadrinista e Taverneiro!

http://tavernadofimdomundo.blogspot.com  Quadrinhos, histórioas e afins
http://baratoeletrico.blogspot.com / Um pouco sobre elétrons em movimento
http://bridget-torres.blogspot.com/  Personal! Do not edit!

=
Instru��es para entrar na lista, sair da lista e usar a lista em
http://www.mat.puc-rio.br/~obmlistas/obm-l.html
=


[obm-l] Re: [obm-l] Re: [obm-l] RES: [obm-l] nome de Matemático

2011-03-02 Por tôpico Johann Dirichlet
Quem me contou algo semelhante foi o Tengan ou o Humberto Naves: um
problema que foi resolvido por um aluno, porque ele se atrasou. Depois
ele entregou o trabalho de casa pro professor, que ficou apavorado!
com a notícia. Resolveu ate publica-los em umas revistas.

O nome e esse mesmo, George Dantzig. O cara virou a maior lenda urbana
da Matematica com isso, e a historia e mais lembrada do que a pessoa.

Outro caso curioso foi o de Erdos:

Ele tinha chegado numa universidade, e perguntou O que e isso na lousa?
Era um problema de uma area da qual Erdos nunca ouviu falar. Ademais,
a solução obtida pelos caras daquela faculdade tinha um monte de
folhas escritas.

Erdos fez umas perguntas sobre alguns detalhes que ele não entendia do
problema (algo de terminologia, creio eu), e depois deu uma solução em
poucas linhas.

Tem uma do Eduardo Tengan que eu conto outra hora...


Em 28/02/11, Welma Pereirawelma.pere...@gmail.com escreveu:
 Obrigada Albert. Eu não tenho certeza se história é real pois ouvi de um
 colega há uns anos atrás na universidade.

 2011/2/27 Albert Bouskela bousk...@msn.com

 Olá, Welma,



 Estudo, com razoável afinco, a história da Matemática e a biografia dos
 seus expoentes. Contudo, desconheço completamente o fato pitoresco ao qual
 você se refere.



 O mais próximo dele, que tenho na memória, é atribuído a Gauss: –
 Acredita-se que Gauss, quando estava no ensino básico, por castigo imposto
 pelo seu professor de Aritmética, tenha sido obrigado a somar todos os
 números compreendidos entre 1 e 100 (incluindo-os).  Gauss, então, deduziu
 a
 fórmula da soma dos termos de uma P.A., ao observar que 1+100 = 2+99 =
 3+98
 etc. = soma dos extremos [ a(1)+a(n) ].



 Atenção:

 i.Não há comprovação de que esta história (ou fábula)
 seja verdadeira;

   ii.A fórmula da soma dos termos de uma P.A. já era, há
 muito, conhecida.



 Albert Bouskela

 bousk...@msn.com



 *De:* owner-ob...@mat.puc-rio.br [mailto:owner-ob...@mat.puc-rio.br] *Em
 nome de *Welma Pereira
 *Enviada em:* 27 de fevereiro de 2011 13:07
 *Para:* obm-l@mat.puc-rio.br
 *Assunto:* [obm-l] nome de Matemático



 Olá Pessoal,



 Será que podiam me ajudar? Estou a procura do nome do matemático que
 resolveu um grande problema porque pensou que era lição de casa?



 Agradeço

 Welma




-- 
/**/
神が祝福

Torres

=
Instru��es para entrar na lista, sair da lista e usar a lista em
http://www.mat.puc-rio.br/~obmlistas/obm-l.html
=


Re: [obm-l] raizes

2011-03-24 Por tôpico Johann Dirichlet
Em 21/03/11, marcone augusto araújo
borgesmarconeborge...@hotmail.com escreveu:

 Onde encontro a fórmula para achar as raízes de uma equação do quarto grau?
A princípio, conheço dois métodos, e ambos só servem pra dizer que
existe uma fórmula usando radicais para as raízes de uma equação de
quarto grau.

O mais legal, EMMO, é o método Gugu-Euler:

Reduza a quártica a uma forma do tipo x^4=p*x^2+q*x+r, e escreva
x=sqrt(y1)+sqrt(y2)+sqrt(y3).

Abrindo as expressões feito um doido, você pode agrupar uns termos e
fazer umas suposições acerca dos y's, e descobrir que os y's são
raízes de uma equação de grau 3.
Aí é só resolver esta coisinha fofa que se origina, e pronto!

Estou sendo bem conciso, mas prometo que alguém poderá explicar em
detalhes depois...

   


-- 
/**/
神が祝福

Torres

=
Instru��es para entrar na lista, sair da lista e usar a lista em
http://www.mat.puc-rio.br/~obmlistas/obm-l.html
=


Re: [obm-l] Prova da OBM-nivel U (problema 6)

2011-03-26 Por tôpico Johann Dirichlet
Concordo plenamente, apesar de eu não ter conseguido fazer sozinho.

Depois de uma grata ajuda do Lopes, foi fácil:
1 - Se n é ímpar, a expressão do numerador é uma soma de quadrados,
logo p teria que ser da forma 4k+1
2 - Se n é par, o seu rtaciocínio prova que k é par.

A próxima ideia que tive era saber se, para todo n, existe um k, e vice-versa.

Em 26/03/11, charles9char...@gmail.com escreveu:
 Agora que papirei reciprocidade quadrática sei fazer :

 10^2n + 8*10^n + 1 = (10^n + 1)^2 + 6*10^n = 0 (60*k + 7)-
 (-6*10^n / 60*k + 7) = 1  (Símbolo de Legendre) .

 Agora é só aplicar as propriedades fundamentais ( tentem fazer! ) :

 (a*b / p) = (a/p) * (b/p)   (p primo ímpar)
 (-1 / p) = (-1)^((p-1)/2)
 (2 / p) = (-1)^((p^2 - 1) / 8)
 e o teorema da reciprocidade quadrática.

 Depois vc chega em algo do tipo : n+nk+k = 0 (mod 2) - (n+1)(k+1) = 0 (mod
 2) -
 n e k são pares!

 É incrivelmente fácil pra um problema 6, vcs não acham?



-- 
/**/
神が祝福

Torres

=
Instru��es para entrar na lista, sair da lista e usar a lista em
http://www.mat.puc-rio.br/~obmlistas/obm-l.html
=


[obm-l] Re: [obm-l] Questão de teoria dos números

2011-03-26 Por tôpico Johann Dirichlet
Isto parece óbvio: a parte inteira de uma fração é justamente oquociente na 
divisão euclidiana clássica. Logo, se aumentamos odivisor, o quociente 
naturalmente diminui.
Talvez a parte difícil seja usar álgebra nisso aí...
Em 26/03/11, enniusenn...@bol.com.br escreveu: Caros Colegas, Como podemos 
provar que a parte inteira de n/(2^k) é maior que a parte inteira de 
n/(5^k), para todo inteiro n1 e n maior ou igual a 2^k?  (k é inteiro 
positivo.) Abraços do Ennius. 
= 
Instruções para entrar na lista, sair da lista e usar a lista em 
http://www.mat.puc-rio.br/~obmlistas/obm-l.html 
=

-- /**/神が祝福
Torres
=
Instru��es para entrar na lista, sair da lista e usar a lista em
http://www.mat.puc-rio.br/~obmlistas/obm-l.html
=


[obm-l] Re: [obm-l] Re: [obm-l] Nº de funções sobrejetoras

2011-03-29 Por tôpico Johann Dirichlet
Cara, algo me diz que isso tá errado. Eu lembro de um artigo na antiga
RPM que contava, de um modo meio complicado, quantas funções existem.
Por exemplo, nesta fórmula (um somatório esquisito usando números de
Stirling), se |A|  |B|, tinha que dar 0.

Se pensarmos de B para A, cada elemento de B terá uma flechinha OU
MAIS chegando de elementos de A.

Posso estar errado, mas não dou certeza...

Em 28/03/11, Marcelo Salhab Brogliatomsbro...@gmail.com escreveu:
 Olá, Pedro,
 para cada elemento de B, temos que ter pelo menos um elemento de A que leve
 a ele.
 Logo, para o primeiro elemento de B, temos n opções.
 Para o segundo elemento de B, temos n-1 opções.
 E assim por diante.

 Assim, ficamos com:
 n*(n-1)*(n-2)*...*(n-m+1) = n! / (n-m)! = Arranjo(n, m) = A(n, m)

 Mas, ainda sobram n-m elementos em A.
 Para esses n-m elementos, tanto faz em qual elemento de B eles levam.
 Logo, para cada um desses n-m elementos, temos m opções.
 Ficando com: m^(n-m)

 Portanto, a resposta fica: A(n, m) * m^(n-m) = m^(n-m) * n! / (n-m)!

 Abraços,
 Salhab


 2011/3/26 Pedro Júnior pedromatematic...@gmail.com

 Um colega me propôs o seguinte problema, e não consegui modelar:
 Seja A um conjunto com n elementos e seja B um conjunto com m elementos,
 com n = m. Quantas funções sobrejetoras, f : A -- B, podemos formar?

 --

 Pedro Jerônimo S. de O. Júnior

 Professor de Matemática

 Geo João Pessoa – PB





-- 
/**/
神が祝福

Torres

=
Instru��es para entrar na lista, sair da lista e usar a lista em
http://www.mat.puc-rio.br/~obmlistas/obm-l.html
=


[obm-l] Re: [obm-l] Geometrias não Euclidianas - Como realizar as construções geométricas ?

2011-04-11 Por tôpico Johann Dirichlet
Bem, creio que não tenha muita lógica em falar de construçoes
euclidianas em espaços não-euclidianos. Mas, levando em conta os
postulados, dá pra brincar um pouco (só não espere algo com muito
sentido :) )

De todo modo, achei este site via Google:
http://cs.unm.edu/~joel/NonEuclid/NonEuclid.html

Em 09/04/11, Marcelo Gomeselementos@gmail.com escreveu:
 Caros amigos da lista,

 Estou impressionado coma  falta de material sobre as geometrias elíptica e
 hiperbólica. Estou tentando reproduzir construções geométricas que envolvam
 estas geometrias em softwares como o Geogebra e ReC.

 Gostaria de perguntar aos senhores se conhecem materiais que demonstrem como
 operar as construções (com régua e compasso) destas geometrias. Se alguém
 souber e quiser compartilhar agradeço muito.

 Abraços, Marcelo.



-- 
/**/
神が祝福

Torres

=
Instru��es para entrar na lista, sair da lista e usar a lista em
http://www.mat.puc-rio.br/~obmlistas/obm-l.html
=


Re: [obm-l] Geometria

2011-05-05 Por tôpico Johann Dirichlet
[cuidado! resposta longa e chata detected!!]

Cara, esse tipo de problema eu sempre fiz do mesmo jeito:
trigonometria até enjoar!
Eu sempre preferi desta maneira, pois pra mim usar álgebra é mais
rápido que usar magia. Nem sempre estes truques são reaplicáveis, e
minha mente computeira se acostuma melhor a coisas que podem ser
guardadas para o futuro.

Mas, antes que falem mal, em hipótese nenhuma estou dizendo que
trigonometria é melhor, mas sim que eu tenho esta preferência por
resolver assim. E também concordo que geometria sintética é muito boa,
e que trigonometria não é superpoderosa (que o diga Erdos-Mordell!)

Como exemplo do que estou falando:
Tem um problema da Inglaterra, 1970, do mesmo estilo destes. Eu
resolvi bem rápido com trigonometria, mas tive que pedir ajuda ao
Naoki Sato, do MathLinks (na verdade ele foi o primeiro a responder),
para obter uma solução sintética (usando somente geometria e
construções auxiliares). Para todos os efeitos, ele reconstrói o
desenho usando uma sequência de lemas, e depois prova que este desenho
construído é igual ao do enunciado.

Vejam em
http://www.artofproblemsolving.com/Forum/viewtopic.php?f=47t=329268
e depois tirem suas conclusões...

P.S.: depois eu vou resolver este com trigonometria, haha!

Em 03/05/11, Marcelo Gomeselementos@gmail.com escreveu:
 Olá professor Albert,

 por favor, se for possível, poderia enviar sua solução também para mim ?

 Abraços, Marcelo.

 Em 28 de abril de 2011 18:04, Albert Bouskela bousk...@msn.com escreveu:

 Olá, Nehab e João,

 O trabalho da Silvana é mesmo bem legal, mas...

 Para resolver o problema proposto - o Nehab tem razão: é um dos mais
 clássicos - prefiro fazer um truque mais palatável: construir triângulos
 auxiliares. Estou enviando - através de um arquivo PDF - a solução para o
 e-mail de vocês.

 Sds.,
 Albert Bouskela
 bousk...@msn.com

  -Mensagem original-
  De: owner-ob...@mat.puc-rio.br [mailto:owner-ob...@mat.puc-rio.br] Em
  nome de Carlos Nehab
  Enviada em: 28 de abril de 2011 17:30
  Para: obm-l@mat.puc-rio.br
  Assunto: Re: [obm-l] Geometria
 
  Oi, João,
 
  O seu exercício é um clássico.
  Ai vai a dica. Um trabalho legal da Silvana: você vai gostar.
 
  http://www.mat.puc-rio.br/~hjbortol/complexidade/complexidade-em-
  geometria.pdf
 
  Capítulo 2 a partir da página 28
  Olhe também a página 36.
 
  Abraços,
  Nehab
 
  Em 26/4/2011 20:22, João Maldonado escreveu:
   O seguinte problema está no livro  Geometria I de Morgado, e não sei
  porque  não estou conseguindo resolvê-lo. Sei que a resposta é 30º, se
  alguém  puder ajudar fico grato.
  
   Em um triângulo isósceles ABC, se base BC, o ângulo  vale 20º. P é um
  ponto sobre AB tal que o ângulo PCB = 60º. Q é um ponto em AC tal que
  QBC
  = 50º. Qual a medida do ângulo CPQ?
   []'sJoão
 
  =
  
  Instruções para entrar na lista, sair da lista e usar a lista em
  http://www.mat.puc-rio.br/~obmlistas/obm-l.html
  =
  


 =
 Instruções para entrar na lista, sair da lista e usar a lista em
 http://www.mat.puc-rio.br/~obmlistas/obm-l.html
 =




-- 
/**/
神が祝福

Torres

=
Instru��es para entrar na lista, sair da lista e usar a lista em
http://www.mat.puc-rio.br/~obmlistas/obm-l.html
=


[obm-l] Re: [obm-l] Re: [obm-l] Olimpíada Universitária.

2011-05-06 Por tôpico Johann Dirichlet
Bem, a dica de estudo é a mesma. Eu sugiro que você pegue as
olimpiadas internacionais também (a IMC é muito legal! e serve bem pra
estudar a OBM universitária).

Na verdade, acho que nem mesmo restrição de graduação deve ter. Lembro
de um aluno que tinha um ano a mais de graduação (por ter quebrado a
grade em porções mais digeríveis) e pôde participar. De todo modo, a
Nelly deve saber :-).

Em 25/04/11, Tiagohit0...@gmail.com escreveu:
 Olá, acho que não tem limite de idade, só de anos de graduação.

 Nas olimpíadas universitárias, a teoria que você tem que saber do ensino
 superior são basicamente Cálculo I e Álgebra linear. Mas o melhor jeito de
 estudar, imagino eu, é pegando as provas anteriores (quem sabe as dos níveis
 mais baixos também) e tentar entender as soluções (obviamente, tentando
 trabalhar nos problemas sozinho antes).

 Então o meu conselho é este, não tente seguir uma bibliografia, tente
 trabalhar nos problemas e, assim que certas assuntos surgirem e você
 perceber que não sabe muito bem a teoria, estude esse assunto.

 2011/4/25 Luís Junior jrcarped...@gmail.com

 Olá a todos,

 Gostaria de ouvir a opnião de vcs com relação a esse meu sonho/projeto.
 Sempre gostei de matemática mas frequentemente, na minha vida, um grande
 esforço se fez necessário para que eu alcançasse a média escolar. De fato,
 posso afirmar que sou um aluno abaixo da média e que 'rala' bastante para
 ser mediano. Ontem, tomei conhecimento das Olimpíadas Universitárias.
 Sempre
 tive esse sonho, de me preparar e participar de uma dessas Olimpíadas.
 Pois
 bem, tenho 30 anos e estou no primeiro semestre de um curso universitário
 regular. Procurei pelo regulamento para saber se há um limite de idade
 para
 os participantes mas não encontrei, então esta se torna a minha primeira
 dúvida. Sendo possível a minha participação, então se iniciaria um projeto
 de 5 anos (tempo médio da graduação) que contemplaria a minha preparação e
 participação no evento. Neste ponto, gostaria de saber a opnião de vcs
 sobre
 a possibilidade/dificuldade de empreender um projeto desses e como começar
 (Revisando o conteúdo de 2° grau?, seguindo uma bibliografia específica?,
 contratando um mestre?) visto que não tenho a mínima idéia.
 Agradeço pela atenção e peço desculpas pelo incômodo. Por favor participem
 com sua opnião!


 ~Carpe Diem~

 L.




 --
 Tiago J. Fonseca
 http://legauss.blogspot.com



-- 
/**/
神が祝福

Torres

=
Instru��es para entrar na lista, sair da lista e usar a lista em
http://www.mat.puc-rio.br/~obmlistas/obm-l.html
=


Re: [obm-l] Descobrir formula geral e provar f(n+1)=2f(n) +3

2011-05-09 Por tôpico Johann Dirichlet
Outra maneira:
f(0)=0
f(n+1)=2f(n) +3

Vendo que f(n+1) é quase o dobro de f(n), uma ideia seria obter uma PG.

f(n+1)+C=2f(n) +3+C= 2(f(n)+(C+3)/2)
Se C=(C+3)/2, ou C=3, obtemos uma relacao interessante:

f(n+1)+3=2(f(n)+3).
E isto é uma PG!
O resto segue acima.

Em 06/05/11, Julio Teixeirajcesarp...@gmail.com escreveu:
 Pessoal, a um tempo acho que vi essa questao aki  e por acaso, ontem me
 deparei com ela em alguns foruns, e o pessoal estava com dificuldades..entao
 vou por aki a minha resolucao..

 questao 157 do Vol. 1 da colecao do G. Iezzi - Fundamentos de matematica
 elemtentar

 157 - Seja f uma funcao, definida no conjunto dos numeros naturais, tal que,
 f(n+1)=2f(n) +3
 com f(0) = 0. Achar a formula geral de f(n) e prova-la por inducao..

 equacao: f(n+1)=2f(n) + 3  e f(0)=0

 para..
 n=0 = f(0+1)=2f(0)+3 = f(1)=3
 n=1 = f(1+1)=2f(1)+3 = f(2)=9
 n=2 = f(2+1)=2f(2)+3 = f(3)=21
 n=3 = f(3+1)=2f(3)+3 = f(4)=45
 n=4 = f(4+1)=2f(4)+3 = f(5)=93

 observando os valores retornado pelas imagens e pondo em produto de um fator
 por 3..
 f(1)=3  = f(1)=3*1
 f(2)=9  = f(2)=3*3
 f(3)=21 = f(3)=3*7
 f(4)=45 = f(4)=3*15
 f(5)=93 = f(5)=3*31

 agora observando os segundos fatores dos produtos acima nas imagens...
 comecamos com 1, depois 3, depois 7, e

 assim temos:
 a diferenca entre  3 e 1 = 2
 a diferenca entre  7 e 3 = 4
 a diferenca entre 15 e 7 = 8
 a diferenca entre 31 e 15 = 16

 obrservando essas diferencas, nota-se que temos uma PG, de razao 2, e com o
 primeiro termo sendo igual a 1

 assim a formula ja comeca a ficar evidente.. sendo 3 vezes essas
 diferencas...

 agora se montarmos essa PG, teremos..

 a1 = 1
 a2 = 2
 a3 = 4
 a4 = 8
 a5 = 16

 opa.. entao a proxima observacao a ser feita eh que, com os resultados
 obtidos temos que,por exemplo,
 f(1)=3*( a1 de nossa PG)
 f(2)=3*( a soma de a1 com o a2 de nossa PG)
 f(3)=3*( a soma de a1 com o a2  e a3 de nossa PG)
 f(4)=3*( a soma de a1 com o a2  e a3 e a4 de nossa PG)

 agora a formula do somatorio de nossa PG seria:
 Sn = a1 * (q^n - 1)/ (q - 1)

 onde substituindo, obteriamos:
 2^n -1

 agora deduzimos entao que a formula geral seria: f(n)= 3 * ( 2^n - 1)

 para provarmos por inducao, vamos provar que eh valido para n=1
 f(1) = 3 * ( 2^1 -1)
 f(1) = 3 * ( 1 ) = f(1) = 3   ( OK, provamos para n=1 )

 agora substituimos por n, por um k, qualquer e obtemos:
 f(k)= 3 * (2^k -1)

 agora substituimos por k+1
 f(k+1)= 3 * (2^(k+1) -1)

 ok, agora note que se pegarmos a formula inicial e aplicarmos n=k, obteremos
 o seguinte..
 f(k+1)=2 * f(k) + 3

 ja que obtemos f(k+1) de nossa formula e f(k+1) da formula original, para
 provarmos que descobrimos a formula geral
 entao o resultado de f(k+1), tem que ser igual, assim tb testamos se eh
 valida para qualquer elemento, provando isso para qualquer sucessor de k, ou
 seja (k+1)
 entao temos o seguinte..
 f(k)= 3 * (2^k -1)
 f(k+1)= 3 * (2^(k+1) -1)
 f(k+1)=2 * f(k) + 3

 agora igualando os f(k+1), obtemos..
 2 * f(k) + 3 = 3 * (2^(k+1) -1)
 substituindo f(k), pelo valor conhecido tb.. ( da nossa formula geral )
 2 * (3 * (2^k -1)) + 3 = 3 * (2^(k+1) -1)
 6 * (2^k -1) + 3 = 3 * (2^(k+1)) -3
 agora, dividimos amobs os lados por 3
 2 * (2^k -1) + 1 = 2^(k+1) - 1
 2^(k+1) -2 + 1 = 2^(k+1) - 1
 2^(k+1) - 1 = 2^(k+1) - 1(OK)
 obtemos assim, a nossa prova...



-- 
/**/
神が祝福

Torres

=
Instru��es para entrar na lista, sair da lista e usar a lista em
http://www.mat.puc-rio.br/~obmlistas/obm-l.html
=


Re: [obm-l] produto interno

2011-05-11 Por tôpico Johann Dirichlet
Em 06/05/11, Samuel Wainersswai...@hotmail.com escreveu:

 qual a diferença entre produto hermetiano e produto interno?sempre ouvi
 falar em operador hermetiano, não em produto hermetiano. Eles são a mesma
 coisa?

 Para toda matriz simétrica A, existe uma matriz invertível P tq: A = (P^-1)
 D (P) onde D é diagonal.?Usa isso num teorema que estou lendo, mas é fato?

Bem, é um fato demonstrável; é fácil achar em bons livros de álgebra linear.

Normalmente ele vem depois de uma série de teoremas. Mas o que você
precisa saber para pesquisar é o seguinte:

0 - Duas matrizes A,B são ditas semelhantes se A=P*B*(P^-1) em que P é
uma matriz conveniente.

1 - Uma matriz quadrada A é dita diagonalizável se é semelhante a uma
matriz diagonal.

Teorema: toda matriz simétrica é diagonalizável!

Entre o 1 e o teorema, se costuma apelar para umas teorias fáceis de
aprender e um pouco difíceis de testar ;-P

Topa uma leitura online?
http://joshua.smcvt.edu/linearalgebra/

100% full, disponível para download.

E um review
http://www.randomhacks.net/articles/2007/03/07/hefferon-linear-algebra-review

-- 
/**/
神が祝福

Torres

=
Instru��es para entrar na lista, sair da lista e usar a lista em
http://www.mat.puc-rio.br/~obmlistas/obm-l.html
=


Re: [obm-l] Teorema sobre mediana (OFFTOPIC)

2011-05-13 Por tôpico Johann Dirichlet
Em 13/05/11, Ralph Teixeiraralp...@gmail.com escreveu:
 Isso é legal, né?

 -- A média minimiza a soma dos quadrados dos desvios
 -- A mediana minimiza a soma dos módulos dos desvios.

 Olhando deste jeito, a mediana parece mais natural do que média para
 resumir os dados de uma sequência alíás, vocês já pararam para
 pensar PORQUE a gente usa a média o tempo todo? No fundo no fundo, é
 só por costume, não há uma razão matemática muito forte não... vou
 exagerar um pouco: se eu pudesse, usava a mediana para calcular as
 notas finais dos meus alunos.


A ideia é que a média é um balanço entre perdas e ganhos.
Por exemplo, a média entre 5 e 7 é 6, e 6 perde 1 de 7 mas ganha 1 de 5.
O problema é que a média é muito sensível a variações dos dados.

Antes de eu fugir da facul, já tive professores que usavam média
harmônica para as provas!
Zerou uma prova, já era! Ou tão pior quanto, ao se tirar vários 10, um
5 te jogava pra baixo.

Mas enfim, acho que a média é mais usada porque embute a ideia que eu
exibi acima.

-- 
/**/
神が祝福

Torres

=
Instru��es para entrar na lista, sair da lista e usar a lista em
http://www.mat.puc-rio.br/~obmlistas/obm-l.html
=


Re: [obm-l] Geometria Cone Sul

2011-05-13 Por tôpico Johann Dirichlet
Em 12/05/11, Luís Lopesqed_te...@hotmail.com escreveu:

 Sauda,c~oes,

 Fonte: Treinamento Cone Sul Volume 2.

 Problema 26 p. 135

 H_b , H_c pés das alturas de B e C.
 H ortocentro
 M_a médio de BC
 Gamma Circuncírculo de ABC
 phi Circuncírculo de AH_bH_c
 S segunda interseção de phi com Gamma

 Mostre que S, H, M_a são colineares

Opa! É Geometria Projetiva na cabeça!

Pegue em mãos sua Eureka! 8, dê uma estudada no artigo do Luciano, e
mãos à obra!
Para melhor acompanhar a demonstração, pegue um papel e faça um
desenho caprichado!

Vamos antes tomar um leminha:

Triângulo ABC;
Alturas AH_a, BH_b, CH_c; ortocentro H
Médios M_a,M_b,M_c;
X_a é o ponto comum a BC e H_bH_c

Demonstrar que M_aH e AX_a são perpendiculares!

Vamlá:
Tome o circulo de centro M_a e raio BM_a, chame-o Epsilon. Como BH_bC
e CH_cB são angulos retos, esta circunferencia passa pelos pontos H_b
e H_c.
Vamos dualizar em Epsilon agora.

A reta polar de A é HX_a
A reta polar de X_a é AH.

Portanto, o ponto polar (polo) de AX_a é H.
Logo, M_aH é perpendicular a AX_a.

Seja Y o ponto comum a AX_a e M_aH.
Pelo lema, AYH=90 graus.

Agora, potência de ponto!

AH_bHH_c é cíclico (angulos retos);
CH_bH_cB é cíclico (angulos retos);
ABC é cíclico (triangulo).

X_a é centro radical dos tres circulos acima descritos (é meio óbvio,
mas basta calcular as potencias de ponto em relação a cada par de
circulos).

Seja Z o ponto em que AX_a corta o circulo AH_bHH_c. Por potencia de
ponto, A,Z,B,C são concíclicos, bem como A, Z, H_c, H.
Logo angulo AZH = angulo AH_cH=90 graus = angulo AYH. Portanto, Y=Z, e
tá demonstrado!


 Como fazer? Com geometria sintética de preferência.

Pra variar um pouco, não vou fazer com trigonometria. Mas a ideia é simples:
Demonstrar que M_aH e AX_a são perpendiculares!


 []'s
 Luís

   


-- 
/**/
神が祝福

Torres

=
Instru��es para entrar na lista, sair da lista e usar a lista em
http://www.mat.puc-rio.br/~obmlistas/obm-l.html
=


[obm-l] Re: [obm-l] Re: [obm-l] número primo e soma de quadrados

2011-05-17 Por tôpico Johann Dirichlet
Você encontrará umas três demonstrações bem legais no livro Proofs
from THE BOOK, Martin Aigner e Günter M. Ziegler.



Em 16/05/11, Tiagohit0...@gmail.com escreveu:
 Existem diversas maneiras de demonstrar isso. Algumas delas usando ideias e
 áreas da matemática bem diferentes.

 http://en.wikipedia.org/wiki/Proofs_of_Fermat%27s_theorem_on_sums_of_two_squares

 2011/5/16 marcone augusto araújo borges marconeborge...@hotmail.com

  Todo número primo da forma 4k+1pode ser escrito de uma única maneira como
 soma de dois quadrados.Como demonstrar?




 --
 Tiago J. Fonseca
 http://legauss.blogspot.com



-- 
/**/
神が祝福

Torres

=
Instru��es para entrar na lista, sair da lista e usar a lista em
http://www.mat.puc-rio.br/~obmlistas/obm-l.html
=


Re: [obm-l] problem numero 15 eureka numero 5

2011-05-17 Por tôpico Johann Dirichlet
1 - Enunciado completo,please!

Vou tentar reescrever para deixar mais claro:

Em um conjunto de MN+1 inteiros positivos, postos em ordem crescente,
uma das duas situações abaixo ocorrerá:

-- haverá uma subsequencia de M+1 inteiros, tais nenhum deles é
divisor de algum outro;
--haverá uma subsequencia de N+1 inteiros, tal que cada termo da
subsequencia será divisor do seguinte;


Um exemplo:
Em um conjunto de 16=3*5+1 elementos,
ou há 3+1=4 inteiros tais que nenhum divide outros,
ou há 5+1=6 tais que cada um divide o seguinte.


Acho que você pensou algo como 5 divide 5+1, o que é obviamente falso ;-P

Em 16/05/11, maurikleber araujomaurikle...@hotmail.com escreveu:

 pessoal alguem ai poderia explicar o problema numero 15 (principio das
 gavetas) da eureka numero 5
 onde ele diz n +1 numeros divide o seguinte ele quer dizer que divide o
 sucessor ou qualquer numero depois
 devo estar entendendo errado porque se for divide o sucessor nao tem como
 provar ou ao menos penso ter achado um contra exemplo
 alguem pode explicar sem resolver a questao   


-- 
/**/
神が祝福

Torres

=
Instru��es para entrar na lista, sair da lista e usar a lista em
http://www.mat.puc-rio.br/~obmlistas/obm-l.html
=


[obm-l] Re: [obm-l] FW: Teoria dos números

2011-05-27 Por tôpico Johann Dirichlet
Ce já estudou congruencias? Um bom começo é pegar a Eureka! 2 na
página da OBM, www.obm.org.br (ou comprar da OBM! É baratinho, uma
anuidade de uns 30 reais e uns 4 contos por cada atrasado que quiser).
Anyway, vou tentar deixar fácil...

1)
2^n=(x-1)(x^2+x+1)

Vamos tentar calcular o MDC:
d|x-1
d|x^2+x+1

x =1 (mod d)
x^2+x+1=0 (mod d)

primeira na segunda, d|3. Como d=3 é impossível (potencias de 2 nao
tem fatores 3 :) ), d=1.

Em especial, x-1=1 ou x^2+x+1=1 (ambos sao potencias de 2, e o MDC é
1, logo um deles é 1).
Ou seja, x=2 ou 0. Substitui e chora!

2)
7|4n^2-3
Multiplica por 2
7|8n^2-6=n^2+1+(7n^2-7)

7|n^2+1
Por congruências, é possível provar que basta testar n de 0 a 6.Mas
vou usar descenso infinito.

Teste de 0 a 6 (larga a mão de ser preguiçoso!). Vai falhar (eu acho :) ).

Se funcionar para algum cara maior que 6, seja F o menor dos caras
para os quais funciona (se existe, existe o menor, este é o lema da
boa ordem).

Seja J=F-7. Então J é maior ou igual a 0.
7|(J+7)^2+1=J^2+2*7*J+7^2+1=7*(um termo chato que não interessa)+J^2+1
7|J^2+1

Mas epa! Achei um cara (J) menor que o menor(F)!
E este é um absurdo, que surgiu quando eu disse que funcionava para
algum cara maior que 6!
Então, só faltaria testar para caras menores que 7. Você já testou,
então sabe que não funciona!

É isso.

P.S.: otruque de multiplicar por 2 facilita a vida pacas, mas não
precisava aplica-lo: a ideia do descenso infinito ainda daria conta.


Em 27/05/11, marcone augusto araújo
borgesmarconeborge...@hotmail.com escreveu:





 From: marconeborge...@hotmail.com
 To: obm-l@mat.puc-rio.br
 Subject: Teoria dos números
 Date: Fri, 27 May 2011 12:28:34 +




  1) Mostrar que para nenhum número natural n ,( 2^n)+1 nunca é um
 cubo.

  Pensei:2^n=x^3-1=(x-1)(x^2+x+1).Se eu conseguisse mostrar q
 mdc((x-1,x^2+x+1)=1 e que x-1 e
x^2+x+1 não podem ser cubos ao mesmo tempo,acredito q resolveria a
 questão.
  Tentei outras formas também ,mas não consegui.

  2) Provar q não exiiste número natural n tal q 7 divide 4n^2-3.

   Considerei n= 7k+ 1 ou 7k-1 ou 7k+2 ou 7k-2 ou 7k+3 ou 7k-3 e
 verifiquei q 4n^2-3 não é múltiplo de 7.
   Sei q há outras formas(e talvez mais interessantes).
   


-- 
/**/
神が祝福

Torres

=
Instru��es para entrar na lista, sair da lista e usar a lista em
http://www.mat.puc-rio.br/~obmlistas/obm-l.html
=


[obm-l] Re: [obm-l] FW: Teoria dos números

2011-05-27 Por tôpico Johann Dirichlet
Poxa! O Ralph destruiu minha mensagem! Mas acabei respondendo do mesmo
jeito (ou nao!:))

Em 27/05/11, Johann Dirichletpeterdirich...@gmail.com escreveu:
 Ce já estudou congruencias? Um bom começo é pegar a Eureka! 2 na
 página da OBM, www.obm.org.br (ou comprar da OBM! É baratinho, uma
 anuidade de uns 30 reais e uns 4 contos por cada atrasado que quiser).
 Anyway, vou tentar deixar fácil...

 1)
 2^n=(x-1)(x^2+x+1)

 Vamos tentar calcular o MDC:
 d|x-1
 d|x^2+x+1

 x =1 (mod d)
 x^2+x+1=0 (mod d)

 primeira na segunda, d|3. Como d=3 é impossível (potencias de 2 nao
 tem fatores 3 :) ), d=1.

 Em especial, x-1=1 ou x^2+x+1=1 (ambos sao potencias de 2, e o MDC é
 1, logo um deles é 1).
 Ou seja, x=2 ou 0. Substitui e chora!

 2)
 7|4n^2-3
 Multiplica por 2
 7|8n^2-6=n^2+1+(7n^2-7)

 7|n^2+1
 Por congruências, é possível provar que basta testar n de 0 a 6.Mas
 vou usar descenso infinito.

 Teste de 0 a 6 (larga a mão de ser preguiçoso!). Vai falhar (eu acho :) ).

 Se funcionar para algum cara maior que 6, seja F o menor dos caras
 para os quais funciona (se existe, existe o menor, este é o lema da
 boa ordem).

 Seja J=F-7. Então J é maior ou igual a 0.
 7|(J+7)^2+1=J^2+2*7*J+7^2+1=7*(um termo chato que não interessa)+J^2+1
 7|J^2+1

 Mas epa! Achei um cara (J) menor que o menor(F)!
 E este é um absurdo, que surgiu quando eu disse que funcionava para
 algum cara maior que 6!
 Então, só faltaria testar para caras menores que 7. Você já testou,
 então sabe que não funciona!

 É isso.

 P.S.: otruque de multiplicar por 2 facilita a vida pacas, mas não
 precisava aplica-lo: a ideia do descenso infinito ainda daria conta.


 Em 27/05/11, marcone augusto araújo
 borgesmarconeborge...@hotmail.com escreveu:





 From: marconeborge...@hotmail.com
 To: obm-l@mat.puc-rio.br
 Subject: Teoria dos números
 Date: Fri, 27 May 2011 12:28:34 +




  1) Mostrar que para nenhum número natural n ,( 2^n)+1 nunca é um
 cubo.

  Pensei:2^n=x^3-1=(x-1)(x^2+x+1).Se eu conseguisse mostrar q
 mdc((x-1,x^2+x+1)=1 e que x-1 e
x^2+x+1 não podem ser cubos ao mesmo tempo,acredito q resolveria a
 questão.
  Tentei outras formas também ,mas não consegui.

  2) Provar q não exiiste número natural n tal q 7 divide 4n^2-3.

   Considerei n= 7k+ 1 ou 7k-1 ou 7k+2 ou 7k-2 ou 7k+3 ou 7k-3 e
 verifiquei q 4n^2-3 não é múltiplo de 7.
   Sei q há outras formas(e talvez mais interessantes).
  


 --
 /**/
 神が祝福

 Torres



-- 
/**/
神が祝福

Torres

=
Instru��es para entrar na lista, sair da lista e usar a lista em
http://www.mat.puc-rio.br/~obmlistas/obm-l.html
=


[obm-l] Re: [obm-l] Re: [obm-l] Re: [obm-l] Re: [obm-l] FW: Teoria dos números

2011-05-30 Por tôpico Johann Dirichlet
O que eu posso fazer se eu sou lento e preguiçoso no computador?

Ah, deixa eu ficar ninja no dvorak... huahuahuahua!

Em 27/05/11, Rogerio Ponceabrlw...@gmail.com escreveu:
 Pois e', Dirichlet, o Ralph tem este pessimo habito...
 :)

 []'s
 Rogerio Ponce

 Em 27 de maio de 2011 17:39, Ralph Teixeira ralp...@gmail.com escreveu:

 Yeah! Ninjei de novo! :) :) :) ;)
 2011/5/27 Johann Dirichlet peterdirich...@gmail.com

 Poxa! O Ralph destruiu minha mensagem! Mas acabei respondendo do mesmo
 jeito (ou nao!:))

 Em 27/05/11, Johann Dirichletpeterdirich...@gmail.com escreveu:
  Ce já estudou congruencias? Um bom começo é pegar a Eureka! 2 na
  página da OBM, www.obm.org.br (ou comprar da OBM! É baratinho, uma
  anuidade de uns 30 reais e uns 4 contos por cada atrasado que quiser).
  Anyway, vou tentar deixar fácil...
 
  1)
  2^n=(x-1)(x^2+x+1)
 
  Vamos tentar calcular o MDC:
  d|x-1
  d|x^2+x+1
 
  x =1 (mod d)
  x^2+x+1=0 (mod d)
 
  primeira na segunda, d|3. Como d=3 é impossível (potencias de 2 nao
  tem fatores 3 :) ), d=1.
 
  Em especial, x-1=1 ou x^2+x+1=1 (ambos sao potencias de 2, e o MDC é
  1, logo um deles é 1).
  Ou seja, x=2 ou 0. Substitui e chora!
 
  2)
  7|4n^2-3
  Multiplica por 2
  7|8n^2-6=n^2+1+(7n^2-7)
 
  7|n^2+1
  Por congruências, é possível provar que basta testar n de 0 a 6.Mas
  vou usar descenso infinito.
 
  Teste de 0 a 6 (larga a mão de ser preguiçoso!). Vai falhar (eu acho :)
 ).
 
  Se funcionar para algum cara maior que 6, seja F o menor dos caras
  para os quais funciona (se existe, existe o menor, este é o lema da
  boa ordem).
 
  Seja J=F-7. Então J é maior ou igual a 0.
  7|(J+7)^2+1=J^2+2*7*J+7^2+1=7*(um termo chato que não interessa)+J^2+1
  7|J^2+1
 
  Mas epa! Achei um cara (J) menor que o menor(F)!
  E este é um absurdo, que surgiu quando eu disse que funcionava para
  algum cara maior que 6!
  Então, só faltaria testar para caras menores que 7. Você já testou,
  então sabe que não funciona!
 
  É isso.
 
  P.S.: otruque de multiplicar por 2 facilita a vida pacas, mas não
  precisava aplica-lo: a ideia do descenso infinito ainda daria conta.
 
 
  Em 27/05/11, marcone augusto araújo
  borgesmarconeborge...@hotmail.com escreveu:
 
 
 
 
 
  From: marconeborge...@hotmail.com
  To: obm-l@mat.puc-rio.br
  Subject: Teoria dos números
  Date: Fri, 27 May 2011 12:28:34 +
 
 
 
 
   1) Mostrar que para nenhum número natural n ,( 2^n)+1 nunca é
 um
  cubo.
 
   Pensei:2^n=x^3-1=(x-1)(x^2+x+1).Se eu conseguisse mostrar q
  mdc((x-1,x^2+x+1)=1 e que x-1 e
 x^2+x+1 não podem ser cubos ao mesmo tempo,acredito q
  resolveria
 a
  questão.
   Tentei outras formas também ,mas não consegui.
 
   2) Provar q não exiiste número natural n tal q 7 divide
 4n^2-3.
 
Considerei n= 7k+ 1 ou 7k-1 ou 7k+2 ou 7k-2 ou 7k+3 ou 7k-3
  e
  verifiquei q 4n^2-3 não é múltiplo de 7.
Sei q há outras formas(e talvez mais interessantes).
 
 
 
  --
  /**/
  神が祝福
 
  Torres
 


 --
 /**/
 神が祝福

 Torres

 =
 Instru�ões para entrar na lista, sair da lista e usar a lista em
 http://www.mat.puc-rio.br/~obmlistas/obm-l.html
 =






-- 
/**/
神が祝福

Torres

=
Instru��es para entrar na lista, sair da lista e usar a lista em
http://www.mat.puc-rio.br/~obmlistas/obm-l.html
=


  1   2   >